Es gibt natürlich noch eine dritte Möglichkeit: Nämlich die, dass unsere Quantenmechanik nicht der Weisheit letzter Schluss ist und dass es eine tieferliegende realistische Theorie gibt (die dann hoffentlich auch noch gleich mit dem Messproblem aufräumt). Da jede korrekte Theorie aber entweder nicht-realistisch oder nicht-lokal sein muss, kann man spekulieren, dass eine realistische und tieferliegende Theorie unser Konzept von “lokal” radikal verändern dürfte. Wir müssten dann unsere Ansichten über Raum und Zeit noch mehr revolutionieren, als wir es eh schon getan haben.

 

1 / 2 / 3 / 4 / 5 / 6 / 7

Kommentare (187)

  1. #1 Niels
    10. Oktober 2012

    Sehr schöne, sehr laienverständliche Aufbereitung.

    Mir ist klar, dass in so einem Einführungstext die Verletzung der counterfactual definiteness oder etwa gar der superdeterminism als Auswege aus dem Problem nicht sinnvoll erwähnt werden können.

    Vielleicht wäre es aber doch eine Überlegung wert, die many-worlds / die many-minds -Theorien noch irgendwie einzuarbeiten?
    Diese Interpretationen sind doch ziemlich bekannt und werden ja üblicherweise als lokal und realistisch aufgefasst.
    Das schaffen sie ja bekanntlich mit Hilfe der Annahme einer universelle Wellenfunktion und der Abschaffung des Kollapses der Wellenfunktion. Dadurch wird dann statt der Lokalität oder des Realismus eben die counterfactual definiteness als eine der Ausgangsbedingungen für die Herleitung der Bellschen Ungleichungen aufgegeben.
    Ich weiß aber auch nicht, wie man das jetzt auf Einsteigerniveau erklären könnte. 🙁
    Vielleicht führt die Erwähnung doch nur zu Verwirrung und eine Einarbeitung macht keinen Sinn.

  2. #2 Niels
    10. Oktober 2012

    Was ich noch schreiben wollte:
    Die Sache mit der Relativität der Gleichzeitigkeit beim Kollaps der Wellenfunktion habe ich übrigens noch nirgends so schön klar wie hier bei dir gelesen.

    Und über den Realismus bei many-worlds hab ich auch noch mal nachgedacht.
    Bei der hier behandelten Frage, ob der Mond bzw. ein Elektron auch dann da ist, wenn niemand hinsieht, hilft das allerhöchstens auf extrem abstrakter Ebene weiter, falls überhaupt.
    Du kannst meine Anmerkung also getrost wieder vergessen. 😉

  3. #3 LipLip
    10. Oktober 2012

    Hallo Martin,

    Ich habe die Scienceblogs erst kuerzlich entdeckt und bin als interessierter Laie ganz aus dem Häuschen.
    Zuerst möchte ich mich dafür bedanken, dass sich Autoren wie du die Zeit nehmen, hier QM der interessierten Öffentlichkeit näher zu bringen!

    Ich würde gerne fragen, ob mein Verständnis der Messung richtig ist:

    Aller Informationsübertragung bedarf es doch zumindest die Übertragung eines Photons vom beobachteten Teilchen/Wellenfunktion zum beobachtenden System. Die Photonen dienen als Informationsmedium.

    Ist es folglich der Entzug der Energie des Photons, der die Wellenfunktion zum Kollabieren bringt?

    Viele Grüße

    Lip

  4. #4 Phero
    11. Oktober 2012

    Nein… denn bei dem Kollaps der Wellenfunktion kann keine Information übertragen werden, wie Martin auch erwähnt und in dem verlinkten Artikel erklärt hat. Außerdem kollabieren ja auch Wellenfunktionen von Elektronen usw.

  5. #5 rolak
    11. Oktober 2012

    Schön.
    Insbesondere schön zum Verlinken für Fragende.
    Und zumindest meiner Meinung nach nicht zu lang.

  6. #6 MartinB
    11. Oktober 2012

    @Niels
    Ich habe hier ganz bewusst fast nichts zu den unterschiedlichen Interpretationen geschrieben, sondern wollte nur erklären, warum die Qm nichtlokal oder nicht-realistisch ist.
    In meinen Augen verwendet auch die Viele-Welten-Theorie (oder die many-minds-Theorie) eine Art Kollaps-Annahme, indem sie letztlich ja sagen, dass in jedem einzelnen Universum immer die “klassischen” Observablen beobachtet werden. Oder kennst du eine gute Begründung dafür, warum in den betrachteten Universen niemals ein Eigenzustand zum Operator (Ort1+Ort2) beobachtet werden kann? Wenn es die gibt, habe ich sie bisher nicht gefunden.

    Aber es stimmt schon, es sollte mal jemand allgemeinverständlich was über die vielen Interpretationen schreiben. Ich wüsste da ja jemanden, der sich gut auskennt….

    Ja, das mit der Relativität der Gleichzeitigkeit habe ich in dieser Form überhaupt nirgends bisher gelesen. Es ist mir selbst erst vor ein paar Monaten klar geworden.

    @LipLip
    Nein, das mit der Energie kann man so einfach nicht sagen, Als Beispiel mal dieses schöne Experiment:
    https://de.wikipedia.org/wiki/Wechselwirkungsfreie_Quantenmessung
    Man kann den Begriff der Information natürlich so interpretieren, wie du es hier tust; das ist dann aber eine ganz besondere Art von “Information”. Dazu habe ich mal was in einem anderen Text über Qm geschrieben:
    https://scienceblogs.de/hier-wohnen-drachen/2011/04/09/quantenmechanik-die-beliebtesten-phrasen-und-was-dahinter-steckt/

  7. #7 roel
    *****
    11. Oktober 2012

    @MartinB Schwieriges Thema, einfach dargestellt – aber irgendwie denke ich, dass ab einer bestimmten Größe Kräfte gegen Überlagerungszustände wirken.

    “Gerade in dieser Woche gab es ein Interview im Zeitmagazin, wo irgendeine Schauspielerin, die jetzt einen Film mit Woody Allan gedreht hat, sagt, dass es kein Glück war, sondern dass sie dank Ihrer Lektüre über Talumd und Quantenphysik (huh??) verstanden hat, dass sie ihr Ziel visualisieren musste, um es zu erreichen.”

    Hier machst du aus der Regisseurin Sophie Lellouche eine Schauspielerin. Hier das Originalzitat aus https://www.zeit.de/2012/41/Rettung-Regisseurin-Sophie-Lellouche/seite-2 : “Ich dachte erst, Woody Allens Einverständnis sei einfach Glück. Jetzt weiß ich es besser, weil ich mich viel mit Quantenphysik und dem Talmud beschäftigt habe. Es hat geklappt, weil ich visualisiert habe, wie die Dinge passieren würden. Um mein Leben zu ändern, musste ich akzeptieren, dass ich die Einzige bin, die dafür verantwortlich ist. Wenn ich ein Ziel habe, unternehme ich alles, um dieses Ziel zu erreichen, und leite meine Energie in die richtige Richtung. Ich weiß, dass alles möglich ist, wenn ich es wirklich will. Wenn ich zweifle, wird gar nichts passieren.”

  8. #8 MartinB
    11. Oktober 2012

    @roel
    “irgendwie denke ich, dass ab einer bestimmten Größe Kräfte gegen Überlagerungszustände wirken. ”
    Aber was für kräfte sollen das sein? Und wie genau wirken die? Lassen sie die WF echt kollabieren? Dann sind wir bei Objektiven-Kollaps-Modellen (so wie bei Penrose, da ist es die Gravitation, die den Kollaps verursacht). Das bringt die oben beschriebenen Lokaliätsprobleme mit sich.

    In diesem Text geht es ja nicht darum, welche Interpretation der QM nun richtig ist oder was genau eine Messung konstituiert, sondern um eine möglichst einfache Erklärung zu den Seltsamkeiten der QM, denen keine Interpretation ausweichen kann.

    “Hier machst du aus der Regisseurin Sophie Lellouche eine Schauspielerin”
    Mein Fehler, aber die ist bestimmt in nem Überlagerungszustand, zumal ich sie als Schauspielerin visualisiert habe…

  9. #9 MartinB
    11. Oktober 2012

    PS: Ich habe mal einen Kommentar dort hinterlassen.

  10. #10 Findelkind
    11. Oktober 2012

    Ein hervorragender Artikel, aus dem ich wieder einiges lernen konnte.

    Da die Anwesenheit von Menschen mit Messgeräten sowohl räumlich als auch zeitlich im Universum eine extreme Ausnahme ist, war der Hinweis “Wechselwirkung mit einem “klassischen Objekt”” ein Treffer ins Schwarze! Habe ich so noch nirgends gelesen oder gehört (kann aber auch an mir liegen).

    Komisch, eben hüpft gerade wieder das kleine Monster aus der Sesamstraße, das unbedingt DA sein möchte, durch meine Gedanken.

  11. #11 MartinB
    11. Oktober 2012

    @Findelkind
    “Habe ich so noch nirgends gelesen oder gehört ”
    Ist eigentlich Standard in der Kopenhagener Deutung; wie problematisch das Konzept ist, steht aber nicht in allen Büchern, viele sagen nichts dazu (und viele Physiker, die sich mit Qm beschäftigen, haen anscheinend sogar eine Abneigung, sich mit solchen Fragen überhaupt auseinanderzusetzen.)

  12. #12 Niels
    11. Oktober 2012

    @MartinB

    Ich habe hier ganz bewusst fast nichts zu den unterschiedlichen Interpretationen geschrieben, sondern wollte nur erklären, warum die Qm nichtlokal oder nicht-realistisch ist.

    Mir ging es ja ebenfalls nicht um die Interpretationen, sondern darum, dass die QM eben doch lokal und realistisch sein kann, nämlich
    3) lokal, realistisch und nicht “kontrafaktisch definit (?)”
    oder
    4) lokal, realistisch und “superdeterministisch”.
    (Wobei ich 4 ziemlich fishy finde.)

    Many worlds habe ich nur als sehr bekanntes Beispiel für 3) erwähnt.
    Aber ich hab ja schon selbst eingesehen, dass das hier eigentlich nicht hingehört.

    Oder kennst du eine gute Begründung dafür, warum in den betrachteten Universen niemals ein Eigenzustand zum Operator (Ort1+Ort2) beobachtet werden kann?

    Nein. Ich hab diese Interpretation aber leider immer noch nicht komplett verstanden. Mir kommt es auch langsam so vor, als ob verschiedene Anhänger dieser Theorie im Detail ganz unterschiedliche Vorstellungen haben, die sich teilweise sogar widersprechen.

    Ja, das mit der Relativität der Gleichzeitigkeit habe ich in dieser Form überhaupt nirgends bisher gelesen. Es ist mir selbst erst vor ein paar Monaten klar geworden.

    Mir ist das aufgefallen, nachdem ich mich mal eine Weile ausführlich mit der ART beschäftigt habe und ich mir dann zufällig kurz danach ein Paper über quantenmechanische Kollapstheorien angeschaut habe.
    Manchmal geht das Gehirn sehr seltsame Wege…

  13. #13 Tom
    11. Oktober 2012

    Danke!
    Das ist einer der besten Artikel den ich je zur QM gelesen habe!

  14. #14 MartinB
    11. Oktober 2012

    @Niels
    Was genau kontrafaktisch definit und superdeterministisch ist, habe ich noch nie verstanden (deswegen will ja auch icht ich nen Artikel zu den Interpretationen schreiben (nudge, nudge)).
    “Mir kommt es auch langsam so vor, als ob verschiedene Anhänger dieser Theorie im Detail ganz unterschiedliche Vorstellungen haben, die sich teilweise sogar widersprechen.”
    Ja, das Gefühl habe ich auch manchmal.

    “Manchmal geht das Gehirn sehr seltsame Wege…”
    In der Tat. Ich habe schon so oft über den Kollaps nachgedacht, und bin trotzdem erst jetzt wirklich über dieses Problem gestolpert – kommt wahrscheinlich vom vielen Blogschreiben…

    @Tom
    Danke 🙂

  15. #15 StefanL
    11. Oktober 2012

    Oder kennst du eine gute Begründung dafür, warum in den betrachteten Universen niemals ein Eigenzustand zum Operator (Ort1+Ort2) beobachtet werden kann?

    Ich weiß nicht ob das als Begründung taugt:
    In einem Hilbertraum lassen sich beliebig viele (jedenfalls ab Dimension \geq \aleph_0 ) \infty-dimensionale paarweise orthogonal zueinander stehende Unterräume finden und jeder “Ort” bekommt so einen zugeordnet und das Skalarprodukt zweier solcher Orte ( alls Überlagerung der Wahrscheinlichkeitsfkt. im Phasenraum?) ist nunmal 0. Also gewissermaßen inkohärent. (?)

  16. #16 MartinB
    11. Oktober 2012

    @StefanL
    Dem kann ich nicht folgen. Warum soll ich nicht genausogut das orthogonale Paar x+y und x-y nehmen (wenn x und y die Orte sind)? Sprich: wer entscheidet, welche Basis ich wähle?

  17. #17 Detlef Schulze
    11. Oktober 2012

    Eine Frage:

    Wieso ist es wichtig zu wissen, ob die Wellenfunktion real existiert oder nicht? Wenn sie bei jeder Messung sowieso kollabiert, dann scheint diese Frage irrelevant, weil die Antwort jenseits unserer erfahrbaren (messbaren) Realitaet liegt.
    Ich verstehe auch nicht, warum man sagen kann, dass ein einzelnes Elektron im oberen Beispiel vom Spiegel reflektiert wird UND gleichzeitig ihn durchdringt, solange man nicht misst.
    Und warum gilt das dann nicht fuer klassische stochastische Prozesse. Kann man sagen, dass eine Muenze nach einem Wurf, Kopf UND Zahl gleichzeitig zeigt, solange man nicht nachgeschaut hat? Klassisch gesehen macht das wenig Sinn, da ja eine Entscheidung gefallen ist (die Muenze zeigt entweder Kopf oder Zahl), man sie nur nicht kennt, solange man nicht nachschaut. Und wieso gilt das nicht fuers Elektron? Wieso ist die Interpretation falsch zu sagen, das es entweder reflektiert wurde oder den Spiegel durchdrungen hat?

  18. #18 MartinB
    11. Oktober 2012

    @Detlef
    Vom Standpunkt der Physik aus ist es -momentan – nicht soo wichtig, deswegen interessieren sich diemeisten Physiker auch nicht so sehr für diese Fragen. Es ist eine philosophische Frage – kann es sein, dass die beste Beschreibung der Welt ein Konzept benutzt, dass in der Realität gar keine Entsprechung hat? Und was sagt das über die Realität aus?

    “Kann man sagen, dass eine Muenze nach einem Wurf, Kopf UND Zahl gleichzeitig zeigt, solange man nicht nachgeschaut ha”
    Nein, das ist hier fundamental etwas anderes. Bei der klassischen Münze ist entweder Kopf oder Zahl oben, ich weiß es nur nicht. Ein Quantensystem ist aber tatsächlich in einem Überlagerungszustand. Habe ich oben kurz erwähnt, aber nicht erklärt. Ein bisschen was dazu steht in diesem Text:
    https://scienceblogs.de/hier-wohnen-drachen/2011/04/09/quantenmechanik-die-beliebtesten-phrasen-und-was-dahinter-steckt/
    Die Interpretation, dass das Elektron eindeutig eins von beiden tut, ist deswegen falsch, weil die beiden Möglichkeiten sich beeinflussen – das sieht man zum Beispiel im Doppelspaltexperiment:
    https://scienceblogs.de/hier-wohnen-drachen/2012/10/03/wie-gros-ist-schrodingers-katze/

    Ganz ausführlich behandele ich das auch im letzten Teil der Serie über die Schrödingergleichung (rechts bei Artikelserien klicken).

  19. #19 LipLip
    11. Oktober 2012

    Hallo Martin,

    danke für die Antwort und die Querverweise! Viele neue Informationen – jetzt muss ich ersteinmal denken 🙂

    Eine grundlegende Frage aber hätte ich noch zur Information:

    Bisher bin ich davon ausgegangen, dass alles, was Eigenschaften besitzt, auch Information mit sich trägt – aber das muss nicht stimmen, richtig?

    Meine Frage: Wird Information erst durch die Messung erzeugt oder liest die Messung bereits vorhandene Information aus?

    Beste Grüße,

    Lip

  20. #20 mar o
    11. Oktober 2012

    MartinB:

    Oder kennst du eine gute Begründung dafür, warum in den betrachteten Universen niemals ein Eigenzustand zum Operator (Ort1+Ort2) beobachtet werden kann?

    Das liegt an der Wechselwirkung zwischen System und Messgerät. Die sorgt für Dekohärenz in einer bestimmten Basis. Ein paar Buzzwords dazu dazu: “`environment-induced superselection”‘, “preferred basis”, “pointer basis”

    Habe leider grad wenig Zeit um auf den Rest einzugehen, der Artikel ist eine schöne Diksussionsgrundlage 😉

  21. #21 Niels
    12. Oktober 2012

    @MartinB

    Was genau kontrafaktisch definit und superdeterministisch ist, habe ich noch nie verstanden

    Ja, das wäre auch mal einen eigenen Artikel wert.
    Das ist leider noch so eine Sache, dich ich noch nie irgendwo gut beschrieben oder sogar anständig zusammengefasst gefunden habe.

    deswegen will ja auch icht ich nen Artikel zu den Interpretationen schreiben (nudge, nudge)

    Das wäre für mich wirklich extremst anstrengend und zeitaufwändig. In diesem Bereich hab ich außerdem noch sehr viel zu lernen und wenn ich da dazu etwas schreiben würde, dann nur, wenn ich mir wirklich hunderprozentig sicher bin, dass das Ganze auch stimmt.
    Allein zeitlich ist das momentan überhaupt nicht machbar, selbst wenn ich motiviert wäre.
    Sorry, warte da also lieber nicht drauf.

    @mar o
    Ja, schon klar, aber es wurde doch nach einer guten Begründung gefragt. Diese Begriff kennt MartinB mit Sicherheit ebenfalls.
    Das Stichwort heißt doch in Wirklichkeit “preferred basis problem! Die Lösung, da in geeigneter Weise Dekohärenz durch “einselection” einzuführen, finde ich nicht so wahnsinnig überzeugend und außerdem irgendwie arg künstlich.
    Aber ehrlich gesagt kenne ich mich mit Zureks Quantendarwinismus nicht gut genug aus, um da eine wirklich fundierte Meinung zu haben.
    Vom Bauchgefühl her kommt mir das Ganze aber sehr merkwürdig vor. 😉

    Wobei:
    Wenn die Theorie des Quantendarwinismus zuträfe, hätte das natürlich unglaublich bedeutendere Konsequenzen als nur die Ausräumung einer Schwierigkeit bei der Viele-Welten-Interpretation.
    Das würde schließlich das komplette quantenmechanische Messproblem in jeder Interpretation lösen, oder?

  22. #22 Niels
    12. Oktober 2012

    @mar o
    Sorry, vergessen:

    Das liegt an der Wechselwirkung zwischen System und Messgerät.

    Hm? Das Messgerät spielt bei der Einselection doch keine ausgezeichnete Rolle, oder?

  23. #23 MartinB
    12. Oktober 2012

    @LipLip
    Das mit der Information hängt davon ab, wie du genau information definierst. Nehmen wir mal ein teilchen in einem bestimmten Zustand, zum Beispiel ein Elektrin mit bestimmtem Ort (also mit einer Wellenfunktion, die nur an einem Punkt nicht null ist) . Wenn ich den Ort messe, habe ich dann die Information ausgelesen oder erzeugt?
    Und wie ist es, wenn die Wellenfunktion vorher ausgebreitet ist? Wird die Ortsinformation dann durch die Messung erzeugt?

    @mar o
    Soweit ich das sehe, sind alle diese Kohärenzmodelle keine wirkliche Antwort auf das messproblem, oder sehe ich das falsch?

  24. #24 Ulrich Berger
    12. Oktober 2012

    Hmmm… ich dachte immer, inzwischen wüsste man, dass nicht-lokal alleine nicht ausreicht? https://www.nature.com/nature/journal/v446/n7138/full/nature05677.html

  25. #25 mar o
    12. Oktober 2012

    Niels:

    Das ist leider noch so eine Sache, dich ich noch nie irgendwo gut beschrieben oder sogar anständig zusammengefasst gefunden habe.

    Ja, sowas suche ich auch schon seit längerem. Im Prinzip sind diese ganzen Begriffe ja nicht kompliziert, es sind nur viele und alle stehen in Beziehung zueinander.

    MartinB:

    Soweit ich das sehe, sind alle diese Kohärenzmodelle keine wirkliche Antwort auf das messproblem, oder sehe ich das falsch?

    Das ist richtig, das Messproblem besteht aber auch aus zwei Teilen: deiner Frage nach der Basis und der Frage nach dem Kollaps auf ein einzelnes Basiselement. Die Quantenmechanik offener Systeme erklärt ganz allgemein, wie sich aus der Wechselwirkung zwischen einem System mit wenigen Freiheitsgraden und einem System mit vielen Freiheitsgraden (“Umgebung”) Dekohärenz in einer bestimmten Systembasis ergibt. Zurek war der erste, der dieses Konzept auf den Messprozess übertragen hat.

    Was damit nicht erklärt wird, warum der Zustand überhaupt kollabiert. Und hier wird weiterhin der Input der verschiedenen Interpretationen benötigt.

  26. #26 Niels
    12. Oktober 2012

    @Ulrich Berger
    Die Verletzung der Leggettschen Ungleichung ist zwar durchaus philosophisch interessant, davon ist aber keine einzige der existierenden Interpretationen betroffen.
    Für die bekannteste realistische Interpretation, die bohmsche Mechanik, spielt das jedenfalls schon mal überhaupt keine Rolle. Für die Viele-Welten-Theorie ebenfalls nicht.
    Durch das Experiment wurde erst einmal nur eine bestimmte Klasse von nicht-lokalen Verborgene-Variablen-Interpretationen ausgeschlossen, wobei aber wie gesagt bisher (soweit ich weiß) noch niemand jemals eine derartige Interpretation entwickelt hat.

    Es ist also sehr fragwürdig, ob dieses Experiment wirklich zeigt, dass “certain intuitive features of realism” aufgegeben werden müssen. Da nehmen die Autoren den Mund ziemlich voll.
    Diese speziellen “Eigenschaften” wurden jedenfalls bisher noch nicht als besonders wichtig oder entscheidend angesehen, niemand betrachtete sie als bedeutende Voraussetzung für den Realismus in der QM.
    Einige Forscher glauben sogar, dass diese hypothetischen speziellen, von der Verletzung der leggettschen Ungleichung betroffenen, nichtlokalen realistischen Interpretationen sowieso völlig unplausibel sind. Selbst, wenn sie jetzt nicht ausgeschlossen worden wären, hätte sich deswegen ihrer Meinung nach trotzdem niemand mit ihnen beschäftigt.

    Ums kurz zu fassen:
    Nein, man weiß nicht, dass nicht-lokal alleine nicht ausreicht.

  27. #27 MartinB
    13. Oktober 2012

    @mar o
    Danke für die Zusatzerklärung. Dann habe ich es bisher doch einigermaßen richtig verstanden.

  28. #28 Andreas
    Berlin
    13. Oktober 2012

    Danke für diesen spannenden Artikel

  29. #29 JaCobi
    13. Oktober 2012

    Unser Bild von der “Wirklichkeit”, ist irritiert vom Verdrängen der Tatsache, daß sich alles entweder positiv oder negativ werten läßt. Alternativen dazu gibt es nicht. Das Höhere hinter vielem, das wir gerne vermuten, resultiert aus dem Unwissen unserer Vorfahren, die sich längst verkrümelt haben in der Kohärenzfalle Welt, und ihrem Bedüfrnis, sich Götter hinter Höherem vorzustellen. Heute empfinden wir Höheres gerne als Macht, die aus unserem immer dichteren Zusammenleben und entsprechenden Kooperationseffekten entsteht. Google hat sich sehr weit nach oben geschafft, kann auch was über den “BinärCode-Blubb” liefern, der vielleicht eine Erklärung für etliche Rätsel unserer Welt ist …

  30. #30 Fossilium
    13. Oktober 2012

    Hi Martin
    Die QM ist doch nichts anderes als ein von Physikern ausgedachtes Modell, mit dem Vorhersagen im mikroskop. Bereich möglich gemacht werden. Das Modell, das im wesentlichen aus Geisterteilchen, Unschärfen und und mathematischem Aberwitz (u.a.komplexwertige Funktionen in unendlich-dimensionalen Räumen) besteht, funktioniert ja auch hervorragend, ganze Hight Tech Indistriezweige haben sich daraus entwickelt (Optik, Datenverarb.). Es ist eine hohe Kunst so was fertigzubringen, also ich ziehe den Hut vor den Physikern.
    Die Probleme treten doch erst auf, wenn ich die Geisterteilchen und ihre Schatten in unendlichdimensionalen Räumen, die in den Modellen so gut funktionieren, auf die Realität übertrage, und so tue, als würde das, das ich mir für das Modell ausgedacht habe, auch tatsächlich in der Realität passieren.
    Als ob ich ein Elektron (ei, was ist denn das?) in einen Kasten einsperren könnte, als ob es ein Lichtteilchen gäbe (wird auch schon mal als virtuelles (?) chen bezeichnet ! Was soll das ?

    Jeder stellt sich doch jetzt vor, das das tatsächlich in der Realität so ablaufen könnte.

    Ich kann nur sagen, Physiker, bleib bei Deinen Modellen.
    Die Wirklichkeit kannst Du damit nicht beschreiben.
    Grüsse Fossilium

  31. #31 Gustav
    14. Oktober 2012

    @MartinB: Tatsächlich gibt es Interpretationen der Quantenmechanik, die von einer physikalisch realistischen Wellenfunktion ausgehen. […]

    Zur Bohmschen Mechanik gibts ja ganz hervorragende Literatur.Neben einer recht netten und einfachen Einführung “Bohmsche Mechanik” und vor allem Bells Aufsätze “Sechs mögliche Welten der Quantenmechanik”, beschäftigten sich ja auch die Lehrbücher zur Theoretischen Physik: Quantenmechanik bzw. Relativistische Quantenmechanik vom Rebhan mit der Bohmschen Mechanik (besser gesagt, als Anhänger dieser Interpretation, erklrät Rebhan diese im Anhang genauer).

    Trotz des Schattendaseins der de Broglie–Bohm Theorie gibts also ganz gute Literatur dazu. Und so eindeutig im Vorteil ist die Kopenhagener Deutung im Bezug zur Bohmenschen Mechanik nicht, wie es oft dargestellt wird. Der Vorsprung ist eher historisch bedingt. Das schöne an der Bohmschen Mechanik ist für mich, dass wichtige Teile des Grundverständnis für Physik nicht aufgegeben werden. Eben der Determinismus. Auch wenn der bei der Bohmschen ein ganz ein anderer ist, als der klassische. Die Many-World Interpretation kann ich nicht ernst nehmen, es werden trilliarden bis zu unendlich viele Welten vorhersagt, diese aber als unbeobachtbar deklariert. Das ist keine wissenschaftliche Theorie. Auch die Entgegnung der BefürworterInnen, dass diese Behauptung aus der Schrödinger-Gleichung folgt, ist unzureichend – denn offenbar muss das nicht zwingend aus der Schrödinger-Gleichung folgen, wie ja die Kopenhagener Deutung beweist. Da ist ja die String Theorie noch mehr ernst zu nehmen. 😉

    Egal, eigentlich wollte ich gar nicht soviel zur Bohmschen Mechanik schreiben, sondern vielmehr fragen ob du , MartinB, mit

    “Es führt dazu, dass es Beobachter gibt, für die der Kollaps der Wellenfunktion sich rückwärts in der Zeit ausbreitet – die Wirkung liegt also vor der Ursache”

    die Time-symmetric theories meinst? Die sind deterministisch und die Wellenfunktion sind real. Dabei ist der wichtigste Punkt in diesen Theorien, dass es eine Symmetrie bezüglich der Zeitumkehr gibt und Ereignisse in der Zukunft Ereignisse in der vergangenheit beeinflussen können (daher die Frage zu deinem Satz). Es ist eine spezielle Art von Kausalität, nicht vergleichbar mit der klassischen. Ansich ja kein problem, denn nach Einstein existiert ja die Zeit als Dimension und Vergangenheit, Gegenwart und Zukunft sind nur Phänomene des Zeitfluss. In wie weit, diese Theorien mit der Thermodynamik und Entripoie kollidieren wäre sicher eine spannende Frage. Vielleicht ist dass nur unter ganz bestimmten Vorraussetzungen möglich und es gibt daher die Unterschiedung zwischen quantenmechanischer und klassischer Welt.

    Aber leider finde ich zu dieser Interpretation sehr wenig, um mögliche Fragen dazu zu beantworten. Papers auf arxiv behandeln zwar immer wieder diese Interpretation, aber immer nur einzelne Aspekte. Und da schaue ich meistens nicht durch (wenn ich sie überhaupt verstehe ;-)), welche der mehreren vorhandenen Time-symmetric theories nun angesprochen ist. Auch eine wirklich gute Zusammenfassung dazu habe ich noch nicht gefunden. Es soll zwar eine Zusammenfassung in der Physical Scripta geben, aber ich geb schon zuviel Geld für Physik-Zeitschriften aus, um mir jetzt nur deswegen eine neue teure Peer-Review-Zeitschrift zu kaufen… 😉

    Vielleicht kennst du oder wer andere ja einer, würde mich freuen über eine solche.

  32. #32 MartinB
    14. Oktober 2012

    @Fossilium
    “Die Wirklichkeit kannst Du damit nicht beschreiben.”
    Beschreiben kann ich sie offensichtlich damit, sonst würde es ja nicht funktionieren. Es gibt zwei Möglichkeiten:

    1. Die Elemente der QM (wie Wellenfunktionen) haben eine Entsprechung in der Realität – unschön wegen der nicht-lokalität etc.

    2- Die Elemente der Qm haben keine Entsprechung in der Realität – mindestens genauso unbefriedigend, denn dann stellt sich die Frage, warum diese Beschreibungen so unglaublich gute Vorhersagen machen können.

    Das ist das Dilemma – einfach zu sagen, Möglichkeit 2 muss zutreffen, bringt in meinen Augen nicht viel.

  33. #33 MartinB
    14. Oktober 2012

    @Gustav
    Dein Kommentar hing im Spamfilter (das Link-Limit ist im Moment auf 4 eingestellt).
    Nein, ich sehe auch nicht, dass die Kopenhagener Deutung gegenüber Bohm so schrecklich im Vorteil ist – soweit ich weiß, ist es aber schwierig, die Bohm-Theorie zu einer Feldtheorie zu erweitern, das ist natrülich ein Problem.

    Was die Frage angeht – ehrlich gesagt sind mir “time-symmetric theories” zumindest unter dem Namen nicht bekannt. Dass es Beobachter gibt, für die sich der Kollaps der WF rückwärts in der Zeit ausbreitet, ist aber, wenn ich mich nicht total irre, für jede Theorie mit realistischer WF und Kollaps so, auch für Bohm.

  34. #34 Fossilium
    14. Oktober 2012

    Hi Martin,
    die Elemente der QM (Wellenfunktion) sind komplexwertige Funktionen in einem n-dimensionelan – letzendlich unendlich dimensionalen – Hilbertraum. Was soll denn daran der Wirklichkeit entsprechen ? Wie muss man im Kopf beschaffen sein, um so etwas für real zu halten ?
    Es ist ein Zeichen für Realitätsverlust, so etwas überhaupt in Erwägung zu ziehen.
    Die andere Frage, warum die Modelle so gut funktionieren, ist natürlich sehr berechtigt. Vielleicht liegt das an der Mathematik, statt an der darin enthaltenen Physik. Nur – das war nicht Thema Deines Beitrags, das ist eine ganz andere Frage.
    Hier geht es um “Quantenmechanik und Realität” (s. Titel). Hier wäre erst einmal zu klären, was für eine Realität Du meinst? Die in Deinem Kopf ?, Oder die, auf die wir uns als objektive einigen ? Oder jeweils unsere eigene Raumzeit ? Das müsste erst einmal definiert werden.
    Ich meine es ist eine Hybris, wenn aus der Tatsache, dass von der Qantenmechanik der richtige Zeigerauschlag vorhergesagt wird, auch noch behsauptet wird, dass die den Ausschlag begründende Theorie , die sich bekanntlich nur im Kopf des Beobachters befindet, mit der Wirklichkeit etwas gemein haben könnte.
    Physiker, die scih mit ihren Modellen beschäftigen, geraten damit in kein Dilemma. Nur die, die glauben, mit ihren Modellen die Welt erklären zu können.
    Grüsse Fossilium

  35. #35 MartinB
    14. Oktober 2012

    @Fossilium
    “mit der Wirklichkeit etwas gemein haben könnte.”
    Offensichtlich hat die Theorie mit der Wirklichkeit etwas gemein, sonst würde sie nicht funktionieren. In irgendeiner Form muss es zwangsläufig eine Möglichkeit geben, Aussagen der Theorie auf die Realität abzubilden, das muss für jede funktionierende Theorie so sein.
    Die Frage ist eben, wie direkt diese Abbildung ist. Nehmen wir an, die Objekte der “echten Realität” (sozusagen die Dinge an sich) würden sich nach irgendwelchen total abgefahrenen Regeln verhalten, die überhaupt nichts mit unserer Beschreibung mit den Mitteln der QM zu tun hat. Dann stellt sich doch die Frage um so schärfer, warum die Qm dann so verdammt gut funktioniert. Einfach zu sagen
    “Die Theorie hat mit der Wirklichkeit nichts gemein, weil sie so mathematisch und abgehoben ist, punktum” hilft da überhaupt nicht weiter, das ist letztlich nur eine Kapitulation.

    Zur Frage der Mathematik habe ich ja letztes Jahr ausführlich geschrieben:
    https://scienceblogs.de/hier-wohnen-drachen/2011/08/15/ist-die-natur-mathematisch/

    Und die Aufgabe der Physik ist es nicht, die Welt zu erklären, sondern sie zu beschreiben:
    https://scienceblogs.de/hier-wohnen-drachen/2010/08/31/kann-die-physik-die-welt-erklaren/

    Wenn aber die Beschreibung verdammt gut funktioniert, dann muss man sich eben fragen, was an der Realität das ermöglicht.

  36. #36 Aveneer
    14. Oktober 2012

    Hallo Martin,
    könntest du beobachten und Beobachter genauer definieren?
    Für mich ist/war der Beobachter in der QM, das Objekt, dass dazu führt das ein Teilchen seinen Aufenthaltsort „preisgibt“. Es ist doch das Photon, das das e- “beobachtet”. Nur dann scheint es mir einleuchtend zu sein, zu sagen: Der Mond wäre nicht da, wenn man ihn nicht beobachtet. Aber jede Art der Wechselwirkung zwischen den elementarsten Teilchen, Quarks, Gluonen, Bosonen…, “EM-Feld”… führt zu einer Lokalisierung des Teilchens. Der Mond hält sich “selber” zusammen. Einen externen Beobachter benötigt man nicht. Je höher die „Energie an einem Ort“, desto höher die Anzahl der möglichen Wechselwirkung/desto höher die Anzahl der Wechselwirkungen, desto stärker die Lokalisation.

    Gruß
    Aveneer

  37. #37 MartinB
    14. Oktober 2012

    @Aveneer
    “könntest du beobachten und Beobachter genauer definieren?”
    Nein wenn ich das könnte,würde ich am 10., Dezember in Stockholm Oslo stehen 😉
    “Es ist doch das Photon, das das e- “beobachtet”
    Erst mal nicht – das Photon kommt dann ja auch in einen Überlagerungszustand.
    Es ist gerade nicht so, dass jede Wechselwirkung ziwschen Teilchen selbst schon eine Messung konstituiert und zum Kollaps der Wf führen kann. Das passiert unserer Erfahrung nach erst bei “hinreichend großen” Systemen, aber was “hinreichend groß” bedeutet und wie sich entscheidet, dass eine Zusammensetzung von vielen Teilchen in der Summe “hinreichend groß” wird, das weiß keiner.

    So einfach, wie du es schreibst, ist es (leider) nicht.

  38. #38 Aveneer
    14. Oktober 2012

    Es ist gerade nicht so, dass jede Wechselwirkung zwischen Teilchen selbst schon eine Messung konstituiert und zum Kollaps der Wf führen kann.

    O.k. nicht jede Wechselwirkung führt gleich zu einer “absoluten” Lokalisation (aber ein „Freiheitsgrad“ wird schon eingeschränkt (keine Ahnung ob Freiheitsgrad hier passt)). Ein bisschen weniger frei?
    In meinem Beispiel meinte ich auch eher die Anregung eines e- durch ein Photon.

    BTW: Und reicht eine solche „schwache“ Wechselwirkung die nicht zum Kollaps der Wf führt aus, um z.b. die Interferenz am Doppelspalt zu unterbinden?

    Das passiert unserer Erfahrung nach erst bei “hinreichend großen” Systemen, aber was “hinreichend groß” bedeutet und wie sich entscheidet, dass eine Zusammensetzung von vielen Teilchen in der Summe “hinreichend groß” wird, das weiß keiner.

    Der Mond wäre aber so ein Objekt – Also so von der Größe her 😉

    Nein wenn ich das könnte,würde ich am 10., Dezember in Oslo stehen

    Stehen wir im Dez. nicht alle ein bisschen in Oslo? Du müsstest aber gleichzeitig auch in Stockholm stehen – was dann aber auch erklären könnest 🙂

    PS: Ich verstehe, dass meine Interpretation zu kurz geraten ist, aber ich lebe lieber damit, als mit einer VW-Theorie. Ich lebe damit solange, bis eine vernünftige Lösung gefunden wurde.

    Gruß
    Aveneer

  39. #39 Thomas Wolkanowski
    14. Oktober 2012

    @Fossilium:
    Obwohl die Quantenmechanik kein Modell, sondern eine physikalische Theorie ist, ist deine Kritik zwar berechtigt, jedoch wird es schwer fallen sie auf dem Fundament der Theorie zu fixieren. Aus einem recht einfachen Grund: Viele Physiker sind erkenntnistheoretisch betrachtet wissenschaftliche Realisten. Es wird angenommen, dass eine Wirklichkeit existiert und jede Bestätigung dieser bedingt meist automatisch die Annahme, dass die durch die Theorie als real beschriebenen Entitäten (z.B. Elektronen) tatsächlich in der Wirklichkeit vorhanden sind. In der Tat versteht man genau eine solche Vorstellung unter dem Begriff “physikalische Theorie”; sie gibt eine Beschreibung der Welt wieder und wir nehmen dabei stillschweigend an, dass die bislang beste Theorie auch die Welt so beschreibt, wie sie tatsächlich ist.

    Aber,…

    @Martin: … du machst es dir etwas zu leicht, wenn du sagst, es müsse in jedem Fall eine Korrespondenz zwischen der Theorie und der Wirklichkeit geben. (Sehr wohl sind Theorien vorstellbar, die wenig oder gar nichts mit der Welt um uns herum zu tun haben, beispielsweise die am Ende gescheiterte Phlogistontheorie. Da bringt es auch nichts sich auf “funktionierende” Theorien zu beziehen, denn schlauer ist man stets erst im Nachhinein.) Wie oben geschildert, resultiert eine solche Einstellung aus der Positionierung auf Seiten der Realisten – es gibt gute Argumente dafür und dagegen. Fossiliums Kritik, so scheint mir, findet deshalb aber auf einer anderen Argumentationsebene statt, die einer philosophischen Diskussion bedürfte, welche in diesem Thread jedoch nicht beabsichtigt ist. Auf diesen entscheidenden Punkt mache ich aufmerksam und warne davor, unvereinbare erkenntnistheoretische Standpunkte zu vermengen. Vorher gilt es wesentliche Begriffe zu klären und genau festzustellen, welche Vorstellungen in welche Kategorien einzuordnen sind.

    Wenn du, Fossilium, also an der “wirklichen Komponente” der Quantenmechanik zweifeln möchtest, dann musst du deine Kritik sehr viel tiefer ansetzen (ich meine, das machst du auch, nur ohne es zu merken). Zustände in unendlich-dimensionalen Hilberträumen korrespondieren nämlich sehr wohl “irgendwie” mit der Wirklichkeit – das steckt in den Postulaten der Quantenmechanik. Wie genau eine solche Beziehung aussieht, darüber zerbricht man sich seit Jahrzehnten den Kopf (übrigens wurde u.A. auch hierfür der diesjährige Nobelpreise für Physik verliehen).

  40. #40 MartinB
    14. Oktober 2012

    @Aveneer
    “aber ein „Freiheitsgrad“ wird schon eingeschränkt”
    Nein, wie kommst du darauf? wenn ich ein Elektron z.B. in einer Überlagerung aus Spin rauf und runter habe und es mit einem anderen Teilchen wechselwirkt (verschränkt wird), dann sind beide in einem Überlagerungszustand – und die Amplitude für das Elektron, in einem der Spins zu sein, hat sich nicht geändert.

    “Der Mond wäre aber so ein Objekt”
    Ja, ist er. Deswegen sage ich ja auch, dass das für alle praktischen Zwecke so ist. Wir haben nur keinen Schimmer, wie oder warum es so ist – und es ist eben nicht einfach die “Summe” aus lauter einzelnen Wechselwirkungen.

    @Thomas
    Unsere Theorien mit der Phlogiton-Theorie zu vergleichen, ist schon etwas gewagt – sehr viele gelungene Vorhersagen konnte man damt ja nicht machen, zu vergleichen mit 10 zählenden Stellen für den g-Faktor. Anzunehmen, dass eine solche Theorie so gut mit der Realität übereinstimmen kann, ohne dass es eine irgendwie geartete Entsprechung zwischen den Entitäten der Theorie und den Objekten der Realität gibt, erscheint zumindest mir auch sehr unplausibel.
    Ich vermute vielmehr, dass irgendetwas an unserer grundlegenden Anschauung fehlerhaft ist und dass dieser Fehler zu den konzeptionellen Problemen führt, die wir haben – und ich tippe sehr stark auf das Konzept von Raum und Zeit.

    Und wie oben schon gesagt – mehr als die Welt zu beschreiben, kann die Physik ohnehin nicht leisten (siehe den Link oben). Dass es aber eine Beschreibung der Welt geben soll, die perfekt funktioniert, von der man aber trotzdem sagen kann, dass wir “nichts über die Welt wissen”, das scheint mir eine sehr merkwürdige Einstellung.

  41. #41 Aveneer
    14. Oktober 2012

    “und die Amplitude für das Elektron, in einem der Spins zu sein, hat sich nicht geändert.

    Aber die Aufenthaltswahrscheinlichkeit? Ist diese exakt so wie ohne ein weiteres e-?

    Habe ich nicht bei dir gelesen, dass die Aufenthaltswahrscheinlichkeit nicht unabhängig von anderen Objekten gesehen werden kann?

    Es ist immer die Wechselwirkung des Systems mit der Umgebung.

    A) Angenommen du fährst alleine auf einer Straße und siehst in weiter Entfernung eine Kreuzung an der du links oder rechts abbiegen kannst,…..

    B) Angenommen du fährst auf einer Straße … neben dir ist noch einer und jeder kann nur einen Weg einschlagen…

    Im Fall B würde ich mich irgendwie weniger frei „fühlen“ auch schon bevor ich an die Kreuzung komme…

  42. #42 Fossilium
    14. Oktober 2012

    Hallo Thomas Wolkanowski,
    das klingt ja alles schon ganz versöhnlich, jetzt haben die Objekte der Theorie nur noch eine „Entsprechung“ in der Realtität, oder die Realität (welche auch immer) „ähnelt“ irgendwie der Theorie.

    In dem Beitrag von Martin werden Elektronen aber so beschrieben, als existierten sie in der Realität und als gäbe es daran keinen Zweifel. Jeder der das liest glaubt genau das. Und das zu verhindern war auch garnicht die Absicht des Autors. Dass die Objekte der Modelle oder der Theorien (ich möchte da keinen Unterschied machen) etwas von der Realität (welche auch immer) in sich tragen, dem stimme ich zu. Nur was ist es, und wie nah sind sie dran an der Realität, und an welcher ?

    Physialische Realisten, der Art, wie Sie diese beschreiben, kommen mir vor wie Hoheprister aus einer vergangenen Zeit. Die haben früher den Menschen Unerklärliches in der Natur mit dem Wirken von Hexen, Teufeln Engeln, Feen, Kobolden und dem Satan nahegebracht, und das mit der hohen Autorität der Kirche. Heute wird uns Unsichtbares mit der Existenz von komplexwertigen Wellenfunktionen, virtuellen Teilchen, Lichtteilchen und Quanten, Objekte ohne Substanz, unendlich dimensinierten und verbogenen Räumen erklärt, diesmal mit der hohen Autorität der Wissenschaft. Diese Anmassung einer Deutungshoheit gründet sich dabei garnicht auf Physik, sondern auf Metaphsik. Nur die befasst sich mit dem, was ist.

    Dass es die genannten Objekte mit Berechtigung gibt wird von mir nicht bestritten – eben in den Modellen. Die Objekte sind aber nicht realitätstauglich.

    Die eigentliche Frage, die sich stellt ist doch die: woran liegt es, dass die Modelle so gut funktionieren obwohl die darin vorkommenden Objekte nicht real sein können. Vielleicht lassen sich bei der Klärung dieser Frage die Objekte genauer bestimmen. Dann muss man aber erst mal von diesem kritischen Standpunkt ausgehen,

    Um nicht missverstanden zu werden: ich sehe, dass die Modelle e t w a s über die Wirklichkeit aussagen. Dies herauszufinden mag eine Aufgabe sein, auch für Physiker. Ich wehre mich aber dagegen, dass die Objekte von den Physikern, wie von selbst, in der Realität angesiedelt werden – und in der Umgangssprache und es in Fachartikeln mit der Realitätsferne der Modelle nicht so erst zu nehmen.

    Wenn es aber die Objekte nicht in der Realität gibt, dann werden die philosophischen Probleme der Quantenmechanik – von denen im Beitrag von Martin die Rede ist – tatsächlich als Scheinproblem entlarvt. Die stillschweigene Unterstellung, dass Quanten in der Realität existieren, oder ein vor der Beobachtung existierendes Lichtteilchen über einen geeigenten Detektor gar observiert werden kann. diese unklare Beschreibungsweise ist es, die die Probleme erzeugt. Grüsse Fossilium

  43. #43 Thomas Wolkanowski
    14. Oktober 2012

    Nun, es mag aus heutiger Sicht als unplausibel erscheinen, zu Zeiten ihres größten Einflusses besaß die Phlogistontheorie aber ein vernünftiges Maß an Aussagekraft. Sie wäre sonst kaum jahrzentelang von nahmhaften Größen vertreten und angewendet worden. Es ist nicht ratsam die Ansprüche moderner Wissenschaft, insbesondere die Forderung nach quantitativer Vorhersagekraft, auf damalige Vorstellungen zu übertragen. Jede heutige Theorie, die sich ja als besser herausgestellt hat, wird zwangsläufig gerade auch im Hinblick auf “bezifferbare” Größen viel tollere oder überhaupt Ergebnisse liefern. In damaligen Zeiten war das aber oft nicht gefragt.

    Fossilums “merkwürdige Einstellung” ist sicherlich ziemlich radikal (und mir ebenfalls unverständlich). Sie wäre diskutierbar würde er wenigstens plausible Argumente liefern.

    Was deine Vermutung bezüglich des Konzepts von Raum und Zeit anbelangt, so bin ich ganz bei dir. (Das sieht man auch sofort, wenn man sich die Versuche anschaut, Gravitation im Rahmen der QFT unterzubringen.) Hier sehe ich auch einen viel größeren Nachholbedarf auf Seiten der Philosohie.

  44. #44 Fossilium
    14. Oktober 2012

    Hi Thomas Wolkanowski,
    was stellen Sie sich denn unter einem “plausiblen” Argument vor ?
    Oder anders gefragt: sind Sie der Meinung, dass ein Quant in der Reialität existiert ? Dann beschreiben Sie es doch mal in seiner ganzen Realitätsnähe.
    Grüsse Fossilium

  45. #45 Thomas Wolkanowski
    14. Oktober 2012

    @Fossilium:
    Die als real in einer Theorie angenommenen Entitäten (z.B. Elektronen) werden genauso als existent angenommen wie das Glas Wasser, das auf dem Tisch steht. Das hat nicht unbedingt etwas mit “Entsprechung” oder “Ähnlichkeit” zu tun. Viel interessanter ist die Frage, was jeder einzelne Physiker unter einem Elektron versteht – ich versichere, hier offenbahren sich wesentliche Unterschiede, ebenso wie wenn man die Frage nach der Interpretation der Quantenmechanik stellt. Für die alltägliche Praxis spielt das aber kaum eine Rolle, solange man sich über ein paar grundsätzliche Aspekte von Elektronen einig ist.

    Um es klar zu sagen: Elektronen gibt es wirklich.

    Der Vergleich mit Priestern aus der Vergangenheit ist nur bedingt zulässig – es gibt beispielsweise auf Konferenzen, auf denen Beiträge zur Stringtheorie präsentiert werden, manchmal den Vorwurft, das Vorgestellte hätte nichts mit Physik zu tun. Und in der Tat gibt es so manches Beispiel von aberwitzigem Hokuspokus gerade auch was noch unverstandene Winkel in der Natur betrifft. Insgesamt aber liegen zwischen den Zauberkünsten und Pseudo-Erklärungen aus der Vergangenheit und “harter” Physik aber mehr als Welten. Darüber lässt sich nicht ernsthaft streiten.

  46. #46 Fossilium
    14. Oktober 2012

    Hallo Herr Wolkanowski,
    natürlich spielen die unterschiedlichen Auffassungen über das Elektron in der Praxis keine Rolle. Da geht es ja um Vorhersagen,die das Modell liefert. Die Auffassung ist die beste, die die beste Vorhersage liefert. Es ist ganz egal, ob das Elektron von Experiment zu Experiment ma Welle mal Teilchen ist, solange das Ergebnis stimmt, ist die Welt ja in Ordnung.
    Sie wollen aber doch nicht bestreiten, dass die Problem da anfangen, wenn ich das in seinem Wesen changierende Gebilde anfange als real anzuehen. Vielleicht sieht jeder ein anderes, also ist es beliebig.
    Wenn es denn beliebig ist, ist es auch gut, dann hat es ja wie von mir behauptet keine Bedeutung.
    Sie müssen schon Klartext reden. Es geht mir allein um den Realismusaspekt und die unscharfe Kommunikation darüber in der Öffentlichkeit
    Grüsse Fossilum

  47. #47 Niels
    15. Oktober 2012

    @Gustav

    Aber leider finde ich zu dieser Interpretation sehr wenig, um mögliche Fragen dazu zu beantworten. Papers auf arxiv behandeln zwar immer wieder diese Interpretation, aber immer nur einzelne Aspekte. Und da schaue ich meistens nicht durch (wenn ich sie überhaupt verstehe ), welche der mehreren vorhandenen Time-symmetric theories nun angesprochen ist.

    Die bei weitem bekannteste und älteste „time-symmetric theory“ ist die
    ”Two-state vector formalism” – Sache von Aharonov.
    Mit ein bisschen Glück stehen unter den “further reading” auch tatsächlich die maßgeblichen Arbeiten darüber.
    Um dieses Modell geht es meines Wissens in der Regel, wenn man von einer „time-symmetric theory“ spricht. Das ist die Standard-Interpretation dazu.
    Der in MartinBs Artikel verlinkte Überblickartikel über die Interpretationen der QM in der englischen Wiki handhabt das auch so.
    Interessanterweise ist diese Theorie laut der dortigen Tabelle ebenfalls lokal und trotzdem realistisch.
    Über den Inhalt dieser Interpretation weiß ich aber überhaupt nichts, sorry.

    Es gibt noch eine sehr aktuelle Variante einer solchen Interpretation von C. J. Wharton. In dieser Arbeit stellt er sie vor.
    Hab ich aber ebenfalls nicht gelesen.

    Es soll zwar eine Zusammenfassung in der Physical Scripta geben, aber ich geb schon zuviel Geld für Physik-Zeitschriften aus, um mir jetzt nur deswegen eine neue teure Peer-Review-Zeitschrift zu kaufen…

    In Physica Scripta hab ich auf die Schnelle nichts derartiges gefunden. Aber wenn du den genauen Link auf diese Zusammenfassung hast, kann man da unter Umständen etwas machen… 😉

    @Fossilium

    Es ist ganz egal, ob das Elektron von Experiment zu Experiment ma Welle mal Teilchen ist, solange das Ergebnis stimmt, ist die Welt ja in Ordnung.
    Sie wollen aber doch nicht bestreiten, dass die Problem da anfangen, wenn ich das in seinem Wesen changierende Gebilde anfange als real anzuehen. Vielleicht sieht jeder ein anderes, also ist es beliebig.

    Elektronen “changieren” aber doch gar nicht. Der Welle-Teilchen-Dualismus ist doch nur ein unserem klassischen Denken angepasstes Modell zur anschaulichen Beschreibung. Das “Wesen” des Elektrons verändert sich der physikalischen Theorie zufolge doch gar überhaupt nicht.
    (Richtigerweise müsste man sowieso über Quantenfelder und Feldtheorien sprechen, da hört es dann mit der Anschaulichkeit völlig auf.)

    Dass es die genannten Objekte mit Berechtigung gibt wird von mir nicht bestritten – eben in den Modellen. Die Objekte sind aber nicht realitätstauglich.
    Die eigentliche Frage, die sich stellt ist doch die: woran liegt es, dass die Modelle so gut funktionieren obwohl die darin vorkommenden Objekte nicht real sein können.

    Woher weißt du, was realitätstauglich ist und was real sein kann?
    Wenn die grundlegendsten physikalischen Theorien die Realität nicht beschreiben, dann können wir sie überhaupt nicht beschreiben! Diese Theorien liefern nämlich schlicht und einfach die beste Näherung an die Realität, die uns überhaupt zugänglich ist.

    @MartinB
    Hat Jörg Friedrich auf Arte-Fakten nicht auch mal erklärt, dass es Teilchen gar nicht wirklich gibt?

  48. #48 MartinB
    15. Oktober 2012

    @Aveneer
    “Ist diese exakt so wie ohne ein weiteres e-?”
    Das hängt sicher vom Versuchsaufbau etc. ab. Es ist aber eben einfach nicht korekt zu sagen “Wechselwirkungen schränken die Freiheit/Möglichkeiten für Zustände ein”. Ich kann ein Photon oder Elektron haben, das in einem bestimmten Spinzustand ist und mit keinem anderen Teilchen verschränkt ist und eins, das in exakt demselben Zustand ist und verschränkt ist.

    @Fossilium
    “jetzt haben die Objekte der Theorie nur noch eine „Entsprechung“ in der Realtität,”
    Ich kann hier nicht in jedem Artikel immer die fundamentalen Fragen der Wissenschaftsphilisophie mit einbauen. Ich schreibe auch über Bonobos, und keiner regt sich auf, dass das biologische Artkonzept sehr problematisch ist und dass die Frage, ob es Bonobos in der Realität gibt, ungeklärt ist.

    “obwohl die darin vorkommenden Objekte nicht real sein können.”
    Woheri weißt du das?

  49. #49 Thomas Wolkanowski
    15. Oktober 2012

    @Fossilium:
    Jetzt wurde ich von Ihnen gesiezt… ich verfahre mal aus Höflichkeit genauso, würde aber gerne beim üblichen “du” bleiben.

    Sie haben beanstandet, Martin bzw. Physiker im Allgemeinen würden die Objekte z.B. der Quantenmechanik so verstehen, dass diese auch so (oder so ähnlich) in der Wirklichkeit zu verorten sind. Sie haben – bis auf die berechtigte erkenntnistheoretische Kritik, welche ich kommentiert habe – soweit ich sehe kein Argument anführen können, das die obige Haltung in Bedrängnis bringt. Aussagen wie, “wie muss man im Kopf beschaffen sein…” helfen hier nicht weiter. Sie entspringen einer subjektiven Einstellung.

    Der Begriff “Quant” wird in der Populärwissenschaft inflationär verwendet. Im Kollegenkreis höre ich selten davon – er wird eigentlich immer dazu gebraucht, um die Tatsache auszudrücken, dass eine Größe “quantisiert”, also in bestimmten Portionen vorkommt oder übertragen wird. Sie können zu Photonen auch Quanten sagen, aber mir scheint, dadurch wird die ganze Angelegenheit nur undurchsichtiger. Es ist müßig in den Kommentaren zu einem Blogeintrag ausschweifend darüber zu schreiben – von der realen Existenz von “Quanten” kann man sich durch Experimente überzeugen. Wenn Sie wirklich Zweifel diesbezüglich haben, empfehle ich einen Blick in die Literatur oder einschlägige Internetseiten. Vielleicht hilft das als Einstieg (im letzteren wird auf experimentelle Erkenntnisse eingegangen):

    https://plato.stanford.edu/entries/qm-copenhagen/
    https://th.physik.uni-frankfurt.de/~drischke/Skript_QM_I.pdf

    Mit Ihrer Forderung nach einer “Beschreibung” von Quanten begehen Sie gerade den Fehler, sich greifbarer machen zu wollen was die Theorie aussagt. In der Quantenmechanik haben wir uns ja gerade davon verabschiedet solche Bilder zu machen. Wir machen sie trotzdem, aber das dient nur als Krücke – sie werden niemanden finden, der mit Gewissheit (s)eine Vorstellung (z.B. über Photonen) als der Weisheit letzter Schluss versteht. Experimentalphysiker aus dem Bereich der Optik stellen sich die Vorgänge in ihren Experimenten anders vor als Theoretiker. Am Ende reden Sie aber meist nur über das, was die Theorie hergibt.

    Sie schreiben: “Es geht mir allein um den Realismusaspekt und die unscharfe Kommunikation darüber in der Öffentlichkeit.” Ich kann Ihnen versichern, wir haben einfach selbst genug Probleme mit dem Realismusaspekt (weswegen viele Physiker sich einfach nicht damit auseinandersetzen.). Die Kommunikation mit der Öffentlichkeit ist ein leidiges Thema. Darüber möchte ich mich jetzt nicht aufregen.

    @Martin: Ich verstehe, dass du mit deinem letzten post deutlich machen möchtest, worum es dir hier geht. Niemand verlangt, dass du in jedem Artikel fundamentale Fragen der Wissenschaftsphilosophie beleuchtest (das wäre furchtbar anstrengend, macht alles sehr viel komplizierter und lenkt den Blick vom eigentlichen Thema ab). Aber in einem Artikel wie diesen hier muss man damit rechnen, dass sich Leser (wie Fossilium) mehr wünschen und darauf aufmerksam machen. Mir reicht dein Vermerk, dass es darum nicht geht.

  50. #50 MartinB
    15. Oktober 2012

    @Thomas
    Ich finde es immer seltsam, dass solche fundamentalen Fragen auftauchen, wenn man über Quanten redet, aber nicht, wenn man über Fußbälle redet…

  51. #51 Thomas Wolkanowski
    15. Oktober 2012

    Ich weiß, das ist leider irgendwie zwangsläufig der Fall. Es ist klar, dass sich die gleichen Probleme auch bei Fußbällen offenbaren.

  52. #52 Doug
    15. Oktober 2012

    @MartinB

    Ich darf sie selbst zitieren:

    Die Wirklichkeit gibt es wirklich, der Mond ist auch da, wenn niemand hinsieht

    In den Kommentaren habe ich mich dagegen gewehrt, dass man die Quantenmechanik ohne weiteres so interpretieren kann. Da die Diskussion im Tonfall zum Teil etwas scharf war (teilweise sicherlich auch, weil ich mal wieder dem Motto “tact is for people not witty enough for sarcasm” gefolgt bin…) und da solche Diskussionen es ohnehin für Außenstehende schwer machen den Argumenten zu folgen, will ich hier etwas genauer erklären, warum die Quantenmechanik meiner Ansicht nach sehr wohl eine Menge philosophischen Zündstoff bietet.

    Es ist besagter Zündstoff an dem sie selbst gezündelt haben, also nicht beschweren, wenn Leute dann etwas “fundamental” um die Ecke kommen.

    Aber die Diskussion ist lustig.

  53. #53 Fossilium
    16. Oktober 2012

    Hallo Herr Wolkanowski,

    ich glaube wir missverstehen uns etwas, hier nochmal ein kurzer Text zur Klarstellung:

    Ich stimme zu, dass in Experimenten quantenmechanische Objekte in der Realität festgestellt werden. Diese Objekte treten durch ihre Wirkung auf einen Detektor ja in Erscheinung. Es ist also etwas da ! Aber was ?

    Sie schreiben: „von der realen Existenz von “Quanten” kann man sich durch Experimente überzeugen.“ Das ist eine nachlässige Sprechweise.

    Ich kann mich davon überzeugen, dass in einem Experiment ein Objekt eine Wirkung hat, selbst beobachtet wird es ja nicht. Und die beobachtete Wirkung mag auch nahelegen, von welcher Art das Objekt sein könnte, und man gibt ihm modellhaft eine gewisse Gestalt und Eigenschaften, das kann im Wechselspiel von Theoriebildung und Messungen sehr präzise geschehen. Und wenn dann mit der Theorie, die das Modell genaustens beschreibt, auch noch tatsächlich eintretende Ereignisse vorhergesagt werden können, dann bedeutet das: das Experiment findet ja in der Realität statt, und das Modell funktioniert sehr gut – und daher muss die Theorie nicht nur das Modell, sondern auch die Wirklichkeit selbst beschreiben.

    Genau in diesem Moment, bei dieser Denkweise, kriegen Sie dann dann die Probleme, die Martin beschrieben hat, z.B. lässt das Modell bzw. die beschreibende Theorie Überlagerungszustände zu. Die finden wir aber nicht in der von uns erfahrenen Realität, auch nicht in unserer Anschauung, sie sind nicht mal vorstellbar.

    Dazu schreiben Sie: „wir haben einfach selbst genug Probleme mit dem Realismusaspekt (weswegen viele Physiker sich einfach nicht damit auseinandersetzen.).“ Na klar haben Sie Probleme damit.

    Um es auf den Punkt zu bringen: es ist etwas da in der Wirklichkeit, das in der Theorie / im Modell sehr gut , aber als Realobjekt nicht mehr beschrieben werden kann. Jede Beschreibung führt zu Widersprüchen (oder zur Aufhebung des Kausalgesetzes oder der Lokalität). In genau diesem Sinne sind Quanten, Elektronen und andere quantemechan. Objekte realitätsuntauglich, genauso wie Felder und gekrümmte Räume.

    Ich kann mir nicht vorstellen, dass Sie da widersprechen wollen.

    Ich will hier nicht noch weiter ausgreifen. Das ist nicht nur ein erkenntnistheoretisches Problem, sondern auch ein berufsständiges. Ich meine dass es Physikern gut anstehen würde, nicht immer nur im Nachherein einzuräumen, dass man die im Brustton der Überzeugung beschriebene Realität, so wie sie ist, nicht als Metaphysk missverstehen solle, dass es je eigentlich immer nur um Beschreibungen von Vorgängen ginge, die mit vielen Vorbehalten vorläufig so und so erklärt werden können, wobei man aber noch viele Unsicherheiten habe. Ihr Text macht auch diese Einschränkungen im Nachherein, nachdem ich etwas heftig Tod und Teufel und die Kirche bemüht habe.

    Ich gebe zu, die Art von seinem Metier zu sprechen, verkauft sich schlecht. Aber in dem Dilemma, für unser Handwerk klappern zu müssen, ohne unredlich zu werden, stecken wir alle.

    Grüsse Fossilium

  54. #54 MartinB
    16. Oktober 2012

    @Doug
    Dieser konkrete text sollte ja erst mal die Grundlage dafür erklären, warum man überhaupt ein Problem mit der QM und der Realität haben kann. Die Kritik von Fossilium geht aber ja darauf gar nicht ein – sie wäre genauso (un-?)berechtigt, wenn wir über klassische Physik sprechen würden, das ist mein Punkt.

    @Fossilium
    “Ich meine dass es Physikern gut anstehen würde…”
    Ich meine, dass man an die Physiker zunächst einmal nicht höhere Maßstäbe anlegen sollte als an andere Menschen. Sie verlangen auch nicht von einem Sportreporter, dass er jedesmal erklärt, dass das Konzept “Ball” natürlich nur eine Idealisierung ist, oder dass es den beschworenen “Teamgeist” eigentlich nicht unbedingt gibt. Auch von einem Philosophen erwarten wir nicht bei jedem Satz, den er sagt, erst mal eine Aussage zur allgemeinen Erkenntnistheorie.
    Wenn wir als Physiker in jedem text, den wir schreiben, erst mal einen seitenlangen philosophischen Disclaimer einbauen, dann hilft das niemandem weiter, sondern sorgt nur für Verwirrung. Mein Problem mit dem Text von Florian rührte ja auch nicht daher, dass er Konsequenzen aus der QM ableitete (das hat er völlig korrekt getan und der Eso-Fraktion eine korrekte Absage erteilt), sondern daher, dass er philosophische Statements eingebaut hat, die so nicht haltbar sind.

    Wir können die Welt so beschreiben, als bestünde sie aus Elektronen etc. Darüber besteht hoffentlich Einigkeit. Wir können sie auch mit Hilfe von Wellenfunktionen beschreiben (auch wenn das wie erklärt Probleme mit sich bringt). Für alle Probleme, in denen es darum geht, über für uns wahrnehmbare Phänomene der Welt zu reden, ist das die angemessene Beschreibungsweise und es ist überhaupt nicht hilfreich, hier jedes Mal eine Grundsatzdebatte zum Realismus führen zu wollen. Wir können ja schlecht sagen “Haroche hat seinen Nobelpreis für die Untersuchung von Phänomenen bekommen, die sich konzeptionell und mathematisch mit Hilfe von Photonen beschreiben lassen” etc.

    Und wer – wie Sie es anscheinend tun – der Ansicht ist, die Erklärungen/Beschreibungen der Physik hätten überhaupt nichts mit der Realität zu tun, der muss natürlich irgendwie plausibel machen, waru sie dan so gut funktionieren.

    “sie sind nicht mal vorstellbar.”
    Ich habe kein Problem damit, mir Überlagerungszustände “vorzustellen”.

    “Ich kann mir nicht vorstellen, dass Sie da widersprechen wollen.”
    Natürlich will ich da widersprechen – was soll denn an gekrümmten Räumen problematisch sein?

  55. #55 StefanL
    16. Oktober 2012

    @MartinB
    Sicherlich ist durch F: x –> U_x als eine Abbildung der “Orte” x in den Verband U der Unterräume eines Hilbertraumes H der Lokalitätsaspekt stark betont.Dieser bezieht sich dabei aber auf den jeweils singulären Ort. In diesem Sinne ist “x+y” oder “x-y” irrelevant, da sowohl “x+y” als auch “x-y”, in ihrer rein Ortskoordinaten bezogenen Interpretation, jeweils ein Unterraum U_{x+y} und U_{x-y} zugeordnet wird und nicht als eine Eigenschaft bezogen auf “Linearkombination sein” (dazu weiter unten in U_x). Es sind da eher die logischen “x und y” und “x oder y” Beziehungen die Konstruktionsgrundlage. Die einzige Bedingung(neben der den Zustand des Ortes zu beschreiben) die da an die Unterräume die als Bilder unter F in Frage kommen ist die, daß sie paarweise orthogonal zueinander sind. Im “Raum” R dieser Abbildungen F gibt es in der Tat keine ausgezeichnete (Selektions-)Funktion F, also auch keinen bevorzugten Beobachter. Sofern aber ein F aus R eine Darstellung des Zustandes des Universums” ( ggfs. als Konglomerat ” Universum=Summe von Welten”) ist, sollten alle anderen F die diese Bedingung erfüllen eine Art Isomorphie zueinander aufweisen – andernfalls würden ja unterschiedliche, gleichzeitige, vollständige und exakte Beschreibungen ein und des selben Ortes x im Universum existieren. A priori gibt es außer der Forderung, daß U_x den Zustand des Ortes x beschreibt ja keine weiteren zu berücksichtigende Invarianzen(wie bspw. örtliche Relationen x+y etc, oder?). Verschränkte Orte [x,y] “selektieren”(mittels F([x,y])) andere Teile des Unterraumverbandes als die singulären Orte x oder y, ja bedingen möglicherweise sogar ein eigenes (Welt-)Model je nachdem ob [x,y] als “x und y” oder “x oder y” oder a priori gewissermaßen als atomares Verbandselement auftritt. Die Frage die da am Anfang steht ist ob x, y oder [x,y] als (singulärer,lokaler) Ort gewählt wird. Was ich hier völlig offen laß’, ist ob diese Unterraumselektionen wieder einen Verband ergeben oder nicht. Jedenfalls könnten sich über Verbandsstrukturen u.U. nicht-distributiv Aspekte ggfs. auch einfacher oder fundamentaler darstellen lassen. Sicherlich ist mit dem “Skalarprodukt von F(x),F(y)”=0 nicht unbedingt eine “gute Begründung” gegeben. Einerseits ist das subjektiv (“gut”) und andererseits abhängig davon was unter “(Ort1+Ort2)” zu verstehen sei,als auch ob tatsächlich die Lokalität oder wie realistisch die Begründung ist im Vordergrund stehen soll/muß – die Eigenwerte des 0-Operators sind aber nunmal 0.

    Betrachten wir nun einen U_x. Aufgrund von \Sum_{n=k}^\infty \frac{1}{n} \longrightarrow\infty können in U_x nur endlich viele Komponenten mit Wahrscheinlichkeit ungleich 0 zutreffen. Insofern ist es ja relativ unproblematisch dort eine endliche Basis zu finden. Ob ich die jetzt nehme oder (bspw.)die gemäß den Eigenwerten des Hamiltonoperators (in U_x) ist doch nicht ‘wirklich’ von Bedeutung – vielleicht im konkreten Beobachtungszustand( FAPP) aber nicht i.a. Der wesentliche Aspekt ist doch der, daß hier die Überlagerungszustände des Ortes x beschrieben sind( quasi Psi²(x)=Skalarprodukt(F(x),F(x)) ). Für den “Weltpunkt x” ist U_x ja ein linearer Unterraum von H in dem die Entwicklung von Psi(x)( eigentlich Psi_x(t) oder Psi(x,t) ) sogar mit der x eigenen Eigenzeit stattfindet. Ein Argument die ‘pointer base’ zu präferieren gibt Zurek pg. 6 : Consider, for example, a correlated state |\Psi_SEi = (|0iS |000iE + |1iS |111iE ) /√2 (6) which could have arisen from a sequence of three system-environment c-nots. All errors afflicting individual qubits of the environment can be classified by associating them with Pauli matrices acting on individualqubits of the environment. We can now inquire about the number of errors which would destroy the correlation between various observables of the system and the state of the environment. It is quite obvious that the states {|0iS , |1iS} are in this sense more robustly correlated with the environment than the states {|+iS , |−iS} obtained by Hadamard transform:|\Psi_SE i = |+iS (|000iE + |111iE ) /√2 + |−iS (|000iE − |111iE ) /√2 (7) For, a phase flip of any of the environment bits would destroy the ability of the observer to infer the state of the system in the {|+iS , |−iS} basis. By contrast, a majority vote in a {|0iE , |1iE} basis would still yield a correct answer concerning {|0iS , |1iS} if any single error afflicted the state of the environment. Moreover,when there are N bits in the environment, N/2 −1 errors can be in principle still tolerated in the {|0iS , |1iS} basis, but in the {|+iS , |−iS} basis a simple phase flip continues to have disastrous consequences.
    (sorry für die verhunzten Formeln im Zitat – aber es gibt vielleicht eine Welt in der das funktioniert 🙂 ; btw – irgendwie funktioniert latex-mode auch irgendwie auf Zufallsbasis?)
    Ein weiterer Zugang zum “Basisproblem” über sogenannte “casual diamonds” wird in The Multiverse Interpretation of Quantum Mechanics beschrieben.

    Noch ein paar Worte zum Mond. Abgesehen von der (wohl in allen Modellen) äußerst geringen Wahrscheinlichkeit, daß tatsächlich alle am Mond beteiligten Teilchen (über die Gravitation in einem verschränkten Zustand) synchron in Andromeda sind, wird da nicht gewissermaßen permanent über die Interferenz mit bspw. dem Sonnenwind die Mond-Superposition zum Kollaps gebracht? Also sozusagen die Null-Hypothesen: “Bei Neumond ist es wahrscheinlicher, daß der Mond nicht in Andromeda ist”… oder “Bei einer Mondfinsternis ist es wahrscheinlicher, daß der Mond in Andromeda ist”…

    …und der Disclaimer: sofern nicht explizit angemerkt, geht sämtlicher “insane contents” auf den Autor zurück oder beruht auf Hören-Sagen.

  56. #56 StefanL
    16. Oktober 2012

    Oh – wohin nur ist die Vorschau entschwunden…

  57. #57 MartinB
    16. Oktober 2012

    @StefanL
    Ja, es wird noch am passenden WordPress-plugin gearbeitet…

  58. #58 MartinB
    16. Oktober 2012

    @StefanL
    Das ist mir jetzt so zu heftig – kannst du vielleicht erst mal sagen, worum es geht?

    “wird da nicht gewissermaßen permanent über die Interferenz mit bspw. dem Sonnenwind die Mond-Superposition zum Kollaps gebracht”
    Dazu muss man aber annehmen, dass Wechselwirkungen mit dem Sonnenwind den Kollaps verursachen. Kann man tun – aber was genau ist dann der Mond? Seine WF?

  59. #59 Fossilium
    16. Oktober 2012

    Hi Martin,
    natürlich ist mein Text etwas scharf gehalten und ich lege an Phsiker auch keine höheren Masstäbe an als an andere, im Gegenteil ich hege grosse Sympathie für sie. Der Mensch in seiner Unsicherheit strebt aber nun mal nach abschliessenden Antworten, und die Physiker sind in unserer von den erfolgreichen Naturwissenschaften geprägten Welt gern bereit, diese zu geben, sie werden ja auch als Erste gefragt (früher war es die Kirche). Selbst wenn die Menschen nicht verstehen, welche Objekte angeblich in der Wirklichkeit umherschwirren, werden sie doch sagen, ich kann es mir zwar nicht vorstellen, aber es wird schon stimmen, schliesslich spricht hier ein Physiker, der wird es schon besser wissen. Physiker müssen mit ihrer Macht, eine Deutungshoheit über die Realität zu haben, sehr verantwortlich umgehen. Das mag ja ein Grund sein, weshalb viele Physiker schweigen, weil sie wissen, man kann bei dieser Deutung eigentlich nur missverstanden werden.
    Ich bin dafür, mit der Übertragung der Welt, wie sie sich in der Mathematik und im Kopf des Beobachters darstellt, auf die Realität, auch wenn manches noch so offensichtlich scheint, vorsichtig umzugehen. Erfahrung gründet auf Beobachtung, und wenn etwas der Grunderfahrung widerspricht, ist es nicht oder nie beobachtet worden. Physik muss sich aber an Bobachtungen orientieren. Daher ist diese Übertragung auch methodisch problematisch,
    Aber es gibt wie auf alles in der Welt verschiedene Sichtweisen, Gott sei Dank. Es ist manchmal schwer einen unüblichen Standpunkt klarzumachen, daher meine anfangs etwas provakativen Formulierungen. Ich glaube nicht dass wir letzendlich wirklich konträr sind.
    Warum die Theorie mit ihren absonderlichen Objekten so gute Vorhersagen macht, idas ist wirklich die interessante Frage. Hier nehme ich aus der Diskussion etwas mit.
    Aber deshalb bin ich ja auch hier.
    Grüsse Fossilium.

  60. #60 MartinB
    16. Oktober 2012

    @Fossilium
    Ich denke, man muss sich eben klar machen, dass auch ein Alltagsbegriff wie “Tisch” ein Konstrukt unseres Verstandes ist. Und so muss man eben auch die Äußerungen von Physikern verstehen.

    “Warum die Theorie mit ihren absonderlichen Objekten so gute Vorhersagen macht, idas ist wirklich die interessante Frage.”
    Na klar, das ist der Knackpunkt. Und vermutlich sind wir uns näher als z.B. ich und Florian, der ja kein philosophisches Problem mit der QM sieht.

  61. #61 StefanL
    16. Oktober 2012

    @MartinB – es geht um die “Viele Welten”-Sicht mit der starken Betonung des Lokalitätsprinzips gegenüber der Forderung an eine Theorie/Sicht/Interpretation realistisch zu sein( QM und Realität 😉 ). Und dann um die Frage wie man u.U. tatsächlich eine bevorzugte Basis (im lokalen Zustandsraum eines Objektes) evaluieren könnte(Zurek, casual diamonds) – die von Dir aufgeworfene Frage .
    Neben der imho netten (idealisierten) Vorstellung davon zu sprechen, daß der Mond wahrscheinlicher da ist (nicht in Andromeda) bei Neumond und weniger wahrscheinlich da ist (in Andromeda) bei totaler Mondfinsternis obwohl er (idealisiert) in beiden Fällen nicht zu sehen ist – was ist der Mond? Ich tendiere zu “der Mond ist seine WF und seine Lokalisation”. Die WF als vollständige Beschreibung und die Lokalisation als beobachtbarer Effekt.

  62. #62 MartinB
    17. Oktober 2012

    @StefanL
    Aber wenn der Mond seine WF ist – in welchem Bezugssystem kollabiert die dann? Oder sieht Perry in der Spacejet einen ganz anderen Mond?
    Was genau meinst du mit “Lokalisation”? Den Kollaps der WF in einem begrenzten Ortsbereich?

  63. #63 JaCobi
    17. Oktober 2012

    Hallo,

    ich hatte am 13. SEP eine Frage gestellt. Damit niemand lange suchen muß, zitiere ich sie:

    “Unser Bild von der “Wirklichkeit”, ist irritiert vom Verdrängen der Tatsache, daß sich alles entweder positiv oder negativ werten läßt. Alternativen dazu gibt es nicht. Das Höhere hinter vielem, das wir gerne vermuten, resultiert aus dem Unwissen unserer Vorfahren, die sich längst verkrümelt haben in der Kohärenzfalle Welt, und ihrem Bedüfrnis, sich Götter hinter Höherem vorzustellen. Heute empfinden wir Höheres gerne als Macht, die aus unserem immer dichteren Zusammenleben und entsprechenden Kooperationseffekten entsteht. Google hat sich sehr weit nach oben geschafft, kann auch was über den “BinärCode-Blubb” liefern, der vielleicht eine Erklärung für etliche Rätsel unserer Welt ist …”

    Was kann ich tun, um wenigstens wahrgenommen zu werden?

    HInterfragt man alles, wozu es letztlich geschieht, kommt man tatsächlich auf nichts anderes als auf Wahrnehmung und Nichtwahrnehmung mit sehr vielen Zwischenstufen. Also sind meine Gedanken dazu doch sicher nicht so daneben, oder?

  64. #64 MartinB
    18. Oktober 2012

    “Was kann ich tun, um wenigstens wahrgenommen zu werden? ”
    Um es ganz ehrlich zu sagen: So schreiben, dass man wenigstens eine Idee hat, was du sagen willst und was deine Frage ist (eine Frage kann ich nicht erkennen, ich sehe nur Aussagesätze).

  65. #65 JaCobi
    18. Oktober 2012

    Danke und sorry! Ich dachte, “BinärCode-Blubb” sei so interessant, daß jemand diesen Begriff bei Google eingibt und dann topgelistet im ARD-Forum lesen kann, was dort zu finden ist. Es gibt übrigens nur 8 Fundstellen, die jeweils die selbe “Idee” behandeln. Ich habe mich nicht getraut, hier einen LINK reinzusetzen, traue mich immer noch nicht.

  66. #66 MartinB
    18. Oktober 2012

    @JaCobi
    Du darfst gern einen Link setzen – wenn du willst, dass ihn jemand anklickt, dann hilft es aber wirklich, wenn du uns sagst, worum es geht.

    Und ganz ehrlich: Dein Text ist nicht besonders verständlich, z.B.
    “Das Höhere hinter vielem, das wir gerne vermuten, resultiert aus dem Unwissen unserer Vorfahren, …, und ihrem Bedüfrnis, sich Götter hinter Höherem vorzustellen. ”
    Das Höhere resultiert also aus dem bedürfnis, sich Götter hinter höherem vorzustellen…? Ist das nicht ein Widerspruch?
    Und Begriffe wie “Kohärenzfalle” (was soll denn das bedeuten) tragen auch nicht gerade zum Verständnis bei.

    Bitte nicht böse sein (siehe das Motto “tact is for people…”) aber was du schreibst liest sich ein bisschen so, als hättest du versucht, Dinge zu schreiben, die dadurch, dass sie obskur klingen, schlau erscheinen. Und dazu gehört auch das Wort “BinärCode-Blubb”. Wenn du etwas sagen willst, dann sag es doch so, dass man dich gleich versteht, ohne dass man erst auf die Idee kommen muss, einen Begriff zu googeln.

  67. #67 volker
    Waakirchen
    19. Oktober 2012

    @martin
    Lieber Martin,
    gelegentlich traue ich mich, als einfacher Ingenieur TH ohne Physikstudium zu Wort, insbesondere wenn ich Probleme mit der Begrifflichkeit habe. Ich wäre Dir (oder wem auch immer) sehr dankbar, wenn Ihr meinem Verständnis helfen könntet. Wenn meine Fragen zu banal sind, bitte ich um Nachsicht und Missachtung. Irrtümer bitte richtigstellen, danke.
    Also: Fing das ganze Problem nicht vor mehr als 100 Jahren an mit dem Dualismus von Teilchen-und/oder Wellencharakter des Lichts (EM-Welle).?Heute sagt (weiß) man, dass das Photon ein quantenmechanisches Objekt ist., und den entspr. Regeln folgt. Gut, dazu aber meine 1. Frage:
    Habe ich das richtig verstanden, das das beobachtbare Teilchen aus einer Überlagerung von Quantenfeldern entsteht, bzw. beobachtbar wird. Das wäre dann doch wohl noch in der physikalischen Realität, so wie man sie bisher definiert hat, oder? Wenn man nun von Wellenfunktion spricht, ist das nicht ein Begriff aus der Mathematik, oder im weitesten Sinne auch aus der Logik? Wenn man dann darüber philosophiert, ob auch die Wellenfunktion Realität hat, dann kommt doch sehr schnell der (logische) Gedanke auf, ob nicht die Mathematik, die Naturgesetzte, der “Logos” die dahinterliegende Realität ist, und die von uns bisher beobachtbare Physik nicht nur das Abbild im Raum-Zeitkontinuum? ich bin nicht religiös, aber war nicht doch am Anfang der Logos die Realität?
    Ich weiß, schwierige Fragen an Physiker, aber ihr habt ja auch im blog philosophiert. oder ist das einfach eine frage der Definition von Realität?
    Noch eine doofe Frage: Was ist der physikalische Unterschied zwischen Welle und Strahlung? Unmittelbar nach dem Urknall (?) gab es nur Strahlung. Wo waren denn da die Quantenfelder?

  68. #68 MartinB
    19. Oktober 2012

    @volker
    “Wenn man nun von Wellenfunktion spricht, ist das nicht ein Begriff aus der Mathematik, oder im weitesten Sinne auch aus der Logik?”
    Das stimmt schon. Das ist aber letztlich in der klasssischen Physik auchs chon so, wenn man an Konzepte wie “Energie” oder “Impuls” denkt – gibt es “Energie” wirklich? Was genau bedeutet das? Ähnlich ist es z.B. bei einer em-Welle in der klassischen Elektrodynamik. Die ist ja auch nur eine “Funktion” im Raum.

    In der Qm kommt eben das Problem des Kollaps hinzu.

    “dann kommt doch sehr schnell der (logische) Gedanke auf, ob nicht die Mathematik, die Naturgesetzte, der “Logos” die dahinterliegende Realität ist”
    Es gibt ja solche Ideen, dass jedes Universum, das mathematisch konsistent existieren kann, auch tatsächlich existiert, siehe die Kommentare hier:
    https://scienceblogs.de/hier-wohnen-drachen/2011/08/15/ist-die-natur-mathematisch/
    (am besten nach dem Namen tegmark suchen)

    “ist das einfach eine frage der Definition von Realität?”
    In gewisser Weise sicher. Wenn “Realität” etwas beobachterunabhängig existierendes sein soll, dann macht es eben gewisse probleme, die WF als “real” anzusehen.

    “Was ist der physikalische Unterschied zwischen Welle und Strahlung?”
    Ich weiß nicht ob das ganz scharf definiert ist.
    “Wo waren denn da die Quantenfelder?”
    Die Quantenfelder sind das fundamentale – “Strahlung” gibt es auch nur mit Quantenfeldern. (“Strahlung” ist letztlich eine bestimmte Anregung von Quantenfeldern.)

  69. #69 volker
    waakirchen
    19. Oktober 2012

    @ martin
    Herzlichen Dank für prompte Antwort und Hinweis auf Marc Tegmarks Veröffentlichung “The Mathematical Universe” Ich habe in den nächsten 10 Tagen im Krankenhaus genügend Zeit zu lesen und in Ruhe darüber nachzudenken, darauf freue ich mich schon. Zusammen mit Roger Penrose könnte das eine Vorstellung von der Realität geben, wie sie mir nahekommt. Leider bin ich schon 70, sodass ich bestenfalls von Wolke 7 aus mitkriegen werde, wie das ausgeht, aber spannend bleibt es allemal! Danke, Volker

  70. #70 MartinB
    20. Oktober 2012

    Hallo Volker,
    dann viel Spaß beim Lesen und ich drück die Daumen, dass die Ärzte dich wieder gut zusammenflicken.

  71. #71 Fossilium
    20. Oktober 2012

    Hi Volker,

    „ist das einfach eine frage der Definition von Realität?”

    Ja, vieles in der Physik ist eine Frage der Realität, in dem Sinne, dass bei allen Aussagen der Physiker immer eine bestimmte Realität a priori vorausgestzt wird, was aber nicht explizit benannt wird. Ich bewundere Dich, dass Du in Deinem Alter noch so neugierig bist, daher gebe ich mir hier die Mühe zu etwas ausführlicheren Gedanken:

    Was Realität ist, drüber streiten sich die Philosophen. Aus der Sicht der Physik lässt sich folgendes sagen:

    a) die Welt ausserhalb von uns wird von unseren körperlichen Sensoren in Form von Reizen erfasst, die als elektrische Signale in unser Gehirn geleitet werden. Aus diesen Signalen formen wir ein Bild der Welt da draussen, und zwar in unsem Kopf. Die Welt wird also nur in unserem Kopf als Abbild wahrgenommen, ist also insofern eine Imagination. Da alle Menschen im Prinzip die gleiche Imagination haben, können sie sich darauf verständigen, dass das Abbild eine äussere Welt wiederspiegelt. Die kollektive Einbildung wird also nur einheitlich gedeutet – wir tun eben alle so, als ob diese äussere Welt tatsächlich existiert. Solange das keine Probleme macht, ist das o.k. Man muss sich nur gelegentlich daran erinnern, dass die äussere Welt nur durch Konvention entstanden ist. Da hast Du also das erste Problem: ob und wie sie tatsächlich existiert, können wir nie und niemals in Erfahrung bringen.
    b) Selbst wenn wir also mal davon ausgehen, dass es die äussere Welt gibt, so ist sie keinesfalls für alle Menschen gleich. Die Relativitätstheorie von Einstein sagt nämlich, dass es kein für alle Menschen gültiges Bezugssytem gibt. Jeder hat sein eigenes Bezugssystem, oder lax gesagt: jeder lebt in seinem eigenen. Wenn ich mich realtiv zu Dir bewege, dann vergeht für mich die Zeit langsamer als für Dich in Deinem Krankenbett. Menschen in schnellen Flugzeugen altern langsamer (im Vergleich zu denen am Boden), sie sehen auch alle Abstände, die rechtwinklig zur Bewegungsrichtung liegen, in Bewegungsrichtung verkürzt. Daher hat jeder seine eigene Raumzeit im wahrsten Sinn des Wortes. Jeder bekommt auch andere Messergebnisse, denn am gleichen Objekt gemessen beobachtet der sich Bewegende eine andere Energie und einen anderen Impuls des Objektes, als der, der mit dem Objekt ruht. Energie und Impuls beiben beim Wechsel von einem Bezugssystem zum anderen nicht erhalten. Das heisst, die Beobachtungen am gleichen Objekt sind tatsächlich für beide anders ! Wir können insofern schon nicht von einer objektiven Welt sprechen (Problem 2a mit der Realität.
    Aber – so könnte man meinen – das ist nicht so schlimm, da die Differenzen ja erst bei sehr schnellen Relaitivbewegungen auftreten. Aber wenn das nicht auch mal ein Irrtum ist. Denn in unserem Alltag kommen sehr grosse Geschwindigkeiten vor, das Licht zum Beispiel, das hat – wie der Name schon sagt – Lichtgeschwindigkeit (LG), und das Licht ist überall. Die Relativitätstheorie (RT) sagt nun – Gott sei Dank – über lichtschnelle Bezugssystem nichts aus, die sind sozusagen singulär, das Bezugssysstem des Lichts wird quasi von der RT nicht erfasst. Das gilt nicht nur für das Licht – alle Wirkungen breiten sich mit LG aus, Wirkungen sind damit aus der RT „ausgeklammert“, für sie gilt vielleicht gar keine Theorie – und siehe da, das sind wir schon mitten drin im Problem Nr. 2b mit der Realität. Denn Wirkung ist verantwortlich für den stetigen Wandel in der Realität, aber keine physikalische Theorie erfasst das Bezugssystem der Wirkung. Ja kann es denn Wirkung ohne Bezugssystem geben ?
    Schon verheddern wir uns vollständig mit unserem Wirklichkeitsbegriff.
    c) Problem Nr. 3: Da sich beim Wechsel des Bezugssystems die grundlegenden Grössen Energie und Impuls ändern, sind für Physiker genau die Grössen so interessant, die bei einem Wechsel des Bezugssystems invariant beiben – die sich also nicht verändern. Was bleibt invariant ? Der Vierer-Energie-Impulsvektor. Der bleibt aber nur deshalb invariant, weil bei einem Wechsel zwischen zwei Bezugssystemen die Änderung der Energie genauso gross ist wie die des Impulsbetrages, diese Änderungen heben sich gegenseitig auf. Die Wirklung nun wird in der Physik als zeitliche Änderung der Energie oder als örtliche Änderung des Impulses (in einer Richtung) definiert.
    Die so definierte Wirkung ist daher als Potiential des Vierer-Energie-Impulsvektors aufzufassen – und dieses Potential – also die Wirkung selbst – ist die einzige wirkliche invariante Grösse, die bei allen Wechseln der Bezugssysteme erhalten bleibt.
    Sieh da, es gibt also eine Erhältungsgrösse, die sich der Relativität der Einsteinschen Theorie entzieht. Die Werte der Wirkung, oder die Werte des skalares Wirkungfeld als Potentialfeld des Vierer-Energie-Impuls-Vektors, sind sozusagen raumzeitunabhängig. Sie sind invariant beim Wechsel zwischen Bezugssystem. Sie haben kein im Rahmen der RT Einsteins definierbares Bezugssystem. Für unser Realitätsproblem bedeutet das, das es offenbar ein ausserhalb von Raum und Zeit liegendes Wirklungsgefüge existiert.
    Das macht die Energie und den Impuls von Objekten bestimmbar, aber ansonsten äussert es sich nicht als beobachtbar.
    Man kommt hier an die Grenze zur Metaphysik, und dann hören wir Physikar dann auf, und denken im Geheimen als Metaphysiker weiter.
    Mit Deinem Logos liegst Du vielleicht garnicht so falsch.
    Lieber Volker, ich habe den Text hier für Dich verfasst, damit Du im Krankenbett ein wenig Stoff zum Nachdenken hast.
    Lieber Martin, vielleicht kannst Du das Thema nochmal aufrollen, würde mich auch über eine Antwort von Dir freuen.
    Grüsse Fossilium

  72. #72 volker
    21. Oktober 2012

    @Fossilium @martin
    “…damit Du im Krankenbett ein wenig Stoff zum nachdenken hast…” Danke an Martin und Fossilium für Eure heftigen Anregungen meines brain-systems.
    Hoffentlich sehen die docs nicht auch noch Anlass, mich wegen Überhitzung des Hirns (Rauch aus den Ohren) zu behandeln. In der klassischen Annahme zur Realität: “da wo Rauch (Wirkung) ist, muss auch ein Feuer (als Ursache) dahinter sein. Jedenfalls habe ich viel zu lesen, nochmal zu lesen, und nachzudenken. Also nochmals, merci beaucoup,
    Volker

  73. #73 MartinB
    21. Oktober 2012

    @Fossilium
    “…Da hast Du also das erste Problem: ob und wie sie tatsächlich existiert, können wir nie und niemals in Erfahrung bringen.”
    Für mich steckt da ein innerer Widerspruch in deiner Aussage: Wenn die Welt nur Konvention ist, dann steltl sich nämlich die Frage “Konvention von wem”? Denn auch über die “anderen Menschen” erfahre ich ja nur etwas über die Sinne – wenn also die Welt nur ein Geisteskonstrukt ist, dann sind es die anderen Menschen auch.

    Wenn du aber annimmst, dass es die anderen Menschen tatsächlich gibt und dass wir uns erfolgreich über das Bild der Welt in unseren Köpfen verständigen können, dann stellt sich natürlich sofort die Frage, warum dieses Weltbild für uns alle (näherungsweise) gleich ist. Das kann natürlichzum Teil daran liegen, dass wir alle schon a-priori-Denkstrukturen mitbringen (beispielsweise das Konzept der Kausalität), die uns gemeinsam sind, und von denen wir uns nicht lösen können. Aber nichtsdesto trotz muss es natürlich auch irgendwelche äußeren Dinge geben, an diese a-priori-Konzepte sich knüpfen (siehe der erste Satz der “kritik der reinen Vernunft”). Und solange wir nicht annehmen, dass wir in Wahrheit gar nicht aus Materie bestehen, müssen wir uns natürlich auch fragen, was die materielle Basis dieser a-priori-Konzepte ist.

    “Daher hat jeder seine eigene Raumzeit im wahrsten Sinn des Wortes.”
    Nein. Die Raumzeit haben wir alle gemeinsam – jeder zerteilt sie nur anders in Raum und Zeit. Wenn das hypothetische Volk der Kompassis als HauptHimmelsrichtungen NO, NW, SO und SW hat, dann sind wir uns zwar nicht einig, wie man Wege im jeweiligen Bezugssystem beschreibt, über die Form der Wege etc. sind wir uns aber alle einig.

    “Wir können insofern schon nicht von einer objektiven Welt sprechen”
    Doch, im Rahmen der klassischen RT können wir das, da gibt es kein Problem, genauso wie es kein Problem ist, von der objektiven Realität eines hauses auszugehen, auch wenn ich die Vordertür und du die Terasse auf der Rückseite siehst. Wir haben nur unterschiedliche Perspektiven, die sich aber widerspruchsfrei zusammenfügen. (Das Realitätsproblem kommt erst mit der Quantenmechanik dazu.)

    DEn Punkt c habe ich nicht nachvollziehen können:
    “Die Wirklung nun wird in der Physik als zeitliche Änderung der Energie oder als örtliche Änderung des Impulses (in einer Richtung) definiert.”
    ??? Die Wirkung ist doch nicht dE/dt oder dp/dx, sondern Integral Lagrangefunktion dt

    Aber ich glaube eine Debatte über den Begriff “Wirkung” hatten wir schon mal, oder?

    @Volker
    Falls du noch anregende Krankenhauslektüre brauchst: Ich empfehle zum Thema “Bewusstsein” Dennetts “Consciousness explained” und natürlich – falls du es nicht längst kennst – Hofstadters “Gödel Escher bach” (auch nach 30 Jahren immer noch super, auch wenn die Neurobiologie-Teile naturgemäß etwas veraltet sind). Und generel in Sachen Physik natürlich das von mir schon letztes jahr ausführlich empfohlene Buch von Feynman “Vom Wesen physikalischer Gesetze”.

  74. #74 StefanL
    21. Oktober 2012

    @MartinB
    Was genau meinst du mit “Lokalisation”? Den Kollaps der WF in einem begrenzten Ortsbereich?
    Ja – etwa wie Einstein beim photoelektrischen Effekt: Wiki:„in Raumpunkten lokalisierten Energiequanten,…”. Bzgl. des Kollapses selbst finde ich die Idee des “dynamischen Kollaps”
    interessant – sie läßt so wie ich das verstehe ja die Möglichkeiten des Kollaps durch (externe) Wechselwirkung, selbstbezüglich (bspw.) durch Eigengravitation oder als eine Art Resonanzbruch( ähnlich einer spontanen Symmetriebrechung?) der Wellen/Wahrscheinlichkeitsfunktion selbst zu. Nur externe Wechselwirkung erleichtert ja auch eine QM der WIMPs nicht gerade…(oder wird dann doch ein String-theoretischer Zugang benötigt?).
    “Aber wenn der Mond seine WF ist -“
    Meine ich so nur zum Teil – “zum Mond gehören”/”der Mond ist” die WF und seine Lokalisation. Die WF allein ist noch nicht der Mond außer sie ist identisch mit (so etwas wie) der (Dirac-)Delta-Distribution dann wäre die Lokalisation auch umgekehrt mit der WF identisch. So ist möglicherweise der Knackpunkt eben genau die Isoliertheit(/Abgeschlossenheit) eines Systems um ‘nur’ die WF zu haben.
    “…in welchem Bezugssystem kollabiert die dann?”
    salopp gesagt in jedem in dem der Mond ist. Die Frage ist dabei – unter dem Modell der Unterräume – in welchem Unterraum( oder “Universum/Welt”) der Mond überhaupt ist. Imho sieht es auch so aus, als ob da die “Eigenzeit” nicht unwesentlich wäre. Gibt es da eigentlich was zu “Synchronisation unterschiedlicher Eigenzeiten als WF Kollaps Trigger” o.ä.? Oder ist das völlig unsinnig?
    “Oder sieht Perry in der Spacejet einen ganz anderen Mond?
    Als erster Gedanke kam mir dazu: “das hängt wohl davon ab wie schnell der gute Perry gucken kann”. Wenn er mehr als 4 Mrd. LJ (Entfernung) schafft wird er in seinen ihm lokal zugänglichen Bezugssystemen bestenfalls die Möglichkeit unseres Mondes (Luna)in den WF(s) erahnen. Vielleicht wird auch die Auffassung/Verständnis von Lokalität herausgefordert, z.Bsp: Verschränkte Photonen sind im U_v und in diesem Sinne am selben Ort “v”. Im Moment der Wechselwirkung mit (e.g.) U_x ‘entsteht’ die ‘Welt’ U_[v,x] mit dem Ort [v,x] (ob das kommutativ ist laß’ ich mal im Sinne der Grundidee außen vor), d.h. eine Messung bei x von U_v bestimmt zwar U_[v,x] aber erst mit einer Messung/Wechselwirkung bei y ist die Welt mit U_[y,x,v] ‘da’ und möglicherweise durch c= const. zerfällt dies dann unmittelbar in die räumlich getrennte Realisierung/Welt von U_[x,v] = U_(x’) und U_[y,v]= U_(y’). Ähnlich erscheint mir da auch bei der Annahme einer diskreten Raumzeitquantelung die “Überwindung der Strecke zwischen zwei Raum(zeit)-quanten” – ein “hindurchschlängeln” durch diese “Lücken” würde ja auch einer “Verbindung” zwischen zwei solcher Raumzeitquanten nahekommen. Für makroskopische Objekte über “weite Entfernungen” aber wohl eine äußerst geringe Wahrscheinlichkeit eine gleiche “Eigenzeitkonfiguration” des Gesamtobjektes über QM-mechanismen zu erhalten ohne sich vorher im Gitternetz der Raumzeitquanten zu verfangen.
    Und die Frage, wenn sich die Spacejet (ohne die üblichen SciFi “Echtzeit”-entzerrerfelder/-kompenastoren etc.) mit annährend c bewegt, die sich mir da stellt ist folgende: Würde sich der Mond nicht je “eingefrorener in der Zeit” er sich zeigt immer mehr in einer Sicht auf seine Superposition darstellen?

    Und der Disclaimer: schön als Laie auch unfundierte Beziehungsgeflechte zu “hypothetisieren” und zur Diskussion stellen zu können.

  75. #75 JaCobi
    21. Oktober 2012

    @MartinB
    Danke für’s Feedback. Ich denke, daß alles so weit verständlich wird, daß nicht mehr viel unklar bleibt mit diesem Text hier: https://forum.daserste.de/showthread.php?t=1266362
    Das “Höhere” hatte in grauer Vorzeit mal Erklärungsfunktion. Unsere Vorfahren erklärten sich das, worauf sie keinen Einfluß hatten mit höheren Mächten. Z. B. warum es donnert, wenn es geblitzt hat. Ich möchte bewußt machen, daß sich unser Unwissen in Bezug auf das, was sich tatsächlich ereignet, mit den Fortschritten wissenschaftlicher Erkenntnismethoden immer mehr auf physikalische Vorgänge reduzierte, und wir mit dem Programm der Evolution im Kopf auch das meiste unseres Zusammenlebens so erfolgreich steuern können, wie uns das die Natur mit atemberaubenden Entwicklungen vorgemacht hat.
    In unserem “Wahrnehmungsräumen” kann nicht das Geringste verschwinden. Alles hängt mit allem anderen mehr oder weniger intensiv zusammen. Deshalb spreche ich gerne von der “Kohärenzfalle” Welt.
    Ich mache seit vielen Jahren erstaunliche Erfahrungen bei meinen Aufklärungsversuchen und werde meist durch Denkfehler gebremst. Die kann ich zwar rasch und widerspruchslos widerlegen, doch erwürge ich damit meist das Interesse an dem, was mir u. a. ermöglichte, das weltweit erste Verfahren zum Berechnen humanitärer Schäden auf die Beine zu stellen, das Adäquanz-Kommensurabilitätsverfahren. Wissen ist Macht, auch wenn man es nur aufbläst.
    Mit meinen “Schrumpfwissen” brachte mir Rolf Dobellis “Die Kunst des klaren Denkens” nur einen Erkenntnisgewinn: Der schweizer Autor, Gründer von zurich.minds, einer Community führender Persönlichkeiten aus Wissenschaft, Kultur und Wirtschaft kennt zwar den Vater aller Denkfehler, nicht aber die Mutter. Und erst wenn man auch die kennt, kann man verhindern, daß die beiden es sich zu gut gehen lassen in unseren Denkwürsten mit entsprechenden Folgen auf unser Denken und Handeln. Hier gibts mehr darüber: https://www.interessen-management.de/rolf-dobelli.html Ich verlinke diese Seite, weil ihr Entstehen auch nur möglich wurde durch Nachvollziehbarkeit des “BinärCode-Blubbs”.

    Ich bin wirklich für jede Kritik dankbar! Dein Eindruck über meine Schreibe ist nicht ganz falsch, läßt sich aber mir dem erklären, was ich zuvor geschrieben habe. Wie schlau banal Einfaches sein kann, hält niemand für möglich. Hinter all das zu kommen, was inzwischen sogar potente Leute im Ausland auf mich aufmerksam gemacht hat, die mich für internationales Litigationsmanagement gewinnen möchten, war einfacher, als es Zeitgenossen verständlich zu machen, die sich intensiv und unbewußt damit beschäftigen, fundamentale Erkenntnisse über die Strukturen unseres Miteinanders, Nebeneinanders, Aneinandervorbeis und Gegeneinanders zu verdrängen. Insofern bitte ich um Nachsicht und Verständnis. Danke!

  76. #76 Christian
    21. Oktober 2012

    @Niels
    “Mir ging es ja ebenfalls nicht um die Interpretationen, sondern darum, dass die QM eben doch lokal und realistisch sein kann, nämlich
    3) lokal, realistisch und nicht “kontrafaktisch definit (?)” oder”

    Unter “Counterfactual definiteness” (CFD) ist aber, wenn ichs richtig verstanden habe, eben das gemeint, was man im allgemeinen als “realistisch” bezeichnet. EPR mein ja:

    “Eine physikalische Größe, deren Wert mit Sicherheit vorhersagbar ist, ohne das System, an dem sie gemessen wird, zu stören, ist ein Element der physikalischen Realität.”

    EPR geht also von CFD aus, Bell ebenfalls (plus Lokalität) und entwickelt auf der Grundlage dieser Annahmen seine Ungleichung. Da die Ungleichung experimentell verletzt wird, muss mindestens eines der beiden Annahmen falsch sein … so die Schlussfolgerung.

    “4) lokal, realistisch und “superdeterministisch”.
    (Wobei ich 4 ziemlich fishy finde.)”

    Bell fand es auch komisch. Aber es wäre der einzige Weg, sowohl Lokalität als auch CFD zu wahren.

    “Many worlds habe ich nur als sehr bekanntes Beispiel für 3) erwähnt.”

    MW würde ich nicht 3) zuordnen. Wenn ich eine Messung durchführe teilt sich das gesamte Universum, ist das ein lokales Ereignis? Und “wann” findet diese Teilung statt, wenn es doch gar keine absolute Gleichzeitigkeit gibt?

    Eine Lösung wäre eine Variante von MW, die die Existenz dieser Paralleluniversen schon vor der Messung konstatiert, d.h. es findet gar keine Teilung statt, vielmehr ein Auseinanderdriften bereits bestehender Paralleluniversen. Aber auch hier stellt sich die Frage nach der Lokalität.

  77. #77 Niels
    21. Oktober 2012

    @Christian

    Unter “Counterfactual definiteness” (CFD) ist aber, wenn ichs richtig verstanden habe, eben das gemeint, was man im allgemeinen als “realistisch” bezeichnet. EPR mein ja:
    “Eine physikalische Größe, deren Wert mit Sicherheit vorhersagbar ist, ohne das System, an dem sie gemessen wird, zu stören, ist ein Element der physikalischen Realität.”

    Dieses Zitat ist aber doch nicht die Definition von CFD?
    Ich sehe den Zusammenhang gerade nicht?

    MW würde ich nicht 3) zuordnen.

    Darüber kann man sich durchaus streiten.
    Ich habe hier aber eigentlich nur die (aktuelle) Mehrheitsmeinung wiedergegeben. Nach kurzem googeln hab ich das sogar zweimal in der englischen Wiki gefunden.
    Unter “Principle of locality”:
    In the many-worlds interpretation both realism and locality are retained, but counterfactual definiteness is rejected by the extension of the notion of reality to allow the existence of parallel universes.
    Unter “Many-worlds interpretation”:
    MWI is a realist, deterministic, local theory, akin to classical physics (including the theory of relativity), at the expense of losing counterfactual definiteness. MWI achieves this by removing wavefunction collapse, which is indeterministic and non-local, from the deterministic and local equations of quantum theory.

    Wie EPR und many worlds zusammenpassen findet man hier:
    https://www.hedweb.com/manworld.htm#epr

    Eine kurze Zusammenfassung der selben Seite:
    https://www.hedweb.com/manworld.htm#local

  78. #78 Niels
    21. Oktober 2012

    @MartinB
    Hängt mein vorheriger Kommentar im Spam-Filter?

    @Christian
    Zum Teil hatte ich dir schon geantwortet, siehe oben.
    Weiter gehts:

    Allgemein zur Lokalität in der many worlds:

    David Albert und Barry Loewer: Interpreting the many worlds interpretation
    https://www.springerlink.com/content/w720n26k6145g0n2/fulltext.pdf
    Not only is the MMV a realist interpretation of quantum mechanics, it also provides an account in which all interactions are local. This is surprising because Bell’s theorem is widely thought to rule out realist, local interpretation of quantum mechanics. To see how the MMV allows for a local realist interpretation without violating Bell’s theorem, […]

    Frank J. Tipler: Does Quantum Nonlocality Exist? Bell’s Theorem and the Many-Worlds Interpretation
    https://arxiv.org/abs/quant-ph/0003146

    Mark A. Rubin: Locality in the Everett Interpretation of Heisenberg-Picture Quantum Mechanics
    https://arxiv.org/abs/quant-ph/0103079

    Bei Rubin findet man auch noch ne Menge weiterer References genau dazu.

    Eine Lösung wäre eine Variante von MW, die die Existenz dieser Paralleluniversen schon vor der Messung konstatiert

    Ich sehe nicht, wie die Interpretation noch Sinn ergeben könnte, wenn es die Paralleluniversen schon vor den Messungen gibt.
    Dadurch vervielfacht sich doch nur das Problem, das man mit many worlds zu lösen versucht?

  79. #79 Fossilium
    22. Oktober 2012

    Hi Martin,
    sorry, in c) war statt Ableitung das Integral gemeint. Das kommt davon, wenn man zu schnell denkt und sich keine Zeit lässt. In der nichtrelativ. Mechanik ist die Energie E die Ableitung der Wirkung nach der Zeit : E = dW/dt und der Impuls ist die Ableitung der Wirkung nach dem Ort : p = dW/dx (hier nur eine Richtg. angegeben). Relativistisch betrachtet ist der Betrag des Vierer-Energie-Impuls-Vektors invariant beim Wechsel zwischen Bezugssystemen, weil abei die Änderung der Energie -dE gerade gleich der Änderung des Impulses dp ist.. Das totale Differential dW = pdx – Edt ist lorenzinvariant – die einzige dynamische Grösse, die unabhängig ist von Zeit und Raum. Folgerung: Was wirkt, wirkt aus einer Seinsebene ausserhalb von Raum und Zeit in die Raumzeit hinein, und verändert dort Energie und Impuls. Hört sich esoterisch an, ist es aber nicht.
    Normalerweise beschreibt man das nicht so, sondern man definiert ein Kraftfeld und tut so, als sei dieses in der Raumzeit vorhanden. Das ist aber noch esoterischer, weil da ein gedankliches Konstrukt (Feld) in die Raumzeit regelrecht hineingedacht wird, das auch noch dort eine reale Wirkung haben soll – bei genauem Hinsehen muss das unter Realitätsaspekten als Blödsinn bezeichnet werden. Die Sicht, dass hier ein nicht beobachtbares Wirkungsfeld ausserhalb von Zeit und Raum in die Realität hineinwirkt, hört sich metaphysisch an, ist im Endeffekt aber plausibler, als das Feldkonzept (Bitte nicht vergessen: wir reden hier über das Realitätskonzept, nicht über praktische Anwendungen einer Theorie)
    Was soll denn das heissen, wir haben die Raumzeit alle gemeinsam ? Wir haben keine gemeinsame Raumzeit. Ich habe meinen eigene Zeit (sie verläuft für mich anders als für Dich) und meinen eigenen Raum (ich habe meine eigenen Längenmasstäbe, die nicht mit Deinen übereinstimmen). Ich messe auch andere Werte an gemeinsamen Objekten. Es ist daher nicht derselbe Raum. Was soll da bezgl. Raumzeit gemeinsam sein. Das einzige, was gemeinsam ist, ist die Tatsache, dass eine Wirkung in meiner Welt sich von der Wirkung in Deiner nicht unterscheidet.
    Wirkung hat etwas Absolutes an sich – eine physikalische Grösse, die ins Metaphysische weist.
    Sowas fürchten alle Physike wie der Teufel das Weihwasser. Aber meine Argumentation ist nicht unschlüssig.
    Grüsse Fossilium

  80. #80 Fossilium
    22. Oktober 2012

    Hi Martin,
    nochmal zur Raumzeit: meine Raumzeit hat eine andere Zeit und eine andere Geometrie, ich bezeichne sie aus diesen Gründen als eine andere. Unterstellt man, dass die Raumzeit aus einem irgendwie gearteten Wirkungsgefüge aufespannt wird, was immer man darunter verstehen mag, dann haben wir die gleiche. Aber was Raumzeit ist, ist ja auch noch nicht definert.
    Grüsse Fossilium

  81. #81 Fossilium
    22. Oktober 2012

    Hi Volker,
    der Empfehlung von Martin, das Buch Escher Gödel Bach zu lesen, stimme ich voll zu – das ist wirklich super.
    Feynman kannst Du auslassen, der hat sich für Fragen, wie die, die hier besprochen wurden, nie wirklich interessiert.
    Grüsse Fossilium

  82. #82 MartinB
    22. Oktober 2012

    @Fossilium
    “Was wirkt, wirkt aus einer Seinsebene ausserhalb von Raum und Zeit in die Raumzeit hinein, ”
    Huh? Nein, das ist falsch. Ich sag’s nochmal (es gibt ja auch das berühmte Minkowski-Zitat dazu): Die Raumzeit ist selbst (insbesondere leicht zu sehen, wenn man sich ein Blockuniversum denkt) ein invariantes Gebilde. Unterschiedlich bewegte Beobachter zerteilen sie anders in Raum und Zeit – aber genauso wenig, wie wir sagen, dass der Raum nicht existiert, nur weil die Richtung, die für mich oben ist, für jemanden in Australien unten ist, genauso wenig können wir sagen, dass die Raumzeit ein problematisches Konzept ist, nur weil wir sie unterschiedlich ansehen.
    In der klassischen Elektrodynamik z.B. haben wir elektromagnetische Felder. Unterschiedliche Beobachter sehen unterschiedliche Anteile von E- oder B-Feld. Trotzdem sind sich alle über ihre Beobachtungen einig und wir können alles konsistent beschreiben.

    “meine Raumzeit hat eine andere Zeit und eine andere Geometrie, ich bezeichne sie aus diesen Gründen als eine andere.”
    Dann hast du auch einen anderen Raum als ich, wenn du in Australien bist, nur weil deine Koordinatenachsen anders liegen? Es ist wirklich kein fundamentaler Unterschied zwischen einer Rotation von Achsen im Raum und einer in der Raumzeit.

    “Feynman kannst Du auslassen, der hat sich für Fragen, wie die, die hier besprochen wurden, nie wirklich interessiert.”
    Nope, der tut nur so.

  83. #83 volker
    22. Oktober 2012

    @fossilium, @martin

    Oui, aua, da habe ich aber einen Titanenkampf zwischen Euch losgetreten, was gar nicht meine Absicht war, aber dennoch spannend, wenn was dabei rauskommt, und alles über der wissenschaftlichen Gürtellinie bleibt. War mir schon klar, dass die Vorstellung einer Realistischen Seins-(Wirk-) ebene außerhalb der beobachtbaren physikalischen Raumzeit ein dicker Brocken für die Gurgel von Physikern sein würde. Aber wir haben ja noch viel Zeit, “je länger desto forscht es”, vivat sequentes! Ich verabschiede mich erst einmal, bis ich wieder aktiv denken kann. PS Gödel Escher Bach habe ich natürlich schon gelesen, vielleicht refreshment gut. Von Feynman kenne ich nur seine QM-Diagramme. Danke Euch, Volker

  84. #84 Niels
    22. Oktober 2012

    @volker
    Na ja, du musst aber bedenken:
    Das eine ist der momentane Wissenstand der Physik, das andere ist eine Privat-“Theorie” vermutlich einer einzigen Person, wobei es zu dieser Idee natürlich auch keine einzige Veröffentlichung gibt. (Von Peer-Reviews mal ganz abgesehen.)
    Da fällt mir die Entscheidung ziemlich leicht, auch ohne tiefere Prüfung.

    Die Leseempfehlung für Feynman unterschreibe ich hiermit. 😉

    @Fossilium
    War wirklich nicht böse oder persönlich gemeint. Ich habe aus dem bisher Gesagten noch nicht einmal verstanden, wie dein “Realitätskonzept” überhaupt genau aussieht.
    Was eine Raumzeit ist, ist aber entgegen deiner Behauptung natürlich doch absolut eindeutig und sogar streng mathematisch definiert.
    Hast du eigentlich irgend eine Quelle für diese Wirkungs-Seinsebenen-Sache, oder ist das komplett deine eigene Idee?

  85. #85 Fossilium
    23. Oktober 2012

    Hi Martin, Hi Niels,
    ich nehm es doch keinem übel, wenn er nicht meiner Meinung ist, oder mich nicht versteht, das liegt ja dann an mir, dass ich mich nicht richtig ausdrücke. Es gibt aber auch Denkgewohnheiten, die einen daran hindern, sich gegenseitig zu verstehen (betrifft mich auch). Dazu gehört, dass sich jeder unter Realität, Kausalität, Raum, Zeit und Kraftwirkung etwas anders vorstellt – oder sich evtl. garnichts vorstellt, bzw. Vorstellungen darüber ungesagt einfach seinen Theorien voraussetzt. Das ist ein etwas anderes Thema als das was Martin unsprünglich angesprochen hat, und wir kamen ja nur darauf, weil man automatisch mit Fragen zu diesen Voraussetzungen konfrontiert wird, wenn man die Welt der Modelle verlässt und behauptet, das was in den Modellen existiert (Quanten, Felder, Teilchen, Kräfte, usw.) auch in der Realität existieren würde. So behauptest Du, Marin, “In der klassischen Elektrodynamik z.B. haben wir elektromagnetische Felder. ” Ja klar, in der Theorie, aber in der Realität sind diese nicht vorhanden. Wenn einer fragt, wieso können wir mit dem Handy über weite Entfernungen miteinander sprechen – dann antwortest Du doch sofort: weil da vom Handy ein elektromagnetisches Feld abgestrahlt wird, und vom Sprechpartner empfangen wird, usw. In Wirklichkeit (ich meine jetzt in der Realität) gibt es dieses elektromagnetische Feld aber garnicht, und wir wissen überhaupt nicht, wie die Kommunikation abläuft. Dieses Beispiel zeigt vielleicht am besten auf, welche Probleme die Realität dem Physiker bereitet.

    Ich hatte das totale Differential der Wirkung, nämlich dW = pdx – Edt, angeführt als die einzige dynamische Lorenzinvariante, alles andere, Energie, Impuls, Kraft, usw. sind nicht erhalten beim Wechsel zwischen Bezugssystemen. Egal wie man zum Begriff der Wirkung steht, das sollte einem zu denken geben.

    Das mit der Raumzeit sehe ich wirklich anders – es ist aber möglicherweise eine Frage der Interpretation. Ich behaupte nicht, dass sich beim Wechswel zwisschen Bezusgssystemen die Zeiten und Längen gegenüber irgendwelchen absoluten Masstäben verändern würden, sondern nur dass in meinem Raum meine Abstände, Zeiten und Messergebenisse andere sind, als in Eurem, wenn ich mich Euch gegenüber gleichförm. bewege, und dass ich deshalb von einer eigenen Raumzeit spreche. Die einzigen Masstäbe, die ich habe (ausser der Wirkung), sind andere. Ich denke, dass diese Unterschiede für den Realitätsbegriff wichtig sind – und die Frage der Objektivität hat Martin ja auch sehr anschaulich angeschnitten (s. oben).
    Vielleicht führt dies zur Klarheit, wenn auch nicht zur Wahrheit.
    Grüsse Fossilium

  86. #86 Fossilium
    23. Oktober 2012

    Hi Martin,
    Du schreibst:
    “Die Raumzeit ist selbst (insbesondere leicht zu sehen, wenn man sich ein Blockuniversum denkt) ein invariantes Gebilde. Unterschiedlich bewegte Beobachter zerteilen sie anders in Raum und Zeit – aber genauso wenig, wie wir sagen, dass der Raum nicht existiert, nur weil die Richtung, die für mich oben ist, für jemanden in Australien unten ist, genauso wenig können wir sagen, dass die Raumzeit ein problematisches Konzept ist, nur weil wir sie unterschiedlich ansehen.”
    Das ist alles o.k.
    Aber Du stellst hier die von der Theorie beschriebene und in Raum und Zeit hineingelegte Geometrie in den Vordergrund.
    Ich stelle mir dagegen vor, welche Realität hat mein Raum. Reailität manifestiert sich für mich durch Beobachtungen, d.h. ich messe Energie und Impuls, die Ergebnisse beschreiben meine Realität – nicht die Geometrie. Und diese Ergebnisse sind anders, daher besteht meine Realität aus einem anderen Raum und einer anderen Zeit.
    Ist es nicht schrecklich schwierig über diese Dinge zu sprechen ? Man muss ständig die Voraussetzungen hinterfragen und sich ständig abstimmen.
    Grüsse Fossilium

  87. #87 MartinB
    23. Oktober 2012

    @Fossilium
    ” In Wirklichkeit (ich meine jetzt in der Realität) gibt es dieses elektromagnetische Feld aber garnicht,”
    Warum nicht? (Wenn wir mal in der klassischen Physik bleiben.) Wenn ich das ablehne, dann muss ich konsequenterweise auch ablehnen, dass es überhaupt irgendetwas gibt bzw. dass irgendetwas so ist, wie es mir erscheint, selbst die Computertastatur vor mir oder mein eigener Körper. Auch die sind doch genau so viel oder wenig Konstrukte meines Gehirns wie em-Felder. Den Standpunkt kann man natürlich einnehmen, aber dann ergibt es wenig Sinn, überhaupt über “Realität” reden zu wollen, wenn man von vornherein davon ausgeht, dass diese prinzipiell nicht erkennbar ist.

    “Egal wie man zum Begriff der Wirkung steht, das sollte einem zu denken geben.”
    Nein, warum. Die Tatsache, dass zwei Leute auf einen Würfel gucken und der eine eine quadratische Fläche sieht und der andere eine sechseckige (weil er entlang der Diagonale guckt), akzeptieren wir ja auch.

    “dass ich deshalb von einer eigenen Raumzeit spreche.”
    Das ist einfach falsch und bleibt auch falsch – du kannst von deinem eigenen Raum sprechen und von deiner eigenen Zeit, das ist o.k. Aber die Raumzeit ist für alle gleich, das ist ja gerade der Witz der SRT (und das habe ich ja auch oben mit dem Kompass-beispiel zu verdeutlichen versucht). Wenn du das nicht einsiehst, dann hast du etwas ganz Fundamentales nicht verstanden, tut mir Leid.

    “Und diese Ergebnisse sind anders, daher besteht meine Realität aus einem anderen Raum und einer anderen Zeit.”
    Nein. Man sollte – wie Einstein – von Ereignissen sprechen, um die Raumzeit zu analysieren. Die Ereignisse “mein Finger tippt das ‘E'” und “mein Finger tippt das ‘r”” sind wohldefinierte Ereignisse in der Raumzeit. Wir sind uns vielleicht nicht einig darüber, wie genau der räumliche und der zeitliche Abstand zwischen beiden Ereignissen ist (genauso wie wir uns vielleicht nicht einig sind, wo rechts und links ist, weil du in eine andere Richtung guckst), aber über den raumzeitlichen Abstand sind wir uns einig. Die Raumzeit konstruiert man aus gemessenen Ereignissen, nicht aus Energien oder Impulsen.

    “Ist es nicht schrecklich schwierig über diese Dinge zu sprechen ?”
    Um ehrlich zu sein, eigentlich nicht – schwierig ist nur, dass du in die Raumzeit ein Problem hineingeheimnissen willst, das nicht da ist (es sei denn du hast auch schon ein Problem mit Rotationen im Raum, auch da ist “mein Impuls” nicht “dein impuls”, weil ich unterschiedliche Zahlen in unterschiedlichen Koordinatensystemen brauche, das erscheint dir nur trivial, weil aus dem Alltag vertraut).

  88. #88 Niels
    23. Oktober 2012

    @Fossilium

    ich nehm es doch keinem übel, wenn er nicht meiner Meinung ist, oder mich nicht versteht, das liegt ja dann an mir, dass ich mich nicht richtig ausdrücke. Es gibt aber auch Denkgewohnheiten, die einen daran hindern, sich gegenseitig zu verstehen (betrifft mich auch).

    Sorry, aber es liegt vor allem daran, dass du ziemlich seltsames Zeug erzählst und dass du außerdem physikalische Begriffe verwendest, aber ihre Bedeutung anscheinend nicht kennst.

    So scheint du zum Beispiel nicht zu wissen, was der physikalische Begriff der Wirkung bedeutet. Die physikalische Wirkung bezeichnet etwas völlig anderes als der umgangssprachliche Ausdruck, der für das Ergebnis einer Ursache oder für eine “Einwirkung” steht.

    alle Wirkungen breiten sich mit LG aus

    Die physikalische Wirkung hat keine Geschwindigkeit und man kann ihr auch keine zuschreiben. Diese Aussage ist sinnlos.

    Denn Wirkung ist verantwortlich für den stetigen Wandel in der Realität, aber keine physikalische Theorie erfasst das Bezugssystem der Wirkung

    Auch das ist Unsinn, wenn es um die Physik geht. Es kann kein Bezugssystem “der Wirkung” geben, zwischen diesen Begriffen gibt es so keinen Zusammenhang.
    Beim “Wandel in der Realität” wird sogar ganz deutlich, dass du eigentlich über den umgangssprachlichen Wirkungsbegriff sprichst.

    Was wirkt, wirkt aus einer Seinsebene ausserhalb von Raum und Zeit in die Raumzeit hinein, und verändert dort Energie und Impuls. Hört sich esoterisch an, ist es aber nicht.

    Doch, das ist hochgradig esoterisch. Außerdem kommt hier schon wieder der Fehlschluss zwischen den Wirkungsbegriffen vor.
    Die physikalische Wirkung “wirkt” nicht. Sie verändert auch nicht Energie und Impuls. Was das mit der Seinsebene soll ist mir völlig unklar. Die Wirkung ist konzeptuell in der selben Liga angesiedelt wie beispielsweise die Begriffe Entropie und Energie. Leben solche Dinge dann auch in einer Seinsebene außerhalb der Realität und was bedeutet das?

    Die Sicht, dass hier ein nicht beobachtbares Wirkungsfeld ausserhalb von Zeit und Raum in die Realität hineinwirkt

    Ein “Wirkungsfeld” ist nach den physikalischen Definitionen von Feld und von Wirkung Quark (das Milchprodukt, nicht die Teilchen).
    Aber ein physikalischen Feld ist sowieso etwas, dass jedem Raumzeitpunkt eine physikalische Größe (Skalar, Vektor, Tensor, …) zuweist. Deswegen kann sowieso niemand wissen, was ein Feld “ausserhalb von Zeit und Raum” sein soll.

    Normalerweise beschreibt man das nicht so, sondern man definiert ein Kraftfeld und tut so, als sei dieses in der Raumzeit vorhanden. Das ist aber noch esoterischer, weil da ein gedankliches Konstrukt (Feld) in die Raumzeit regelrecht hineingedacht wird, das auch noch dort eine reale Wirkung haben soll

    Ich weiß nicht, warum es esoterisch(er) sein soll, wenn sich etwas auch tatsächlich in der Raumzeit befindet. Wo denn sonst? Vor allem, wenn Feld auch nur gneau so definiert ist.
    Warum sollte es seltsamer sein, dass etwas in der Raumzeit auch andere Dinge in der Raumzeit beeinflusst, als das etwas außerhalb der Raumzeit Dinge beeinflusst?
    (Da bekommst du übrigens mal wieder Begriffs-Wirrwarr, wenn du da statt beeinflussen “reale Wirkung” schreibst.)

    In der nichtrelativ. Mechanik ist die Energie E die Ableitung der Wirkung nach der Zeit

    Das ist falsch.

    der Impuls ist die Ableitung der Wirkung nach dem Ort

    Das auch.

    Das totale Differential dW = pdx – Edt

    Das soll so sein? Wo kommt diese Gleichung her?
    Die physikalische Wirkung ist definiert als das Integral der Lagrange-Funktion über die Zeit.
    Würde mich sehr wundern, wenn deine Gleichung im Allgemeinen richtig wäre.

    Das totale Differential ist lorenzinvariant

    @MartinB:
    Ist das wirklich so? Die Lagrange-Dichte kann man lokal definieren und sie ist invariant, deswegen benutzt man sie ja so gerne in den relativistischen Theorien.
    Aber das totale Differential der Wirkung?

    Das totale Differential […] ist lorenzinvariant – die einzige dynamische Grösse, die unabhängig ist von Zeit und Raum.

    Es gibts natürlich vielmehr unzählig viele weitere lorentzinvariante Größen. Du hast doch zum Beispiel selbst das Skalarprodukt des Vierer-Impulses genannt? Dann gibts auch noch den Energie-Impuls-Tensor, …
    Da kommen doch eindeutig ebenfalls “dynamische Grössen” vor. Was ist mit “dynamische Grösse” eigentlich genau gemeint und warum sollten solche Größen in irgendeiner Weise besonders ausgezeichnet gegenüber anderen Größen sein?

    “Unabhängig von Raum und Zeit” ist darüber hinaus eigentlich auch eher kein Synonym für lorentzinvariant.

    Relativistisch betrachtet ist der Betrag des Vierer-Energie-Impuls-Vektors invariant beim Wechsel zwischen Bezugssystemen, weil abei die Änderung der Energie -dE gerade gleich der Änderung des Impulses dp ist.

    Alle Vierervektoren haben invariante Skalarprodukte.
    Dein “weil” ist nur eine Folgerung daraus, dass sich beim Viererimpuls E^2 + p^2 = m^2 ergibt.

    Ist es nicht schrecklich schwierig über diese Dinge zu sprechen ? Man muss ständig die Voraussetzungen hinterfragen und sich ständig abstimmen.

    Na ja, wie schon angemerkt:
    Deswegen muss man sich entweder absolut peinlich genau an die bekannten Definitionen halten. Oder aber man man muss selbst jedesmal ausdrücklich anmerken, dass man einen Begriff jetzt anders als üblich verwendet und wie er diesmal genau zu verstehen hat.

    “In der klassischen Elektrodynamik z.B. haben wir elektromagnetische Felder. ” Ja klar, in der Theorie, aber in der Realität sind diese nicht vorhanden

    Da waren wir vor einer Weile ja schon einmal, dort hattest du aber leider nicht reagiert:
    Woher weißt du das?
    Und gibt es noch andere Menschen, die die Wirkung für “real” halten, elektrische Felder dagegen nicht? Oder ist das komplett deine eigene Idee?

    Man kommt hier an die Grenze zur Metaphysik, und dann hören wir Physikar dann auf, und denken im Geheimen als Metaphysiker weiter.

    Ist jetzt wieder nicht böse gemeint, aber bist du tatsächlich ausgebildeter Physiker oder war das jetzt mehr so ein allgemein gemeintes wir?

  89. #89 Niels
    23. Oktober 2012

    @Fossilium
    Das wollte ich oben noch schreiben, hab es dann aber vergessen.
    MartinB ist zwar schon sehr ausführlich darauf eingegangen, aber noch einmal zur Bekräftigung:

    Das mit der Raumzeit sehe ich wirklich anders – es ist aber möglicherweise eine Frage der Interpretation.

    Das kann man allerdings einfach nicht sinnvoll anders sehen, da gibt es keinen Interpretationsspielraum,.
    Wie schon geschrieben, der Begriff der physikalischen Raumzeit ist eine streng definierte Fachvokabel.
    Deswegen muss man die Geometrie in den Vordergrund stellen, dass verlangt die Definition.
    Da gibt es keinen anderen Zugang.

    Wenn du das anders machen willst, redest du einfach nicht mehr über die Raumzeit der Physik.
    Dann müsstest du dir aber eigentlich auch eigene Begriffe ausdenken, diese sinnvoll definieren und das Ganze dann in ein widerspruchsfreies, weiterführendes Konzept bringen.

    Ich stelle mir dagegen vor, welche Realität hat mein Raum. […] ich messe Energie und Impuls, die Ergebnisse beschreiben meine Realität – nicht die Geometrie. Und diese Ergebnisse sind anders, daher besteht meine Realität aus einem anderen Raum und einer anderen Zeit.

    Dann fängt du am besten mal damit an, die Begriffe Raum und Zeit zu definieren. Wenn Geometrie dabei zweitrangig sein soll, hat das mit den üblichen Bedeutungen nämlich extrem wenig zu tun.

  90. #90 Niels
    23. Oktober 2012

    Hm, Klammer nicht zugemacht.
    War so gedacht:

    Ich stelle mir dagegen vor, welche Realität hat mein Raum. […] ich messe Energie und Impuls, die Ergebnisse beschreiben meine Realität – nicht die Geometrie. Und diese Ergebnisse sind anders, daher besteht meine Realität aus einem anderen Raum und einer anderen Zeit.

    Dann fängt du am besten mal damit an, die Begriffe Raum und Zeit zu definieren. Wenn Geometrie dabei zweitrangig sein soll, hat das mit den üblichen Bedeutungen nämlich extrem wenig zu tun.

  91. #91 JaCobi
    23. Oktober 2012

    Lieber Martin Bäker,

    bitte lasse dir Zeit mit einer Antwort. Ich bin schon froh, daß Du Bereitschaft gezeigt hast, dich mit meiner “Schlu§folgerung” zu beschäftigen, der ich mich quasi von vorne genähert habe über sehr viele unterschiedliche Fallstudien mit Menschen, die um ihr gutes Recht kämpfen mußten nach schuldlos erlittenen Schicksalsschlägen. Was mich dabei immer wieder erstaunte war, daß Ursache und Lösung sich immer mehr aufeinander zubewegten. So konnte ich ganz zum Schluß meiner Entdeckungsreise renommierten Konfliktexperten die Frage stellen, ob sie sich vorstellen können, daß ein einziges, banales Verständigungshandicap für sämtliche zwischenmenschlichen Probleme und Konflikte zuständig ist. Daraus entwickeln wir dann Krisen, weil niemand die Ursache kennt. Bis auf einen Experten, der das für denkbar hielt, haben alle anderen entschieden abgelehnt mit dem Hinweis, daß die Vielfalt unserer sozialen Störungen einfach zu groß sei. Ich brauchte dann keine 10 Minuten, um sie davon zu überzeugen, daß tatsächlich nur ein winziger Störenfried verANTWORTlich ist.

    Und mit dieser Erkenntnis bewegte ich mich dann auch immer weiter auf physikalische Vorgänge zu und der Quantenphänomenologie. Bis ich irgendwann sagen konnte “ALLE PHILOSOPHIE ENDET AM ANFANG”. – Dir geht es ja auch um mehr als um wissenschaftliche Beschäftigung.

    “Quantenmäßig” betrachtet sind wir nichts anderes als kleine aktive Systeme wie all das, was im Makrokosmos im Weltall herumfliegt. Allerdings mit dem feinen Unterschied, daß uns “Flügel” gewachsen sind. Damit können wir uns auf andere zubewegen oder uns von ihnen fernhalten. Die Entscheidung dazu muß im immer dichter werdenden Gewusel der Menschenwelt immer schneller fallen und zwingt uns, die für eine Partnerschaft notwendigen Informationen durch Einbildungen, auch Erfahrungen, zu ersetzen. Genau das ermöglicht uns einerseits viele Beziehungen, beendet sie aber auch schnell vorzeitig, wenn wir zu wenig beachten, daß andere, die in unsere Reichweite kommen, ebenfalls unter Wahrnehmungseinschränkungen leiden.

    Vernunftwesen haben es geschafft, sich zusätzlich auch noch einzubilden, sie könnten alles den Tatsachen entsprechend wahrnehmen und Partner wahrheitsgemäß informieren, damit der angestebte Energieaustausch auch für alle Beteiligten funktionieren kann. That’s all. Universe …! – Es funktioniert nur für die Menschenwelt mit unendlich vielen Problemen und Karambolagen, weil uns das nicht bewußt wird und wir alles tun, um es im Verdrängungsstatus zu lassen.

    Es wäre nett, Du würdest mir kurz per E-Mail mitteilen, wenn Du dich mit meinen Gedanken beschäftigt hast und dazu aus der Sicht eines Wissenschaftler was sagen willst, der sehr verständlich aufklären kann. Danke und schöne Grüße!

  92. #92 MartinB
    23. Oktober 2012

    @Jacobi
    Nach wie vor kann ich in deinen Kommentaren hier keinen Sinn erkennen, tut mir Leid.

  93. #93 JaCobi
    23. Oktober 2012

    @MartinB

    Vermutlich läßt sich deine Erkenntnisschwäche damit erklären, daß Du zwischen der Absicht, ein Ziel zu erreichen (z. B. Film drehen mit Woody Allen), die man auf vielfältige Weise unterstützen kann, und der Nichtabsicht, ein Kind zu haben mit einer tödlichen Krankheit, die in vielen Fällen unabwendbares Schicksal bleiben wird, so wenig Unterschied machst, daß Du dich über sowas aufregst. Quantenmechanik und Realität ist für mich was anderes. Deshalb kann ich auch mit etlichen Wikipediaartikeln nicht viel anfangen, weil dort Wahrheit und Wiklichkeit verwechselt wird. Nichts für ungut. Ich suche mir einen anderen Wissenschaftler. Danke dennoch für die Zuwendung!

  94. #94 Fossilium
    24. Oktober 2012

    Hi Niels,
    ich würde nicht so lauft herausrufen, dass meine Energie- und Impulsdefinitionen über den Bgeriff der Wirkung falsch sind, und dass Du das totale Differential dS nicht kennst – lies mal was nach über die Hamilton-Jakobische Gleichung nach (nicht unbedingt bei Wkipedia, dort aktuell grauenhaft erklärt, “Aus dem Artikel geht der Sinn des Formalismus überhaupt nicht hervor” – Zitat aus einem Diskussionsbeitrag zu Wiki – ist auch meine Meinung).

    Das Wort Wirkung wird umgangssprachlich verwendet, um die Wirkung einer Kraft, oder eine Wärme(ein)wirkung, oder im Zusammenhang von Ursache und Wirkung verwendet – und genau von dieser umagngssprachlichen Bedeutung gehe ich auch aus. Wovon sonst ?
    Wie messe ich Wirkung ? Gar nicht, sondern ich kann nur das Ergebnis einer Wirkung messen, z.B. eine Ortsveränderung, allgemeiner misst man immer Energie- und Impulsänderungen, das sind die grundlegenden Observablen in der Physik, und zwar für jede Art von Dynamik (ich verwende ldatu machmal das allgemeinere Wort “veränderungen”).
    Ich kann hier natürlich keine Vorlesungen über Grundkurs Physik abhalten, aber ich stehe genau auf diesen Grundlagen bei all meinen Ausführungen. Die Verhältnisse sind auch sehr einfach.

    Nun geht es hier aber nicht um Physik, sondern um das Verhältnis von Physik und Realität (s. Titel) Also muss man zunächst Realität definieren, was ich nicht gemacht habe, aber für mich ist Realität das was ich messen kann. Diese Sicht folgt aus der Methodik der Physik. Was kann ich messen ? Die allgemeinste nichttriviale Aussage dazu ist: Energie und Impuls.
    Die sind nun aber nicht lorenzinvariant, d.h. sie bleiben nicht erhalten beim Wechsel zwischen zwei Bezugssystemen.
    Das Einzige was erhalten bleibt, ist
    a) die Ruhemasse (die ist aber auch wieder mit der Witkung verknüpft – darüber will ich mich aber hier nicht auslassen)
    b) der Betrag der Vierervektoren (wenn man die Raumzeit mit Vierervektoren beschreibt), die Betrag des Vierer-Energie-Impuls-Vektors bleibt auch erhalten, daber das ist nur eine etwas andere Aussage von c)
    c) das was im Hamilton-Jakobischen Formalismus als Wirkungsfunktion beschrieben wird (das i s t die Wirkung)
    bleibt erhalten
    a) und b) lassen sich aus der Erhaltung der Wirkung (c) ableiten – b) besonders einfach, hab das alles schon mal oben beschrieben.
    Also einfacher kann ich es Dir nicht erklären – mit spinnerten Ideen hat das garnichts zu tun, sondern das sind Grundlagen der Mechanik und der RT, in jedem Handbuch nachzulesen, meist nur sehr kompliziert abgeleitet – wird Martin bestätigen.

    Ich erklärs aber auch noch mal ohne formal zu werden:
    Wenn Impuls- und Energieänderungen beobachtbare in der Realtät stattfindende, und mit den Methoden der Physik beschreibbare Ereignisse sind, dann müssen diese Ereignisse auch eine p h y s i k a l i s c h e Ursache haben, d.h. eine die sich in der Realität auswirkt und mit physikaischen Methoden beschreibbar ist, und nicht eine Ursache, die aus irgendeiner Zauberei herrührt, aus dem Nirwana kommt, aus meinem Gedankenapparat, oder einer in den Raum gedanklich hineingelegten Geometrie oder Feldfunktion. Unter diesem Aspekt gibt es eben ein Problem mit der Physik und der Realität, weil ein Kraftfeld, dass im üblichen physikalischen Sprachgebrauch diese Energie-und Impulsänderungen herbeiführt, und daher die Ursache darstellt, definitionsgemäß keine reelle, sondern eine in den Raum hineingedache Funktion von x und t ist. Nicht mehr und nicht weniger, Darüber lohnt sich nachzudenken.
    Denn der Tiel von Martin lautet: Quantenmechanik und Realität – also ein Realitätsproblem gibt es wirklich, nicht nur in der Quantenmechanik. Aber wenn man mal das Problem in der klassischen Physik herausgearbeitet hat, dann kann man das vielleicht auch in die QM übertragen und dort evtl Analogien finden und zu Lösungen kommen.
    Das ist doch das Ziel. ich will hier nicht Recht behalten, sondern ein Problem aufzeigen – wenn Du das noch nicht gesehen hast, dann haben das vielleicht viele andere auch noch nicht gesehen und dann wäre es umso wichtiger das in alle Breite mal darzustellen.
    Um nochmal (letztmalig) auf die RT zu sprechen zu kommen: in der RT legt man eine Minkwski-Geometrie gedanklich in den Raum, und jeder von uns sieht den Raum (ich rede jetzt nur vom Raum – nicht von Ereignissen)
    verzerrt, das heisst jeder sieht eine andere deformierte Geometrie. Dabei gibt keine undeformierte Geometrie, das ist die zentrale Aussage der RT. Es ist alos nicht so, dass wir nur nur Deformationen einer undeformierten Geometrie sehen, denn dann wäre ja die undeformierte Gepomietrie die eines absoluten Bezugssystems – nein, jeder sieht seine eigene, individuelle Deformation und niemand eine undeformierte – es gibt nur deformierte Geomietrien ! In diesem Sinne hat jeder seinen eigene Geometrie, wie gesagt kein deformietes Abbild einer absoluten, sondern s e i n individuelles Bild. Weil in diesem Sinn zu jedem Beobachter sein eigenes Bild der Geometrie gehört, muss man davon sprechen, dass er seinen eigenen Raum hat. Sonst fällt man ja wieder auf den Trugschluss herein, es gäbe doch ein absolutes Bezugssystem und wir würden nur Verzerrungen von dieser absoluten Geometrie wahrnehmen. Kapito ?
    Jetzt mach ich endgültig Schluss.
    Grüsse Fossilium

  95. #95 MartinB
    24. Oktober 2012

    @Fossilium
    “enau von dieser umagngssprachlichen Bedeutung gehe ich auch aus. ”
    Schade, das hat dann mit dem Wirkungsbegriff der Physik nichts zu tun, aber das haben wir schon mal bis zum Abwinken diskutiert. Nein, in der Physik ist eine Ortsveränderung nicht das Ergebnis einer Wirkung.

    Mal doof gefragt: Die Wirkung in der Physik ist doch das Integral über (T-V). Wieso soll das eigentlich nicht vom Bezugssystem abhängen? I Ruhesystem eines Teilchens ist T=0, in anderen Systemen nicht?

    “Was kann ich messen ? Die allgemeinste nichttriviale Aussage dazu ist: Energie und Impuls.”
    Wie “misst” du denn direkt eine Energie in der Mechanik?

    “das was im Hamilton-Jakobischen Formalismus als Wirkungsfunktion beschrieben wird (das i s t die Wirkung)”
    Nicht im alltäglichen Sinne.

    “es gibt nur deformierte Geomietrien”
    Denen liegt aber ein invariantes Gebilde zu Grunde, eben die Raumzeit. genauso wie es unteschiedliche Perspektiven eines Hauses gibt, aber das Haus dahinter ist immer gleich – da wir es auf eine 2D-Abbildung reduzieren, sieht es nur immer anders aus.

    “, dass er seinen eigenen Raum hat.”
    Ja, jeder hat seinen eigenen Raum und seine eigene zeit, so wie jeder, der das Haus anguckt, seine eigene Perspektive hat. Die Raumzeit ist aber allen gemeinsam.

  96. #96 Niels
    24. Oktober 2012

    @Fossilium

    Was kann ich messen ? Die allgemeinste nichttriviale Aussage dazu ist: Energie und Impuls.

    Sagt wer?
    Und wie misst du direkt Energien?

    ich würde nicht so lauft herausrufen, dass meine Energie- und Impulsdefinitionen über den Bgeriff der Wirkung falsch sind, und dass Du das totale Differential dS nicht kennst – lies mal was nach über die Hamilton-Jakobische Gleichung nach (nicht unbedingt bei Wkipedia, dort aktuell grauenhaft erklärt

    Dann schaun wir doch mal in die englische wiki oder du gibst ein Zitat aus einem Lehrbuch?
    (Dir ist schon klar, dass man bei diesem Formalismus generalisierte Koordinaten verwendet?)

    Das ist aber nur das Vorspiel, du brauchst außerdem noch ein Zitat, in dem steht, dass dieses totale Differential auch noch lorentzivariant ist.

    Wobei selbst bei Erfüllung dieser Punkte deine Idee leider immer noch ziemlicher Unsinn ist. Siehe die vielen anderen Punkte, die MartinB und ich angesprochen haben.
    Ob es diese Formel gibt und ob sie invariant ist, ist eigentlich nur ein unwichtiger Nebenaspekt.

    Unter diesem Aspekt gibt es eben ein Problem mit der Physik und der Realität, weil ein Kraftfeld, dass im üblichen physikalischen Sprachgebrauch diese Energie-und Impulsänderungen herbeiführt, und daher die Ursache darstellt, definitionsgemäß keine reelle, sondern eine in den Raum hineingedache Funktion von x und t ist.

    Jetzt behauptest du schon zum x-sten Mal, dass Felder nicht real (bei reell sehe ich kein Problem?) sein können. (Jetzt sind sie es sogar schon “definitionsgemäß” nicht.)
    Deswegen auch zum x-ten Mal die Frage:
    Woher weißt du das? (Bzw. nach welcher Definition sind sie es nicht?)
    Gibt es diesmal eine Antwort oder wieder nur eine Wiederholung der Behauptung?

    genau von dieser umagngssprachlichen Bedeutung gehe ich auch aus. Wovon sonst ?

    Wie schon oft gesagt: Wenn du Formeln für die physikalische Wirkung verwenden willst, musst du von der physikalischen Wirkung ausgehen. Nicht von irgend etwas anderem, dass damit nichts zu tun hat.
    Genau wie du von der physikalischen Raumzeit sprechen musst, wenn du Begriffe wie die Minkowski-Metrik in die Diskussion verwenden willst.
    Niemand weiß, was Minkowski-Metrik im Zusammenhang mit deinen “deformierten Geometrien” und individuellen Räumen bedeuten soll.

    Solange wir uns nicht einmal darüber einigen können, welchen Bedeutungsinhalt Begriffe in sich tragen, kann keine Kommunikation stattfinden.
    Wenn du darauf bestehst, umgangssprachliche statt physikalische Bedeutungen zu verwenden verwendest, können wir auch nicht über Physik sprechen.

  97. #97 Niels
    24. Oktober 2012

    @Fossilium
    Ah, ich glaube ich weiß jetzt, worum es dir geht.

    Du hältst das Prinzip der kleinsten/stationären Wirkung für besonders fundamental, weil man daraus alle anderen grundlegenden physikalischen Gleichung ableiten kann. Richtig?
    Dieses Prinzip ist in der klassischen Mechanik allerdings mathematisch äquivalent zu den Newtonschen Gesetzen, anders ausgedrückt ist es also einfach eine mathematische Umformulierung.
    Genau so ist es bei der Elektrodynamik und bei allen anderen fundamentalen Wechselwirkungen.
    Allerdings muss man dazu die Wirkung für jede dieser Wechselwirkungen passend wählen und dann tauchen zwangsweise auch Felder und Potentiale im Wirkungsfunktional auf. Nur mit “dynamische” Größen kommt man da nicht aus.

    Deswegen kann man auch nicht einfach Felder ablehnen aber an die Wirkung bzw. an das Prinzip der kleinsten Wirkung glauben.

    Ob dieses Prinzip eine tiefere Bedeutung hat, ist eine ganz andere, schwierigere Frage. Darüber kann man durchaus reden, dazu fühle ich mich aber nicht besonders kompetent.
    Eine überzeugende anschauliche Bedeutung oder gar einen tieferen Sinn hat meines Wissens jedenfalls noch niemand gefunden. Oder liege ich da falsch?

    Das hat aber doch absolut nichts mit der SRT, Lorentzinvarianz, der Raumzeit, “deformierten Geometrien”, totalen Differentialen oder den anderen von dir in die Diskussion gebrachten Begriffen zu tun?

  98. #98 MartinB
    24. Oktober 2012

    @Niels
    “Eine überzeugende anschauliche Bedeutung oder gar einen tieferen Sinn hat meines Wissens jedenfalls noch niemand gefunden. Oder liege ich da falsch?”
    Naja, in der QM bzw. QFT hat das ja schon eine Interpretation als Geschwindigkeit der Phase der Wahrscheinlichkeitsamplitude. Ich finde das irgendwie schon anschaulich (aber ich bin auch seltsam…).

  99. #99 rolak
    24. Oktober 2012

    Ich finde das irgendwie schon anschaulich (aber ich bin auch seltsam…).

    Da fällt mir noch etwas ein, das ich seit dem blog-Umzug ein wenig vermisse: “Kommentar der Woche”.

  100. #100 Niels
    24. Oktober 2012

    @MartinB

    Naja, in der QM bzw. QFT hat das ja schon eine Interpretation als Geschwindigkeit der Phase der Wahrscheinlichkeitsamplitude.

    Magst das das näher erläutern?
    Das Prinzip der kleinsten/stationären Wirkung kann man in der QFT als welche Aussage über die Geschwindigkeit der Phase der Wahrscheinlichkeitsamplitude interpretieren?
    Läuft das irgendwie auf Pfadintegral hinaus?
    (Irgendwie glaube ich aber eher nicht, dass etwas in dieser Art Fossilium zufriedenstellen wird. 😉 )

    Wenn man dieses Prinzip für besonders fundamental für die Realität hält, müsste man aber doch etwas finden, dass auch für die Relativitätstheorie funktioniert.
    (Na ja, man könnte sich wieder mit Verweis auf die noch zu findende Quantengravitation herausreden.)

  101. #101 Fossilium
    24. Oktober 2012

    Hallo Martin,

    Du schreibst:
    “.. den deformierten Geometrien…” liegt aber ein invariantes Gebilde zu Grunde, eben die Raumzeit … genauso wie es unteschiedliche Perspektiven eines Hauses gibt, aber das Haus dahinter ist immer gleich – da wir es auf eine 2D-Abbildung reduzieren, sieht es nur immer anders aus.”
    Ich hoffe ich verstehe Dich nicht falsch. Wenn das so wäre, wie Du schreibst, dann sieht das so aus, als ob die 2-D Abbildung des Hauses, oder die in die Raumzeit gelegte konstante Geomietrie, nach Wahl eines beliebigen Nullpunkts darin ja ein absolutes Bezugssystem darstellen könnte Der Kern der Relativistätstheorie ist ja, dass es genau diese e i n e für alle geltende Geometrie (d.h. gleiche Abstände für alle) nicht gibt – das ist doch auch klar, Physik ist das, was wir beobachten, und wir beobachten alle etwas anderes, es gibt nichts, was unabhängig von den Beobachtungsperspektiven immer gleich abläuft, gleich aussieht, genauer: was mit einer Geometrie beschrieben werden könnte, in der wir alle gleich lange Masstäbe verwenden.
    Ich wähle meinen Ursprung in mir und lege dann meine Geometrie da ein – und Du wählst Deinen Ursprung in Dir und legst Deine darein, und wir können uns auch auf einen Ursprung an einem externen Ort einigen, und wir sehen dann alles anders. Aber dieser Usprung ist willkürlich, bei einem anderen Ursprung würden wir wieder alles anders sehen, usw. Es gibt kein ausgezeichneten Ursprung und davon ausgehend auch keine ausgezeichnete Geometrie.
    Das gleiche gilt für die Zeit, alle Uhren gehen anders, es gibt keine, die den Takt vorgibt, wir können immer nur einen Taktunterschied definieren (z.B. nach einer Synchronisierung stellen wir den im Anschluss an eine Relativbewegung denTaktunterschied fest).
    Also sind Raum und Zeit immer und für jeden anders (d.h. die Beobachtungen sind andere) und Raum und Zeit sind keine Frage der Perspektive auf etwas das unverrückbar feststeht, z.B. eine das All umfassende Geometrie.

    Wir wursteln alle mit unseren eigenen Geometrien – oder in der gleichen Geometrie mit unterschiedlichen Abstandsmasstäben – herum.

    Deshalb sind die Grössen, die immer gleich bleiben bei jeder Messung oder unter jeder Perspektive so wichtig.

    Die Wirkungsfunktion im Hamilton Jakobischen Sinn ist so eine Grösse. Deshalb ist sie auch nicht beobachtbar. Wie soll denn das gehen ? Würden wir diese Grösse direkt beobachten können, dann hätten wir ja einen Blick auf etwas Unveränderliches, und könnten einen universellen Masstab darauf konstruieren, das widerspricht der RT.

    Zur Zentralbegriff der Wirkung:

    Man kann Wirkung einfach definieren als Kraft mal Weg mal Zeit, aber jede Definition muss ja zweckmässig sein. Niels fragte, was man denn messen, d.h. beobachten kann.

    Direkt beobachten kann ich nur Verschiebungen und Zeitdauern, somit auf direkte Weise eigentlich auch die Geschwindigkeit, und vielleicht eine Kraft (Gewicht), wenn man mal in der klassichen Mechanik bleiben will. Aber die Veränderungen eines Systems aus vielen Körpern damit zu beschreiben ist äussert mühselig – wieviele Listen müsste ich führen, um Verschiebungen, Zeiten und Kräfte für jedes Systemelement festzuhalten. Daher hat man geeignete andere Grössen definiert, die strukturbeschreibenden Charakter haben. Damit lassen sich die Veränderungen aller Elemente eines Systems in einer Zahl zusammenfassen. Auf solche strukturbeschreibenden Grössen kommt man, wenn man prüft, welche Kombinationen aus den genannten Grundgrössen Verschiebung, Zeit, Geschwindigkeit, Masse und Kraft denn bei einer Änderung des Systems erhalten bleiben. Wenn man das macht, dann kommt man dahinter, dass die Produkte Masse x Geschwindikeit (genannt: Impuls) und Kraft x Weg (genannt Energie) sich nicht ändern. Diese stellen somit sehr effektive Beschreibungsgrössen dar, und im Modell und bei Beobachtungen blickt man ganau auf diese Grössen. Sie sind das bevorzugte Ziel von Messungen, weil sie eine effektive Beschreibung auch zusammengsetzter Systeme gestatten. Das meine ich, wenn ich sage: wir messen in erster Linie immer Energie und Impuls. Es sind sehr effektive Strukturbeschreibungsgrössen.

    Dann stellt man fest, das die zeitliche Änderung des Impulses gleich der räumlichen Änderung der Energie ist, auch diese Grössen sozusagen noch zusammenhängen. Dieser Zusammenhang wird über den Kraftbegriff formalisiert: d( dS (x,t) /dx + dS/dy + dS/dx )/dt = F (Kraft) (d denke man sich als partielle Ableit.) nicht bilden – was nichts anderes bedeutet: Wirkung kann als Potential von Kraft aufgefasst werden. Daraus ergibt sich, das S (die Wirkung) sich aus der räumlichen Integration über den Impuls oder der zeitlichen Integration der Energie berechnen lässt in Einheiten Joule x sek. Alle Bewegungsgleichungen lassen sich aus diesen wenigen Überlegungen ableiten (egal welcher Formalismus).
    Das ist eine äussert allgemeine Beschreibung von Allem und Jedem und daher wird diese Integration in allen Feldern der Physik durchgeführt. Der Begriff Wirkung hat in seiner Formulierung als Potential der Kraft eine dem normalen Sprachgebrauch äusserst ähnliche Bedeutung. Kraft entsteht aus einem wirkenden Potential, z.B. die Erdanziehungskraft aus der Gravitation(sfeld), oder eine elektrische Kraft aus dem elektrostatischen Potential(feld). Wenn ich in Wikipedia lese, das die physikalische Wirkung mit dem alltagsgebräuchlichen Begriff nichts zu tun hat, so ist das schlichtweg Humbug. Der Text in Wikipedia ist eigentlich zum Löschen bestimmt, er enthält nichts im Hinblick auf die Lorenzinvarianz der Wirkung im Hamilton-Jakobischen-Sinn, und ist auch noch falsch, wenn er behauptet, das „physikalische Bahnen die kleinste Wirkung hätten“ – wie kann man so einen Quatsch schreiben ?
    Physikalische Bahnen eines Objektes sind die, längs denen die Wirkung auf das Objekt extremal ist, die physikalische Wirkung im obigen Sinne wohlbemerkt. Das Prinzip der kleinsten Wirkung hat schon den falschen Namen, es handelt sich nämlich bei der Beschreibung nicht um ein Minimalprinzip, sondern um ein Extremalprinzip.
    Wie auch immer und der langen Rede kurzer Sinn: diese allgemeinen Grundüberlegungen zu mechanischen Systemen oder zur Wirkung insgesamt müssten eigentlich bekannt sein, und ich wundere mich sehr, dass hier so viele Bedenken auftauchen. Wird denn heute an den Universitäten nur noch Formalkram gemacht und geht man dort auf die Bedeutung der einzelnen physikalischen Begriffe garnicht mehr ein ? Ich bin leider schon sehr lange von dort weg.
    Also, Niels, ich habe Dir eine physikalische Begründung für das Prinzip der extremalen Wirkung gegeben und es gibt keinen Grund, sich über mich zu ärgern. Jeder kann seine Meinung zu einer Sache haben, gerade in der Physik ist auf deren Grunde garnichts klar, es ist so vieles stillschweigende Übereinkunft (vor allem in Wikipedia), diese Übereinkunft muss man nicht mitmachen. Es ist sehr instruktiv, die Diskussion über die Wikipedia Artikel nachzulesen, da merkt man, das vieles, was so allendgültig behauptet wird, nur Perspektive ist.

    Sorry, so lang sollte der Beitrag garnicht werden.
    Nur noch ein Hinweis: wenn die Wirkung als Potenial der Kraft aufgefasst wird, dass steht sie auch für ein Kraftfeld im herkömmlichen Sinne. Ob ich das Feld meine, das die Kraft bewirkt, oder das Wirkungspotential in Joule x sec, es ist nur eine andere Beschreibung. Aber ich sage: Wirlung ist lorenzinvariant und nicht beobachtbar, und eine beobachtbare physikalische Reaktion mit einer unbeobachtbaren Ursache zu verknüpfen (egal ob mit gedanklichem Feld oder unbeobachtbarer Wirkung) liefert Dir ein Realitätsproblem. Das ist genauso, also würdest Du behaupten, der liebe Gott steuert uns in der unserer Realität zu unseren Taten. Genauso steuert das Feld oder die Wirkung aus dem Mysterium kommend alle Obejkte längs Bahnen, an denen die Wirkung tangential extremal ist. So kann man (muss nicht) begründet behaupten, die Physik beruhe genauso auf Glauben an eine höhere Macht wie die Religion. Ich habe bescheidener formuliert, dass es ein Wirkungsgefüge ausserhalb von Raum und Zeit gibt, dass die Kräfte hervorrufen.
    Die Quantenfeldtherie sagt in ihrer Theorie der Wechselwirkungen auch nichts anderes. Virtuelle Austauschteilchen, die aus dem Vakkum kommen, kommen auch von ausserhalb von Raum und Zeit.

    Es wird jetzt wirklich zuviel.
    Grüsse Fossilium

  102. #102 MartinB
    25. Oktober 2012

    @Niels
    “Läuft das irgendwie auf Pfadintegral hinaus?”
    Ja liest du denn meine QFT-Serie nicht midnestens einmal täglich?
    https://scienceblogs.de/hier-wohnen-drachen/2011/10/24/qft-fur-alle-von-der-quantenmechanik-zur-klassischen-physik/

    @Fossilium
    “Der Kern der Relativistätstheorie ist ja, dass es genau diese e i n e für alle geltende Geometrie (d.h. gleiche Abstände für alle) nicht gibt”
    Genau das ist *nicht* der Kern der SRT, die sagt das exakte Gegenteil nämlich:
    Es gibt eine Raumzeit, die universell gültig ist. Unterschiedliche Beobachter haben unterschiedliche Raum- und Zeit-Achsen, aber die Raumzeit ist für alle gleich. Deswegen kann man ja auch Minkowski-Diagramme malen und da Ereignisse eintragen. Unterschiedliche Bezugssysteme kann ich im selben Minkowksi-Diagramm unter unterschiedlichen Winkeln eintragen, aber das Diagramm ist dasselbe – genau wie unterschiedliche Landkarten unterschiedlich ausgerichtet sein können (Norden Oben, Westen Oben etc.), aber das Land trotzdem dasselbe ist.
    Ich habe genau diesen Punkt jetzt schon bestimmt 5 mal erklärt – wenn du immer noch darauf beharrst, dass es anders ist, dann musst du bitte mal prüfen, wo dein fundamentales Missverständnis liegt.

    “Wirkung einfach definieren als Kraft mal Weg mal Zeit,”
    Arrgh. Nein, kann man nicht. Man muss drei unterschiedliche Wirkungsbegriffe unterscheiden:
    1. Der Begriff “Wirkung” im Alltag (etwas “wirkt” = “etwas verursacht etwas anderes”)
    2. Der Begriff “Wirkung” als physikalische Größe (Alles, was die Einheit Energie mal Zeit hat)
    3. Der Begriff Wirkung als Integral über die Lagrangefunktion.
    Diese drei Dinge bezeichnen alle nicht dasselbe – entgegen deiner Annahmen.

    “Das meine ich, wenn ich sage: wir messen in erster Linie immer Energie und Impuls.”
    Wie du aber selbst sagst, tun wir das in Wahrheit gar nicht, sondern messen vor allem Orte und Zeiten. Es ist schon schwierig, dich zu verstehen…

    “er enthält nichts im Hinblick auf die Lorenzinvarianz der Wirkung im Hamilton-Jakobischen-Sinn”
    Kannst du bitte nochmal erklären, in welchem Sinne die Wirkung Lorentzinvariant ist? Über die Definition Integral L dt entlang Pfad kann ich das nicht nachhvollziehen.

    “diese allgemeinen Grundüberlegungen zu mechanischen Systemen oder zur Wirkung insgesamt müssten eigentlich bekannt sein,”
    Da diese allgemeinen Grundüberlegungen zum guten Teil falsch sind (s.o.), werden sie an Unis auch nicht gelehrt. (Sorry, aber arrogant kann ich auch 😉 )

    “Wirkungspotential”
    Was soll denn das schon wieder sein?

  103. #103 Niels
    25. Oktober 2012

    @MartinB

    Ja liest du denn meine QFT-Serie nicht midnestens einmal täglich?

    😳
    In Zukunft werde ich sie wohl zweimal täglich lesen müssen.

    Kannst du bitte nochmal erklären, in welchem Sinne die Wirkung Lorentzinvariant ist?

    Nachdem man zeigen kann, dass das Prinzip der kleinsten Wirkung in der Mechanik mathematisch dasselbe aussagt wie die newtonschen Gesetze, kann das doch nicht stimmen.
    Oder ist die newtonsche Physik seit neuestem auch lorentzinvariant?
    Und warum soll es eigentlich nochmal eine Rolle spielen, ob die Wirkung (oder das totale Differential der Wirkung?) invariant ist?

    @Fossilium

    Wird denn heute an den Universitäten nur noch Formalkram gemacht und geht man dort auf die Bedeutung der einzelnen physikalischen Begriffe garnicht mehr ein ?

    Na ja.
    Wenn man heutzutage in einer Prüfung behauptet, dass Kraft x Weg die Definition für die Energie ist (statt die Definition von diesem hier), dass Kraft x Weg eine Erhaltungsgröße ist oder dass die Wirkung ein Kraftfeld (oder ein Potential oder beides) ist, bekommt mit Sicherheit Probleme.
    Ich glaube aber, dass das früher auch nicht anders war.

    Virtuelle Austauschteilchen, die aus dem Vakkum kommen, kommen auch von ausserhalb von Raum und Zeit.

    Echt? Quelle?

    Also, Niels, ich habe Dir eine physikalische Begründung für das Prinzip der extremalen Wirkung gegeben

    Wo denn?
    Du erzählst falsche Dinge über die Wirkung und behauptest Verschiedenes über ihr Verhalten unter Lorentztransformationen.
    (Wobei es ursprünglich um das totalen Differentials der Wirkung ging, jetzt sind wir aber auf einmal doch bei der Wirkung selbst?)
    Außerdem redest du noch Unsinn über die SRT.

    Das Prinzip der kleinsten/stationären Wirkung habe doch ich selbst erst zum Thema gemacht, vorher war davon nie die Rede.
    Und anschließend war deine einzige Bemerkung dazu, dass dieses Prinzip den falschen Namen hätte. Darüber hinaus hast du nur deine alten Behauptungen wiederholt und ein paar neue, zusätzliche Fehler eingebaut.

    es gibt keinen Grund, sich über mich zu ärgern

    Doch, den gäbe es schon.
    Du ignorierst einfach alle Hinweise, Antworten und Fragen, die man dir stellt und reagierst nur mit einer Wiederholung des schon Gesagten.

    Eigentlich ärgere ich mich aber gar nicht.
    Ich habe nur bemerkt, dass wir nicht über Physik diskutieren können, solange du dich auch auf Nachfrage hin weigerst, Begriffe ihrer physikalischen Bedeutung gemäß zu verwenden.
    Das ist tatsächlich meine Überzeugung. Ob ich mich ärgere oder nicht spielt dabei keine Rolle.

    diese allgemeinen Grundüberlegungen zu mechanischen Systemen oder zur Wirkung insgesamt müssten eigentlich bekannt sein, und ich wundere mich sehr, dass hier so viele Bedenken auftauchen

    Ich habe dich doch schon mehrfach nach Belegen, Quellen und Zitaten gefragt. Das sollte doch im Fall von allgemeinen Grundlagen nicht so furchtbar schwer fallen?
    (Deine mehrfache Beteuerung, dass alle Artikel in der deutschen Wikipedia (warum nicht wie vorgeschlagen in die englische Ausgabe schauen?) darüber falsch sind, helfen übrigens kein bisschen weiter. )

  104. #104 Fossilium
    26. Oktober 2012

    Hi Martin,
    Du schreibst: „.. wenn du immer noch darauf beharrst, dass es anders ist, dann musst du bitte mal prüfen, wo dein fundamentales Missverständnis liegt.“
    Offen gesagt, mir vergeht die Lust bei dieser Art der Argumentation. Ich denke ich habe mir einige Mühe gegeben, bei der der Suche nach Missverständnissen. Warum begibst Du dich nicht mal auf die Suche ?
    Du könntest so fündig werden: Ich kann natürlich mir eine Geometrie ausdenken, und die in den Raum hineingelegt denken, die all die unterschiedlichen Längen, die wir beide beobachten, wegfiltert, sozusagen so hinbiegt, dass meine Längen und Deine gleich sind. Dazu brauche ich einen viedimensionalen Vektorraum mit komplexwertigen Vektoren. Wenn Du nun davon ausgehst, dass die Realität diesem Vektorraum entspricht, dann hast Du nicht nur das gleiche Problem mit der Realität wie in der Quantenemechanik, wo du in deinem eigenen obigen Text lang und breit beschrieben hast, wie problematisch der Realitätsbezug der komplexwertigen Wellenfunktion ist, sondern Du hat mein Anliegen garnicht verstanden. Ich habe behauptet, das wir als Beobachter unserer gemeinsamen (gemeinsam vereinbarten) Realität alle etwas anderes beobachten, jeder andere Längen und andere Zeiten. In diesem Sinne hat jeder seine eigene Raumzeit, d.h. hier: er hat seinen eigenen Raum und seine eigene Zeit, jeder seinen eigenen dreidimensionalen Raum in Echtzeit. Du redest die ganze Zeit von einem Vektorraum mit komplexwertigen Vektoren, der die Wirklichkeit so beschreibt, dass alle Unterschiede wegnivelliert werden, und unterstellst möglicherweise sogar, dass unsere Zeit und unserer Raum in Wirklichkeit dieser Vektorraum ist. Ja mein Gott, man kann das alles so sehen. Jeder hat das Recht, die Welt so zu sehen wie er will. Missverständnisse kommen auf wenn einer den anderen nicht verstehen will.
    Ich habe den von mir verwendeten Begriff der Wirkung von Grund auf entwickelt und begründet. Du behauptest nun, das der nicht stimme, gibst aber keine Begründung. Meinetwegen. Jeder kann sich damit zufriedengeben, dass der Begriff in einer Formel so und so heisst, und ihn ansonsten nicht weiter hinterfragen, obwohl er in den wichtigsten Grundgleichnungen der Physik die Hauptrolle spielt.
    Hi Niels,
    Du schreibst:
    „Du erzählst falsche Dinge über die Wirkung und behauptest Verschiedenes über ihr Verhalten unter Lorentztransformationen.
    (Wobei es ursprünglich um das totalen Differentials der Wirkung ging, jetzt sind wir aber auf einmal doch bei der Wirkung selbst?)
    Außerdem redest du noch Unsinn über die SRT.“
    Ich denke es hat keinen Zweck, weitere Ausführungen zu machen, die Du eh nicht nachvollziehen willst. Ich kann auch nicht sehen, dass Du eigene Gedanken entgegenhältst. Du verweist immer nur auf andere Experten, die das und das gesagt hätten, und ich sollte auch noch andere Experten anführen. Also ich kann meine Behauptungen selbst begründen. Ich denke ich habe das sogar in ziemlicher epischer Breite getan. Es gibt leider keinen heiligen Gral der Wahrheit in der Physik, auf dem Grunde schwimmt alles, und in den Höhen ist das richtig, was praktisch ist. Insofern kommt es mir auch nicht darauf an, unbedingt das letzte Wort zu haben. Du kannst in jedem guten Lehrbuch nachlesen, welche Grössen lorenzinvariant sind, oder quäle Dich durch die kanonischen Transformationen, um den Lagange Formalismus zu am Ende nur formal zu verstehen, wenn Du meiner simplen Erklärung nicht glaubst. Jeder muss selbst sehen, wie er zurechtkommt.
    Grüsse Fossilium

  105. #105 MartinB
    26. Oktober 2012

    @Fossilium
    “Warum begibst Du dich nicht mal auf die Suche ?”
    Weil ich – anscheinend im gegensatz zu dir – weiß wie der begriff “Raumzeit” in der Physik verwendet wird.

    “Dazu brauche ich einen viedimensionalen Vektorraum mit komplexwertigen Vektoren.”
    Du willst die Raumzeit konsistent mit einem komplexwertigen 4D-Vektorraum beschreiben? Wie geht das denn?

    “hat jeder seine eigene Raumzeit, d.h. hier: er hat seinen eigenen Raum und seine eigene Zeit,”
    Und das ist wieder dein fundamentales Missverständnis: Raumzeit ist nicht einfach die Summe aus Raum und Zeit. Siehe das berühmte Minkowski-Zitat.

    “ch habe den von mir verwendeten Begriff der Wirkung von Grund auf entwickelt und begründet”
    Er hat aber nicht viel mit dem Wirkungsbegriff der Physik zu tun – wenn du eien Begriff umdefinierst, dann kannst du nicht erwarten, dass alles, was auf die Original-Definition zutrifft, auch auf den neuen begriff zutrifft. Deswegen solltest du das, was du dir da ausgedacht hast, anders nennen (und bitte präzise definieren).
    Warum deine oder die handelsübliche Wirkung lorentzinvariant sein soll (das kann sie in der normalen Physik gar nicht sein, wenn die Wirkung in der QM die Rotation der Phase angibt, weil Rotationsfrequenzen nicht lorentzinvariant sind), hast du auch nicht erklärt.

    “Jeder muss selbst sehen, wie er zurechtkommt.”
    Das mag in der Philosophie funktionieren, in der Physik gibt es gesicherte Erkenntnisse, an denen man nicht vorbeikommt. Eine davon ist, dass die Wirkung nicht lorentzinvariant ist. Wenn du ein neues fundamentales und lorentzinvariantes Konzept definieren willst, dann solltest du es nicht Wirkung nennen.

  106. #106 Niels
    26. Oktober 2012

    @MartinB

    Du willst die Raumzeit konsistent mit einem komplexwertigen 4D-Vektorraum beschreiben?

    Fossilium wirft doch vielmehr dir vor, dass du die Raumzeit so beschreibst:
    Du redest die ganze Zeit von einem Vektorraum mit komplexwertigen Vektoren, der die Wirklichkeit so beschreibt, dass alle Unterschiede wegnivelliert werden, und unterstellst möglicherweise sogar, dass unsere Zeit und unserer Raum in Wirklichkeit dieser Vektorraum ist.
    Anscheinend hat Fossilium auch keine Ahnung davon, wie der Minkowski-Raum definiert ist.
    Wahrscheinlich kommt sein Fehlschluss mit den komplexen Vektoren daher, dass manchmal die Notation i *c*t verwendet wird, also eine komplexe Zahl auftaucht.

    @Fossilium

    Du kannst in jedem guten Lehrbuch nachlesen, welche Grössen lorenzinvariant sind

    Das habe ich getan. In keinem mir bekannten Lehrbuch steht dass die Wirkung loren t zinvariant ist.
    Wie gesagt, aus verschiedenen Gründen kann das auch gar nicht stimmen.
    Deswegen habe ich nach einer Quelle, einem Zitat oder einem sonstigen Beleg gefragt.
    Offenbar kannst du so etwas aber nicht liefern.

    Du verweist immer nur auf andere Experten, die das und das gesagt hätten, und ich sollte auch noch andere Experten anführen. Also ich kann meine Behauptungen selbst begründen. […] Es gibt leider keinen heiligen Gral der Wahrheit in der Physik, auf dem Grunde schwimmt alles,

    Das ist Unsinn. Definitionen und Axiome müssen eindeutig sein, da kann nichts schwimmen. Da kann man auch nicht einfach anderer Meinung sein.

    Übrigens:
    Wenn alle Experten anderer Ansicht sind als man selbst, sollte man darüber vielleicht mal nachdenken.
    Wenn es um solche grundlegenden Dinge wie die theoretische Mechanik oder die spezielle Relativitätstheorie geht, gilt das sogar in noch deutlich größerem Maße.
    Wenn die vielen Millionen Menschen, die sich auch schon damit befasst haben, alle zu einem anderen Ergebnis gekommen sind als man selbst, hat das mit unfassbar großer Wahrscheinlichkeit sehr gut Gründe.

  107. #107 MartinB
    26. Oktober 2012

    @Niels
    Ach so – Fossilium meint, dass ich den unendlichdimensionalen Hilbertraum für die Raumzeit halte? Das ergibt ja noch weniger Sinn…

    “also eine komplexe Zahl auftaucht.”
    Eigentlich nur eine imaginäre Zahl – so oder so hat der Vektorraum 4 reele Freiheitsgrade, nicht 8. Aber keine Ahnung, ob Fossilium das meint…

  108. #108 Niels
    26. Oktober 2012

    @MartinB
    Eigentlich nur eine imaginäre Zahl
    Mein Fehler.
    Mir ist übrigens durchaus klar, dass der Minkowski-Raum keine acht reellen Freiheitsgrade hat. 😉

    Fossilium meint, dass ich den unendlichdimensionalen Hilbertraum für die Raumzeit halte?

    Fossilium wirft dir vor, einen viedimensionalen Vektorraum mit komplexwertigen Vektoren zu verwenden, oder verstehe ich das auch schon falsch?
    Fossilium hat aber irgendwo am Anfang selbst geschrieben:
    die Elemente der QM (Wellenfunktion) sind komplexwertige Funktionen in einem n-dimensionelan – letzendlich unendlich dimensionalen – Hilbertraum</i.
    An 4-dim. Vektorräumen bleibt also nur noch der Minkowski-Raum übrig, deswegen denke ich, dass es darum gehen muss.

    War aber nur eine Idee.
    Vielleicht meint Fossilium doch den Hilbert-Raum, der eben auf einmal vierdimensional ist.
    Vielleicht er vermischt er auch irgendwie beides?

  109. #109 MartinB
    26. Oktober 2012

    “Mir ist übrigens durchaus klar, dass der Minkowski-Raum keine acht reellen Freiheitsgrade hat. ”
    Und ich dachte schon, ich müsste dich auf einen meiner Artikel verweisen, sowas soll ja vorkommen 👿

    So wie ich Fossilium verstehe, kann er selbst den 4D-komplexen Vektorraum so konstrueiren, dass er alle zulässigen geometrien gleichzeitig enthält….?

  110. #110 Niels
    26. Oktober 2012

    Nochmal:
    @MartinB

    Eigentlich nur eine imaginäre Zahl

    Mein Fehler.
    Mir ist übrigens durchaus klar, dass der Minkowski-Raum keine acht reellen Freiheitsgrade hat. 😉

    Fossilium meint, dass ich den unendlichdimensionalen Hilbertraum für die Raumzeit halte?

    Fossilium wirft dir vor, einen viedimensionalen Vektorraum mit komplexwertigen Vektoren zu verwenden, oder verstehe ich das auch schon falsch?
    Fossilium hat aber irgendwo am Anfang selbst geschrieben:
    die Elemente der QM (Wellenfunktion) sind komplexwertige Funktionen in einem n-dimensionelan – letzendlich unendlich dimensionalen – Hilbertraum
    An 4-dim. Vektorräumen bleibt also nur noch der Minkowski-Raum übrig, deswegen denke ich, dass es darum gehen muss.

    War aber nur eine Idee.
    Vielleicht meint Fossilium doch den Hilbert-Raum, der eben auf einmal vierdimensional ist.
    Vielleicht er vermischt er auch irgendwie beides?

  111. #111 Niels
    26. Oktober 2012

    @MartinB
    Dieses “ich” verwendet doch so wie oben das “wir Physiker”, oder?
    Von so einer genialen Eigenentwicklung war jedenfalls vorher noch nie die Rede.

    Und ich dachte schon, ich müsste dich auf einen meiner Artikel verweisen, sowas soll ja vorkommen

    Frechheit.
    Ich habe Sie nur getestet und Ihnen mit dem Stichwort Pfadintegral sogar noch eine goldene Brücke gebaut.
    Trotzdem waren Sie nicht in der Lage, den Sachverhalt in einem kurzen Satz zu paraphrasieren.
    Das gibt mit Gnade gerade noch ein Ausreichend, Herr Bäker.

  112. #112 MartinB
    26. Oktober 2012

    “Dieses “ich” ”
    Huh wu- da hab ich jetzt den Bezug verloren…

    “Ich habe Sie nur getestet und Ihnen mit dem Stichwort Pfadintegral sogar noch eine goldene Brücke gebaut.”
    Tja, aber ich bin ja kein Doktorand mehr:
    Der Doktorand muss es wissen,
    der Post-Doc muss wissen wo es steht
    Der Prof. muss wissen, wo der nächste Post-Doc ist…

    (Wieso Siezen wir uns plötzlich?)

  113. #113 Niels
    26. Oktober 2012

    @MartinB

    Huh wu- da hab ich jetzt den Bezug verloren…

    Kein Wunder.
    Scienceblogs braucht dringend wieder eine Kommetarvorschau. In letzter Zeit schick ich immer mal wieder Sätze ab, die keinen Sinn ergeben.
    Vielleicht brauch ich aber auch einfach Urlaub. Oder Schlaf.

    Gemeint war:
    “Dazu brauche ich einen viedimensionalen Vektorraum mit komplexwertigen Vektoren” = irgend jemand hat irgend wann mal hergeleitet, dass man dazu so etwas braucht
    So wie seine Aussagen über die Wirkung einerseits “von Grund auf selbst entwickelt und begründet” hat, sie aber andererseits angeblich auch als absolute Grundlagen in jedem Lehrbuch zu finden sind.

    Wieso Siezen wir uns plötzlich?

    Weil ich so deutlich machen wollte, dass das überhaupt nicht ernst gemeint ist. War aber zugegebenermaßen weder witzig noch irgendwie clever.
    Sorry.:oops: Ich hab mal wieder so eine Woche…

  114. #114 Niels
    26. Oktober 2012

    Hm, wird der Smiley 😳 nicht unterstützt?
    Hab ich den nicht schon mal verwendet?

  115. #115 MartinB
    26. Oktober 2012

    @Niels
    “irgend jemand hat irgend wann mal hergeleitet, dass man dazu so etwas braucht”
    Ja, das hat Fossilium sich wohl irgendwie überlegt – wie genau auch immer…

    “War aber zugegebenermaßen weder witzig noch irgendwie clever.”
    Nö, hatte ich schon verstanden, mir fiel nur selbst gerade keine tolle Antwort ein…

  116. #116 Fossilium
    28. Oktober 2012

    Hallo Martin,

    Du schreibst:
    „Warum deine oder die handelsübliche Wirkung lorentzinvariant sein soll (das kann sie in der normalen .. (?) … Physik gar nicht sein, wenn die Wirkung in der QM die Rotation der Phase angibt, weil Rotationsfrequenzen nicht lorentzinvariant sind), hast du auch nicht …“

    Ich habe im Interent ein bisschen gegoogelt und tatsächlich findet man nicht viel zur Lorenzinvarianz der Wirkung.

    Dass die Wirkung S, das Zeitintegral der Lagangefunktion, lorenzinvarant ist, ist z.B. hier explizit vermerkt:

    https://www.phys.ethz.ch/~mrg/QFT/QFT.pdf

    In den meisten Fällen findet sich die Lorenzinvarianz in der Aussage wieder, das sich die Wirkung unter Eichtransformationen nicht ändet, z.B. hier:

    https://www.ph.tum.de/studium/praktika/ferienkurse/2010w/fk_edyn_02_skript.pdf

    Dass das Zeitintegral der Lagrangefunktion poincareinvariant ist (was ja Lorenzinvarianz einschliesst), findet sich allerdings an sehr vielen Stellen.

    Ich hatte in meinem Kommentar von 22.10.12 eine formale mechanische Definition des Begriffs „Wirkung“ gegeben, im Kommentar vom 20.10.12 hatte ich Wirkung als Potential des Vierer-Impuls-Energie-Vektors auch für den relativistischen Fall definiert (dieser Vektor kommt im Internet ziemlich häufig vor), es gibt also klare formale Definitionen von mir.

    Wenn Du daher behauptest, ich hätte Wirkung nicht definiert, dann hast Du die Definitionen wohl überlesen.

    Ich habe aber auch das Lagrange Integral als Wirkung im umgangssprachüblichen Sinn gedeutet, und eine Reihe von Autoren im Internet sehen das genauso. Der Lagrangeformalismus ergibt sich ja daraus, dass die Zwangskräfte aus den Newtonschen Bewegungsgleichungen eleminiert werden – und dann eben nur noch die Wirkkräfte übrigbleiben (Wirkung im Sinne von Kraftwirkung verstanden).

    Eine sehr anschauliche und mit meinen Vorstellungen über Wirkung und Lorenzinvarianz der Wirkung
    vollkommen übereinstimmende Darstellung findest Du hier:

    https://www.tphys.physik.uni-tuebingen.de/braeuer/Skripte/Ganzheitl_Physik.pdf

    Du lehnst zwar meine Äusserungen zur Definition der Lagrangefunktion, zum Begriff der Wirkung, zur Lorenzinvarianz ab, sagst aber nicht, was Du darunter verstehst. Deine physikalische Deutung des Lagrange Integrals steht nach wie vor aus.

    Leider ist Deine obige Bemerkung auch nicht sehr hilfreich. Wirkung ist eine physikalische Grösse und wird in physikalischen Einheiten gemessen, die Phase einer Funktion ist aber eine Grösse ohne Einheit. Deswegen kannst Du Phase nicht gleich Wirkung setzen.

    Noch ein Hinweis: Die zuletzt genannte Quelle lohnt sich, vollständig zu lesen, dort wird ein Äquivalent zur Schrödingergleichung aus realistischen Grössen durch Kontinuitätsüberlegungen zu einem Wahrscheinlichkeitsstrom verbunden mit einem Unschärfepotential abgeleitet. Finde ich sehr interessant. Die Wirkung wird darin übrigens als wichtigster physikalischer Begriff überhaupt angesehen.

    Grüsse Fossilium

  117. #117 MartinB
    28. Oktober 2012

    @Fossilium
    Es ist etwas mühsam, sich jetzt durch 129 Seiten Skript zu hangeln – kannst du die genaue Stelle angeben?

    “Deine physikalische Deutung des Lagrange Integrals steht nach wie vor aus.”
    Nein, ich habe die oben verlinkt (QFT-Serie Kap. 4/5): Die Wirkung gibt die Rotation der Wahrscheinlichkeitsamplitude an.
    Damit ist, wenn ich es richtig sehe, das Integral entlang eines Pfades tatsächlich lorentzinvariant, weil die Anzahl der Drehungen entlang des Pfades eindeutig sein muss, die Lagrange-Funktion (also das Differential der Wirkung) ist aber nicht invariant. (Niels, sehe ich das richtig?)

    “Deswegen kannst Du Phase nicht gleich Wirkung setzen. ”
    Ja, man muss durch ħ teilen, das hielt ich jetzt mal für selbstverständlich…

    “als Potential des Vierer-Impuls-Energie-Vektors”
    Was ist denn das Potential eines Vierer-Vektors?

    Was die Quelle von Bräuer angeht: wenn ich solche Sätze lese:
    “In der Quantenmechanik berücksichtigt man nun diese offensichtlichen Mängel klassischer raum-zeitlicher Beziehungen, indem man mit statistischen Mitteln die
    Unschärfe der raumzeitlichen Punkte berücksichtigt.”
    dann wundere ich mich schon – seit wann arbeit die QM mit einer Unschärfe der raumzeitlichen Punkte?

    Und Wirkungen können maximal bis auf ein Wirkungsquantum genau festgelegt werden? Wieso sollte das so sein?

    Und dann sehe ich Bild 2-2 und mir gruselts – was hat denn der Jungsche Begriff der “Wirkung” mit dem physikalischen zu tun?

    Und was ist das “Wirkungsfeld” auf Seite 17?

  118. #118 Niels
    28. Oktober 2012

    @Fossilium
    Was du mit deinem „viedimensionalen Vektorraum mit komplexwertigen Vektoren” meinst möchtest du offenbar nicht klarstellen?

    Ich hatte in meinem Kommentar von 22.10.12 eine formale mechanische Definition des Begriffs „Wirkung“ gegeben, im Kommentar vom 20.10.12 hatte ich Wirkung als Potential des Vierer-Impuls-Energie-Vektors auch für den relativistischen Fall definiert (dieser Vektor kommt im Internet ziemlich häufig vor)

    Das wäre also relativistisch (20.10):
    Die Wirklung nun wird in der Physik als zeitliche Änderung der Energie oder als örtliche Änderung des Impulses (in einer Richtung) definiert.
    Die so definierte Wirkung ist daher als Potiential des Vierer-Energie-Impulsvektors aufzufassen

    und nicht-relativistisch (22.10):
    In der nichtrelativ. Mechanik ist die Energie E die Ableitung der Wirkung nach der Zeit : E = dW/dt und der Impuls ist die Ableitung der Wirkung nach dem Ort : p = dW/dx
    Außerdem gilt noch (22.10):
    Das totale Differential dW = pdx – Edt ist lorenzinvariant – die einzige dynamische Grösse, die unabhängig ist von Zeit und Raum

    Relativistisch gilt also: W = dE/dt oder W = dp/dx.
    Nicht-relativistisch gilt: E = dW/dt und p = dW/dx
    Relativistisch zudem: dW = pdx – Edt

    Das ergibt echt total viel Sinn, super.
    Übrigens solltest du genau diese Formeln belegen. Außerdem noch, dass die so definierte (!) Wirkung und dieses (!) totale Differential lorentzinvariant sind.
    Deine Antwort dazu, dass nämlich der Viererimpuls im Internet ziemlich häufig zu finden ist, kann ja wohl nur ein schlechter Witz sein.

    Eine sehr anschauliche und mit meinen Vorstellungen über Wirkung und Lorenzinvarianz der Wirkung vollkommen übereinstimmende Darstellung findest Du hier

    Endlich nennst du mal Ross und Reiter. Du hättest auch von Anfang an sagen können, dass du versuchst, dieses Script nachzuerzählen.
    Dort wird auch über die “Ausbreitungsgeschwindigkeit von Wirkungen” philosophiert, da kommt auch der von dir geschrieben Unsinn über die Raumzeit her
    Das kann nur bedeuten, dass Raum und Zeit keine eigenständige, vom Beobachter unabhängige Existenz haben
    […]
    Die Raumzeit, die wir uns vorstellen, gibt es nicht wirklich und unser Weltbild sollte das berücksichtigen

    und auch deine anderen „selbst entwickelten“ Ideen haben offensichtlich dort ihren Ursprung.
    Hier findet man auch endlich die Gleichung
    dS = pdx * Edt
    Wobei S das “Wirkungsfeld”, p das “Impulsfeld” und E das “Energiefeld” sind.

    Dort steht allerdings auch
    Die Grundlage unserer Betrachtungen ist das Wirkungsfeld. Als Feld ist es eine Größe, die von Raum und Zeit abhängt.
    Du benutzt dieselbe Formeln, behauptest aber als zentrale Aussage über die Realität:
    Die Sicht, dass hier ein nicht beobachtbares Wirkungsfeld ausserhalb von Zeit und Raum in die Realität hineinwirkt, hört sich metaphysisch an, ist im Endeffekt aber plausibler, als das Feldkonzept
    Wie kann das Wirkungsfeld von Raum und Zeit abhängen und gleichzeitig außerhalb von Raum und Zeit sein?

    @Fossilium @MartinB
    Man kann die die Euler-Lagrange-Gleichung in eine relativistisch kovariante Form überführen, wenn es um Felder geht.
    Die Wirkung einer Feldtheorie ist invariant unter Poincaré-Transformationen.
    Die Lagrange-Funktion ist allerdings trotzdem nicht invariant.
    In diesen Fällen (und wenn die Feldtheorie lokal ist) benutzt man dann häufig die Lagrange-Dichte und integriert über die Raumzeit. Diese Lagrange-Dichte ist dann eine Poincaré-Invariante.
    https://upload.wikimedia.org/math/4/4/8/44850a2d921d4b9933793affdf324e26.png
    Die Euler-Lagrange-Gleichung ist dann
    https://upload.wikimedia.org/math/8/3/6/836aaa6a72661fdca9d9e354ff25f2d8.png
    Habe ich übrigens oben schon mal angesprochen.
    Siehe zum Beispiel die Elektrodynamik mit dieser Lagrange-Dichte: https://upload.wikimedia.org/math/9/d/b/9db803f9b0c1f2b69dfa6b9961ff2741.png
    Da ist dann natürlich auch die Wirkung lorentzinvariant.

    Es geht hier aber doch gar nicht um Feldtheorien, sondern um die newtonsche, klassische Mechanik, um allgemeine Grundüberlegungen zu mechanischen Systemen.
    Tatsächlich geht es doch sogar ganz speziell nur um die von Fossilium aufgestellten Formeln als Definitionen für die Wirkung. Jedenfalls hab ich das bisher so verstanden.

    Mit dem totalen Differential hat das alles aber trotzdem nichts zu tun.

    @MartinB

    Damit ist, wenn ich es richtig sehe, das Integral entlang eines Pfades tatsächlich lorentzinvariant, weil die Anzahl der Drehungen entlang des Pfades eindeutig sein muss, die Lagrange-Funktion (also das Differential der Wirkung) ist aber nicht invariant

    Ich glaube auch, dass die Wirkung hier lorentzinvariant ist, die Lagrange-Funktion dagegen nicht.
    Es muss ja am Integral liegen, aber über die Sache mit den Drehungen muss ich noch mal nachdenken.

    Und was ist das “Wirkungsfeld” auf Seite 17?

    In diesem Text gibt Bräuer direkt am Einfang einen Überblick über seine “Ganzheitliche Theorie der Physik”:
    https://www.uni-tuebingen.de/fileadmin/Uni_Tuebingen/Fakultaeten/MathePhysik/Institute/ITP/Braeuer/Skripte/2011_Ganzheitliche_Physik/2011_Physik_Ganzheitlich.pdf
    Für die Lorentz-invariante Behandlung eignet sich das Wirkungsfeld. Es ist ein skalares Feld, also unabhängig vom Bezugssystem, und kann als Potential des Impulsfeldes aufgefasst werden.
    Steht wahrscheinlich auch irgendwo im langen, von Fossilium verlinkten Script. Hab ich aber nur ganz kurz überflogen.

    was hat denn der Jungsche Begriff der “Wirkung” mit dem physikalischen zu tun?

    Ganz furchtbar viel. Ist doch klar. Steht so in jedem grundlegenden Lehrbuch. 🙄
    Übrigens glaubt jemand wie Bräuer natürlich auch nicht an die ART:
    Die Schwarzschild-Metrik und die Verhältnisse auf dem Ereignishorizont haben sich hier allein aus der Lorentz-Invarianz und der Annahme eines erhaltenen Impulsgradienten ergeben. Üblicherweise gewinnt man diese Ergebnisse aus den Einstein-Gleichungen der ART. Dort wird vor allem auch die Energie des Gravitationsfeldes berücksichtigt und führt zu einer komplizierten Nichtlinearität.
    Hier wirft sich die Frage auf, ob diese Energie überhaupt zu berücksichtigen ist. Es sieht doch so aus, als wären Gravitationsfeld und Raumzeitkrümmung zwei äquivalente Konzepte, wobei jedoch das eine oder das andere gewählt wird. In den Einstein-Gleichungen der ART werden die Konzepte jedoch vermischt, sie werden gleichzeitig angewendet.
    Die Einstein-Gleichungen liefern Aussagen für den Innenraum der Schwarzen Löcher, die jedoch nicht überprüfbar sind.
    Alle bisherigen Beobachtungen zur ART werden durch die SchwarzschildMetrik richtig erklärt, und diese ergibt sich auch ohne die Einstein-Gleichungen

    Er weiß anscheinend gar nicht, worum es in der ART geht und was die Formeln wirklich aussagen.
    Seine Folgerung hat allerdings Hand und Fuß.
    Klar, bei starken Feldern wie beispielsweise bei den Pulsaren PSR1913+16 hat man natürlich mit der Schwarzschild-Metrik gerechnet. Kommt genau das Richtige raus. Aus dieser Metrik ergeben sich bekanntlich ja auch die Gravitationswellen. Und in der Kosmologie rechnet man ja sowieso nie mit etwas andrem als Schwarzschild.

  119. #119 MartinB
    28. Oktober 2012

    Sorry Niels,

    waren wohl zu viele links für unseren Filter…

  120. #120 MartinB
    28. Oktober 2012

    @Niels
    Ansonsten danke für die ausführliche Antwort.
    Nun wissen wir wenigstens wo diese “neue” Physik herkommt. Manchmal staunt man schon, wer alles so an Unis lehren darf…

  121. #121 Niels
    28. Oktober 2012

    @MartinB
    Bräuer hält darüber sogar regelmäßig eine Vorlesung, für die man sogar einen Credit-Point bekommt!
    https://www.uni-tuebingen.de/fileadmin/Uni_Tuebingen/Fakultaeten/MathePhysik/Institute/ITP/Braeuer/Vorles/2012_SS_PhilAMP/2012_SS_Philo_Aspekt.htm
    Da müsste man eigentlich echt mal ne Email an die Uni schreiben…

    Dort findet man übrigens eine Power-Point-Folie, die dir das mit Jung viel genauer erklären kann:
    https://www.uni-tuebingen.de/fileadmin/Uni_Tuebingen/Fakultaeten/MathePhysik/Institute/ITP/Braeuer/Vorles/2012_SS_PhilAMP/2012_SS_04_Unus_Mundus.pdf

    Die Zusammenfassung hab ich mal überflogen, macht einen guten Eindruck:
    https://www.uni-tuebingen.de/fileadmin/Uni_Tuebingen/Fakultaeten/MathePhysik/Institute/ITP/Braeuer/Vorles/2012_SS_PhilAMP/2012_SS_12_Zusammenfassung.pdf

    Klasse.
    Die anderen Folien sind wahrscheinlich auch lesenswert, merkt man ja schon an ihren Namen.

  122. #122 Niels
    28. Oktober 2012

    Auch schön:
    Eine einheitliche formale Begründung der hier vorgestellten Inhalte sind im Skript “Ganzheitliche Theorie der Physik” zusammengefasst.
    Dieses ist Grundlage der Modulveranstaltung ‘Theoretische Physik mit Maple’ im Wintersemester

    Da will man ganz unschuldig lernen, wie man ein Computeralgebrasystem bedient, und findet sich plötzlich in der Twilight Zone wieder. 😯 😆

  123. #123 rolak
    28. Oktober 2012

    Da will man ganz unschuldig…Twilight Zone

    Hübsch formuliert, Niels, fehlt nur an der Hörsaaltür ein Schild á la “Betreten auf eigene Gefahr”, denn wie hieß es im Intro immer so schön:

    It is a dimension as vast as space and as timeless as infinity. It is the middle ground between light and shadow, between science and superstition, and it lies between the pit of man’s fears and the summit of his knowledge. This is the dimension of imagination. It is an area which we call the Twilight Zone.

    Mensch beachte die innovative, bis heute gerne genutzte Verwendung des Wortes ‘Dimension’, welches -wir Deutschen sind ja ach so gründlich und erbsenzählerisch- afaik hierzulande zur ‘fünften Dimension’ synchronisiert wurde.

  124. #124 MartinB
    28. Oktober 2012

    Boah, das ist ja gruselig. Die Freiheit von Forschung und Lehre kommt halt auch mit nem Preis…

  125. #125 MartinB
    28. Oktober 2012

    Nachtrag:
    Obwohl ich das ja auch so mache: Da halte ich ne Vorlesung, die ganz unschuldig “Funktionswerkstoffe für Maschinenbauer” heißt, und dann juble ich den nichtsahnenden Studis Quantenmechanik und Festkörperphysik unter, muhaha.

  126. #126 Fossilium
    28. Oktober 2012

    Hallo Martin,
    Du schreibst:
    “Deswegen kannst Du Phase nicht gleich Wirkung setzen. ”
    Ja, man muss durch ħ teilen, das hielt ich jetzt mal für selbstverständlich…”

    Wieso selbstverständlich ? Wenn ich S durch ħ teile, habe ich ja gerade eine Grösse ohne Einheit.

    Ich habe den Text von Herrn Bräuer vor einiger Zeit gelesen und fand meine Vorstellung von Wirkung als Grundbegriff der Physik darin bestätigt. Bräuser stellt die Grundbegriffe der verschiedenen Teilbereiche der Physik und die grundlegenden Zusammenhänge sehr deutlich dar und leitet grundlegende Gleichungen auf sehr verständliche Weise ab. Das verdient Anerkennung. Es ist unredlich, daraus ein paar Sätze zu zitieren und diese abzuqualifizieren, ohne den Text als Ganzes zuvor zu würdigen. Ich bin auch nicht mit allem einverstanden, was er schreibt, aber ich finde den Text bemerkenswert und beurteile ihn insgesamt als positiv, auch aus didaktischer HInsicht. Jeder mathematsiche Formalismus in der Physik hat eine Bedeutung, und die gilt es aufzuzeigen, dass macht Bräuer besser als andere Autoren, die ich kenne.

    Niels schreibt hier:
    Du erzählst falsche Dinge über die Wirkung und behauptest Verschiedenes über ihr Verhalten unter Lorentztransformationen.
    Außerdem redest du noch Unsinn über die SRT.
    Das ergibt echt total viel Sinn, super
    kann ja wohl nur ein schlechter Witz sein.
    … dass du versuchst, dieses Script nachzuerzählen.
    Dort wird auch … philosophiert, da kommt auch der von dir geschrieben Unsinn über die Raumzeit her.

    Du wirst verstehen, dass ich auf die Argumentation von Niels nicht eingehe. Niels scheint zu einer sachlichen Diskussion nicht in der Lage zu sein. Ich wundere mich, dass Du persönliche abfällige Bemerkungen von Kommentatoren tolerierst und bitte Dich, hier einzuschreiten. Solltest Du mich nicht schützen wollen, werde ich Deinen Beitrag verlassen.
    Grüsse Fossilium

  127. #127 MartinB
    29. Oktober 2012

    @Fossilium
    Sorry, aber das hier ist Wissenschaft – wenn jemand Unsinn schreibt, dann schreibt er Unsinn. Bräuer definiert nicht mal, was sein “Wirkungsfeld” sein soll, will aber etwas daraus ableiten. Und der Versuch, Querverbindungen zu Jungs Archetypen herzustellen, ist dann wirklich abstrus.
    Es gibt da draußen unglaublich viele solche und ähnliche “Theorien”, die alle auf den ersten Blick sympathisch aussehen (und seit ich diesen Blog schreibe, bekomme ich mindestens einmal im Monat eine mail von jemandem, der mir seine Theorie zeigen will). Woran man nahezu sofort erkennt, ob so etwas Hand und Fuss haben könnte, ist an der Frage, ob und wie grundlegende Größen definiert werden.

    Und nur weil Bräuer sich nett liest, muss er nicht recht haben.

    “Niels scheint zu einer sachlichen Diskussion nicht in der Lage zu sein.”
    Nun ja, sachlich haben wir oben versucht…
    Was ist denn an “Du redest Unsinn” ein persönlicher Angriff? Du redest Unsinn, darf man das nicht sagen? Und dass du uns hier erst viele viele Kommentare hinschreibst, bevor du mal sagst, woher das alles stammt, ist wirklich ärgerlich – da hätten wir uns viel Zeit sparen können.

    “itte Dich, hier einzuschreiten”
    Tut mir leid, aber ich sehe keine persönliche Beleidigung. Es besteht ein Unterschied zwischen
    “Du schreibst Unsinn” (das lasse ich problemlos gelten) und z.B. “du bist doof” (da würde ich den Kommentator darum bitten, so etwas nicht mehr zu schreiben, da gab’s hier genügend Beispiele). Ersteres ist keine Beleidigung sondern eine – im Ton scharfe – Feststellung. Wenn du mir sagst, wo Niels eine persönlich abfällige bemerkung gemacht hat, dann werde ich gern einschreiten.

  128. #128 Fossilium
    30. Oktober 2012

    Hallo Martin,

    die Diskussion hier wird zunehmend unsachlicher und respektloser. Die Argumente werden nicht mehr gelesen, um verstanden zu werden, sondern um darin etwas zu finden, dem man möglicht heftig, oder ironisch, oder abfällig widersprechen kann. Es geht hier immer weniger um die Sache, sondern darum Recht zu haben. Du bist naiv, wenn Du das nicht erkennst.

    Es ist eine Bindenwahrheit, dass dieser Ton immer auf der Seite aufkommt, auf der die Unsicherheit zunimmt. Wer sich seiner Sache sicher ist, braucht diesen Stil nicht.

    Und braucht ihn auch nicht zu verteildigen.

    Grüsse Fossilium

  129. #129 MartinB
    30. Oktober 2012

    @Fossilium
    Wer keine Argumente hat, muss sich halt über den Tonfall beschweren…

  130. #130 Phero
    2. November 2012

    Oh Gott… jetzt hat man auch an anderen Orten in Deutschland unseren Bräuer entdeckt. In Tübingen waren schon mehrere Stundenten aus meinem Jahrgang in seiner Vorlesung “Philosophische Aspekte der Modernen Physik” , um ein bisschen Abwechslung vom tristen Unialltag zu haben und kräftig zu lachen.

  131. #131 MartinB
    3. November 2012

    @Phero
    Irgendwo ist so was ja auch traurig. Scheint aber ja ein häufig vorkommendes Phänomen bei älteren Wissenschaftlern zu sein, die vielleicht hoffen, noch mal zum Abschluss der Karriere die Welt zu erklären.
    Drückt mir die Daumen, Leute…

  132. […] Detailliert findet ihr das in diesem Blog ziemlich oft (zum Beispiel hier, hier, hier oder hier), deswegen gibt es hier nur eine […]

  133. #133 volker
    @martin
    22. November 2012

    Hallo Martin, ich bin wieder zurück unter den aktiv Lebenden, um genau 46 gr. leichter, und nun in der Reha, Es geht mir bestens, die Dichtigkeit (nicht die Dichte) liegt bei 95%.
    Ich habe eine Bitte: Ein guter Freund hat mir einen Büchergutschein geschenkt, den ich gerne in ein QFT-Buch umsetzten möchte. Du weißt, ich bin kein Mathematiker, aber auch kein greenhorn. Hättest Du eine Idee oder Empfehlung, was von der klassischen Physik zum Verständnis der Quantenwelt führt? Es darf schon etwas tiefer gehen als Gödel Escher Bach. Mit Dank auch für Deine guten Wünsche, HG, Volker

  134. #134 MartinB
    22. November 2012

    Hallo volker
    Wie sind denn die Randbedingungen – deutsch oder auch englisch? Wie mathematisch soll’s denn sein; echt für Studis oder noch eher im populärwissenschaftlichen angesiedelt?

  135. #135 volker
    @martin
    23. November 2012

    Deutsch oder English ist ok. Ingenieur -Mathe, d.h. Differential, Integral, Schwingungsgleichungen, Vektor- und Matrizenrechnungen. Die Rechenregeln und Zeichen in Formeln für Operatoren wie Lagrange, Hamilton ugl. sollten kurz erklärt sein. Klassische Dynamik und ED ist mir einigermaßen geläufig. So in der Art. Früher habe ich mir das physikalische Verständnis mehr vom Wirkprinzip als von der math. Formel hergeleitet. Deine Artikelserien habe ich auch mehrmals gelesen. Ich wollte halt immer verstehen, wie’s funktioniert. Mit Dank für Deine Bemühungen um einen “Altstudenten”, HG, Volker

  136. #136 MartinB
    23. November 2012

    O.k., wenn das so ist (du also echte hardcore-QFT betreiben willst), dann empfehle ich das Buch von Zee “QFT in a nutshell” als Starter – das ist zwar mathematisch oft etwas schlampig, erklärt aber die Ideen ganz gut.
    Ebenfalls gut (aber nicht ganz so anschaulich) ist Ryder “QFT”.
    Ansonsten schau mal auf die Internetseite qftfieldinfo von Bob Klauber (daraus soll auch demnächst ein Buch werden), da sind auch viele Aspekte gut erklärt (allerdings nicht die Frage, was ein Zustand ist).

    Generell findet man (mit etwas Suche) auch viele Dinge als pdf-Dateien im Netz. Schick mir doch mal ne private mail…

  137. […] Grund ist sicher, dass die Konsequenzen der Quantenmechanik für unser Bild der Wirklichkeit schwerer zu durchschauen sind, aber ich denke, es kommt hier noch […]

  138. […] in ganz unterschiedlicher Weise interpretiert werden kann. Über die Interpretation kann man sich beliebig lange streiten – zum Glück ist für alle praktischen Zwecke klar, was in einer bestimmten Situation […]

  139. #139 stein
    freiberg in sa.
    27. Mai 2013

    sehr geehrter herr Bäker,

    also, erst mal glückwunsch zu Ihren seriösen beiträgen !
    Ich hätte da was, woran sich kaum einer traut, aber mindestens genauso spannend ist wie quantentheorie und ko.
    ich mach es kurz, es geht um das sog. machsche prinzip.
    es ergeben sich viele fragen…ist es nun mit der art vereinbar oder nicht…gilt es oder nicht usw. usw.. , eine überaus spannende frage.
    hätten sie lust da mal was zu schreiben ?

    vielen dank

    stein

    chemiker i.R. und leiter sternwarte teichmühle

    und gestatten sie mir noch einen nachschlag….wo bleibt die energie eines rotverschobenen photons im expandierenden universum ?
    auf beide fragen finde ich seit jahren keine seriöse antwort

  140. #140 MartinB
    27. Mai 2013

    Hallo Herr Stein

    nein, das machsche Prinzip steht nicht auf meienr Liste – bin mir nicht sicher, was ich dazu schreiben sollte…

    “.wo bleibt die energie eines rotverschobenen photons im expandierenden universum ?”
    Dazu gibt’s irgendwo einen Spektrum-der-Wissenschaft-Artikel; wir haben das hier mal irgendwann diskutiert, aber ich erinnere mich nicht mehr, wann und wo das war. Kann sein, dass es dieser Artikel hier war:
    https://www.spektrum.de/alias/astronomie-physik/verliert-das-universum-energie/1044837

  141. #141 stein
    27. Mai 2013

    ja, danke, vielleicht klappts da mal irgendwann, der artikel in spektrum befriedigt mich leider nicht, kenne ihn, oder wir diskutieren ihn hier ?

    stein

  142. #142 Rinaldo
    8. August 2013

    Hallo Martin Bäker,

    ich bin auf Deinen Artikel gestoßen und danke Dir für diesen sehr gelungenen Artikel, nach dem ich schon lange gesucht habe.

    Ich arbeite seit einigen Monaten an einem Erklärungsansatz, und Dein Artikel hat mir geholfen, wichtige Einzelheiten zu analysieren. Ich hoffe, dass meine Überlegungen auch für Dich von Interesse sind.

    Meine Theorie ist sehr einfach: Alles ist lokal und realistisch durch eine konsequente, vorbehaltslose Anwendung der SRT von Einstein zu erklären.

    Hier meine Antwort in aller Kürze zu Deinen Raumzeitdiagrammen des Photons:

    1.-Man muss die Lichtwellen (v=300000 km/s) vom eigentlichen Photon unterscheiden, sie sind nämlich (nur) dessen überall wahrnehmbares Abbild, das der Zeitdilatation unterworfen wurde.

    2.-Aus Zeitdilatation und Längenkontraktion ergibt sich umgekehrt auch, dass aus der Sicht der Photonen Zeit=0 und Raum/Weg=0. Photonen sind demnach nichts anderes als ein teleportierter Impuls (als Teilchen kann man sie nicht auffassen). Sie haben keine Geschwindigkeit. Beobachter in Ruhesystemen nehmen diese Teleportation jedoch nicht wahr. Stattdessen erscheint für sie – stets im Einklang mit den Prinzipien von Zeitdilatation und Längenkontraktion – ein elektromagnetischer Lichtstrahl.

    3.-Eine Besonderheit der Photonen ist es, dass ihre Abbilder (Lichtstrahlen) universell in allen Ruhesystemen wahrnehmbar sind und überall die gleiche Geschwindigkeit haben (siehe 2. Satz der SRT). Aus den Versuchen der Quantenphysik kann man erkennen, dass mehr als nur eine Projektion entstanden ist – Urbild (teleportierter Impuls) und Abbild (Lichtstrahl) beeinflussen sich gegenseitig.

    Beispiel: Ein Polarisationsfilter. Der Impuls würde durch die Filteröffnung durchgehen. Er wird aber durch sein Abbild, die Lichtwelle, die nicht durch den Filter passt, zurückgehalten.

    4.-Wie ist nun Dein Beispiel im Artikel zu interpretieren? Hier ist es zunächst umgekehrt: Der Impuls steht vor einer Wahlmöglichkeit. Dabei würde ich von einem “Überschießen” der Lichtwelle nach beiden Richtungen sprechen, was dann zu Interferenzen führen kann. Letztlich wählt der Impuls “seine” Richtung und wird gemessen.

    Nach meinem Lösungsmodell breitet sich die Welle, die den Impuls begleitet, zunächst nach beiden Richtungen aus (offenbar kann die Welle mehrere Alternativen berücksichtigen, nicht jedoch der Impuls, der definitiv nur in eine Richtung stattfindet), wobei Welle und Impuls wie durch ein mechanisches System (vorzustellen als eine Art Gummiband) verbunden sind, das physikalisch auf der Natur der Zeitdilatation beruht. Auf den Begriff der “Messung” verzichte ich bis auf weiteres vollständig.

    Für andere Teilchen (Elektronen, Moleküle) ist die Situation weniger klar als für das Photon. Aber ich vermute, dass man einige der genannten Grundsätze auch hier verwenden kann. Der Wellencharakter sollte dabei stets analog als eine Beobachterperspektive betrachtet werden, die auf der Zeitdilatation beruht.

    Und hier meine Antwort auf die Frage: Ist die Wellenfunktion realistisch?
    Die Welle begleitet real den Photonimpuls. Sie kann auch Einfluss auf den Photonimpuls nehmen (am klarsten beim Polarisierungsfilter). Deine geschilderte Versuchsanordnung konstruiert ein Dilemma für das Impuls-Wellen-Paar, sozusagen eine Münze, die auf ihren Rand fällt, oder eine Kugel eines Nagelbrett-Zufallsgenerators, die auf einem Nagel stehenbleibt. Dieses Dilemma ruft bei der Welle merkwürdige Reaktionen hervor (man findet sie plötzlich im Überlagerungszustand beiderseits wieder), aber es handelt sich eben um eine provozierte Ausnahmesituation (halbdurchlässiger Spiegel) und nicht um ihr normales Verhalten.

    Dieser Überlagerungszustand betrifft jedoch stets nur die Welle, und niemals das Urbild, den Impuls (das “Teilchen”), dies hast Du auch in Deinem Artikel im Kapitel über Moleküle klargestellt. Auch der Kollaps einer Wellenfunktion führt nicht etwa zu “Welle wird Teilchen”, sondern Teilchen/Impuls und Welle bestehen als Urbild und Projektion unabhängig nebeneinander. Für esoterische Überlegungen besteht kein Bedarf.

  143. #143 MartinB
    8. August 2013

    @Rinaldo
    Ich habe das nicht wirklich verstanden, dafür waren die Erklärungen nicht präzise genug, aber das mit der begleitenden welle erinnert vage an die Pilotwellentheorie von Bohm/de Broglie.
    “wobei Welle und Impuls wie durch ein mechanisches System (vorzustellen als eine Art Gummiband) verbunden sind, das physikalisch auf der Natur der Zeitdilatation beruht. ”
    ??? Wie “beruht” ein mechanisches System auf der Zeitdilatation?

    “Auf den Begriff der “Messung” verzichte ich bis auf weiteres vollständig.”
    Dann wirst du Schwierigkeiten bekommen, Experimente wie “quantum eraser”, “delayed choice” usw. zu erklären.

  144. #144 Rinaldo
    8. August 2013

    @Martin
    Vielen Dank für Deine interessanten Anmerkungen, die mir erst einmal Futter zum Grübeln geben.
    Das Gute an Deinem anregenden Artikel war, dass er auf wenige einfache Phänomene beschränkt war. Deine Fragen und Anregungen gehen über meinen Erkenntnishorizont jedoch klar hinaus, so dass ich auf die Schnelle nicht vernünftig antworten kann:

    Bohm/ deBroglie: sehr detaillierte Ideen, die meinen Anschauungen entsprechen könnten. Der Ansatz ist jedoch nicht der gleiche (bei mir gibt es keine bewegten Teilchen, nur teleportierte Impulse).
    Mechanisches System: Ich schrieb “wie ein mechanisches System” – es handelt sich um meine Vermutungen beim Versuch der Deutung der Standardexperimente der Quantenmechanik.
    Quantum eraser: Damit muss ich mich noch beschäftigen – wann kommt Dein Artikel darüber? 😉

    Eine detailliertere Darstellung meiner Ausgangsüberlegungen findet sich auf https://www.quanten.de/forum/showthread.php5?t=2370 (deutsch) und https://cosmoquest.org/forum/showthread.php?141255-The-Twin-Paradox-as-a-Model-for-the-Wave-Particle-Duality (englisch)

  145. #145 MartinB
    9. August 2013

    @Rinaldo
    Ich sag’s mal hart aber ehrlich (und hoffe, du bist mir deswegen nicht böse): Wenn Dein Wissen über Quantenmechanik auf einem Niveau ist, dass du weder deBroglie/Bohm noch den Quantenradierer kennst, dann solltest du mit deiner Zeit vermutlich besser etwas anderes anfangen als zu versuchen, mal eben schnell als Hobby eine neue Deutung der Quantenmechanik zu erfinden, die letztlich auf einem Wortspiel beruht (dem Begriff “teleportieren” für die Tatsache, dass es kein Ruhesystem eines Photons gibt).
    Von jemandem, der nur weiß, dass ein Auto vier Räder hat und dass da irgendwo ein Motor drin ist, würdest du deine Karre auch nicht reparieren lassen.

    Aber was die Leute in den von dir verlinkten threads, die es mit viel Geduld versucht haben, nicht geschafft haben, werde wohl auch ich nicht erreichen. Innerhalb der QFT wird der Welle-teilchen-Dualismus letztlich ohnehin vollkommen befriedigend gelöst (das Messproblem bleibt allerdings).

  146. #146 Rinaldo
    9. August 2013

    @Martin
    Das hast Du sehr treffend und amüsant formuliert, bleibt allerdings etwas an der Oberfläche, und ich habe (wie Du bestimmt auch) keine Lust, auf Nebenkriegsschauplätzen zu diskutieren.

    Nur eine Klarstellung zu meiner Ehrenrettung: Der WTD mag für Physiker “vollkommen befriedigend gelöst sein”, wie Du sagst. Aber das Verständnis der breiten Öffentlichkeit ist gleich null, und das ist ein Problem. In dieser Situation habe ich gehandelt und ein Modell entwickelt, das ein grundsätzliches Verständnis ermöglicht. Photon-Teleportation ist nicht nur ein Wortspiel, sondern mathematisch erfassbare Realität. Danach erscheinen unzählige Beiträge über das Licht in Foren und anderswo (aus Unkenntnis und in Ermangelung einer Modellvorstellung) als haarsträubend falsch, gestellte Fragen sind plötzlich einfach zu beantworten.

    Du sollst Dir von Gott kein Bildnis machen. Ist Physik unsere neue Religion, in der man keine Modelle entwickeln darf?

    Allerdings gehe ich einen Schritt weiter und versuche, diese Theorie auch in den Tiefen der Quantenphysik auszutesten. Dass ich dabei leicht auf die Nase fallen kann, ist mir klar, aber wer nichts wagt, der nichts gewinnt. 😉

  147. #147 MartinB
    9. August 2013

    @Rinaldo
    Es ergibt keinen Sinn, ein “physikalisches” Modell nur mit dem Ziel zu machen, dass die Wahrheit für Laien zu schwer verdaulich ist; stattdessen muss man versuchen, die Theorien vereinfachend, aber nicht falsch, zu erklären (Man kann z.B. einen Blog schreiben 😉 ).

    Ein physikalisches Modell muss quantitativ nachprüfbare Vorhersagen machen – die QFT tut das mit Bravour; was dein “Teleportationsmodell” tatsächlich quantitativ kann (welche Formeln stecken dahinter?) ist ja nicht klar.

  148. #148 Rinaldo
    9. August 2013

    @Martin
    Meine Theorie ist noch gar nicht mal falsch – vgl. mein englischsprachiger Thread, wurde dort als schon bekannt und ohne Interesse beurteilt.

    Die dahintersteckenden Formeln sind wirklich zu einfach, als dass sie nicht bereits bekannt sein könnten.
    Es geht um Einsteins Zeitdilatations- (und Längenkontraktions-) formel, die ich einfach auch auf das Licht selbst anwende: T’ = T x √1-v2/c2 Alles was ich tue ist, diese Formel auf das Licht (v=c) anzuwenden. Damit erhalte ich eine zweite Zeit, die Eigenzeit des Lichts, die 0 ist. (Beispiel: Die vom Licht benötigte Zeit von der Sonne zur Erde beträgt T=8 Minuten und T’=0 Minuten). Die 0 Minuten sind die Eigenzeit des Lichts (des Photonimpulses), die 8 Minuten sind die Zeit, die alle Beobachter wahrnehmen (und die sehen kein Teilchen, sondern eine Lichtwelle).

    Mit dieser “Zweigleisigkeit” des Lichts kann man mehr Dinge erklären. Und das Licht wird nichts anderes als eine “remote momentum transmission” im Gegensatz zum Stoß, der eine “local momentum transmission” darstellt. Eine Teleportation reinster Güte, die sogar wie bei Perry Rhodan von einem elektromagnetischen “Lichtblitz” (Lichtwelle) begleitet wird.

  149. #149 Eberhard
    22. November 2014

    Hallo Martin,
    zunächst danke für obigen ausserordentlich klaren Text.
    Aber deswegen schreibe ich nicht.

    Ich habe vor einiger Zeit einige – jedenfalls für mich – interessante Texte über die Grundlagen der Physik gefunden
    die ich infolge mangelnder Sachkenntnis nicht beurteilen kann.
    Aus Deinen Blogs spricht Offenheit für neue Ideen.
    Ich schicke Dir im Folgenden eine Kostprobe – vielleicht spricht es Dich an.
    In diesem Sinne Alles Gute.

    Hallo Eberhard.
    Ich habe die Textwand gelöscht, spätestens das kleine C im Kreis am Ende deines zweiten Posts hätte dir sagen können, dass so etwas nicht geht. Du kannst hier gern links posten, aber bitte keine vollständigen und geklauten Texte von vielen Seiten Länge. – Martin B

  150. #150 Eberhard
    23. November 2014

    Hallo Martin,

    danke für das (den?) Feedback !

    Gut ,ich habe in Deinen Augen einen Fehler gemacht, aber Deine Antwort beantwortet nicht den Zweck meines Hinweises.

    Was hälst Du von den Thesen und Vorstellungen dieser Seiten. ??

    Hast Du keine Zeit oder Lust Dich damit zu befassen oder ist alles Deiner Ansicht nach Unsinn?

    Was sagt ein kompetenter Physiker dazu ?

    Das würde mich sehr interessieren.

    PS: Copyright:
    Meine Ansicht ist, dass das Copyright die grösst mögliche Entwicklungsbremse ist die denkbar ist.
    Die Sonne scheint schliesslich auch umsonst, nicht wahr.
    Ältere Artikel sollten unter Nennung des Autors frei sein.
    Schliesslich schöpft auch der Urheber aus der allgemeinen Quelle des Wissens, steht auf den Schultern seiner Vorgänger.

    Aber das ist natürlich nur meine unmassgebliche Meinung und dass Du mir jetzt nicht schlauerweise nur auf dieses PS antworterst – falls Du magst – statt auf obige Fragen 🙂

  151. #151 MartinB
    23. November 2014

    @Eberhard
    Ich habe den text nur überflogen, und fühlte mich sehr an die beliebten “Dialoge” zwischen dem klugen kreationistischen Schüler und dem dogmatischen Lehrer erinnert. (Vielleicht tue ich dem text damit Unrecht)
    Ich war beim Querlesen nicht in der Lage, zu verstehen, worum es da eigentlich gehen soll – alle Begriffe der Physik werden umdefiniert? Konzepte wie “Beschleunigung” sollen irgendwie nicht sinnvoll sein? Gibt es irgendwo mal eine Kurzfassung, aus der man entnehmen kann, was eigentlich das Thema ist, ohne sich durch einen mäßig gut geschriebenen Dialog wühlen zu müssen? (Und klugerweise bitte einen Link posten…)

    “Die Sonne scheint schliesslich auch umsonst, nicht wahr.”
    Und du arbeitest auch umsonst?

  152. #152 Eberhard
    23. November 2014

    OK Martin,

    jetzt begehe ich nochmal einen Fehler indem ich erneut etwas kopiere (oder “klaue” wie Du es nennst) , aber es ist wichtig, jedenfalls in meinen Augen.

    Lies es wenigstens bevor Du es wieder löschst.
    Der Link ist:
    https://www.tewari.org/index.html

    Ich habe weiter nach dem Autor dieser Artikel über die Fundamente der Physik gesucht und etwas gefunden.
    Besonders interessanz finde ich, dass er auch eine wie er schreibt funktionierende praktische Anwendung gebaut hat.
    Aber lies selbst:

    Paramahamsa Tewari was born on January 6, 1937, and graduated in Electrical Engineering in 1958 from Banaras Engineering College, India, and held responsible positions in large engineering construction organizations, mostly in Nuclear Projects of the Department of Atomic Energy, India.

    He was also deputed abroad for a year at Douglas Point Nuclear Project, Canada. He retired in 1997 from his position as Executive Nuclear Director, Nuclear Power Corporation, Department of Atomic Energy, India, and is the former Project Director of the Kaiga Atomic Power Project.

    Fundamentals of physics attracted Tewari’s imaginations right from the early school and college days. Over the last two decades his new ideas on the basic nature of space, energy, and matter have concretized into definite shape from which a new theory (Space Vortex Theory) has emerged. The theory reveals the most basic issue of relationship between space and matter precisely pinpointing that space is a more fundamental entity than matter. The physical significance of mass, inertia, gravitation, charge and light are revealed by extending the analysis in the theory beyond material properties and into the substratum of space, which again is broken down into fieldless voids, thus showing the limit to which a physical theory can possibly reach. The real universe is shown to be opposite to the current concepts of concrete-matter and empty space. The books that he has authored on Space Vortex Theory are:

    The Substantial Space and Void Nature of Elementary Material Particles (1977)
    Space Vortices of Energy and Matter (1978)
    The Origin of Electron’s Mass, Charge Gravitational and Electromagnetic Fields from “Empty Space” (1982)
    Beyond Matter (1984)
    Spiritual Foundations (1996)
    Universal Principles of Space and Matter (2002)

    He has lectured as invited speaker in international conferences in Germany, USA, and Italy on the newly discovered phenomenon of Space Power Generation.

    For the practical demonstration of generation of electrical power from the medium of space, Tewari has built Space Power Generators that operate at over-unity efficiency, thereby showing that the space medium indeed is the source of generation of basic forms of energy.

  153. #153 Eberhard
    23. November 2014

    Hallo Martin,
    die Sache ist ernst.

    Auf der zuvor erwähnten Seite gibt es auch Kritiken von namhaften Forschern. Wieder etwas geklautes, folgende Stellungnahmen:

    ———————–
    John Archibald Wheeler
    (often called ‘The father of the Black Hole”)

    J.A. Wheelers letters to P. Tewari:

    8/11/75 – Describes Tewari’s work as “majestic”

    2/1/77 – Mentions his work on “Polyelectrons” and how Sternglass, Schild, and most recently Tewari have renewed his interest.

    2/14/85 – Thanks Tewari for his book Beyond Matter and the pleasure of his association.

    Here are letters from Wheeler
    Letter from 8/11/75
    Letter from 2/1/77
    Letter from 2/14/85

    A review of J.A. Wheeler’s autobiography may be found here.

    Click here for a short review of his life’s work.
    —————
    Abus Salam
    Director, International Centre for Theoretical Physics
    International Atomic Energy Agency
    Trieste, Italy

    3/25/81 – Agrees that mass charge relationship is a key to understanding of physics.

    click here for a letter from Abus Salaam 3/25/81
    ———————
    Prof. Umberto Bartocci
    Universita’ di Perugia
    Dipartimento di Matematica e Informatica
    Via Vanvitelli
    06100 PERUGIA – ITALY
    Tel.: (39)-075-5002494
    Fax: (39)-075-5855024
    E-mail: bartocci@dipmat.unipg.it
    https://www.dipmat.unipg.it/
    This is a very singular book, in front of the current paradigm of contemporary
    Physics, a “cartesian-inspired” work which calls for reorientation in
    the foundations, by wisely warning that: The conclusion of the modern
    physics that absolute space, time, simultaneity, and space filling media
    are discredited ideas is certainly premature (p. 178). The books deals
    with arguments such as: Discovery of Charge and Mass Equations; Fundamental
    States of Cosmic Energy, Fields and Forces; Gravitation; Universal Constants;
    Motion of Electron; Atomic Structure; Light; Creation of Cosmic Matter
    (about Tewari’s physical conceptions see also the second section of this
    Episteme’s special issue).

    —————–

    Ja, meine Erkenntnisse stelle ich umsonst zur Verfügung.
    Meine Brötchen habe ich mir auf andere Weise verdient.

    Ich habe eine Nase für wichtige Entwicklungen und das könnte eine grosse Sache sein,

    Kennst Du das “Laterale Denken” von De Bono?
    Darin erklärt er folgendes

    Jedes Denksystem beruht auf Annahmen, Stabilen Daten, Axiomen, Evidenzen wie man es auch nennen mag.
    Darauf errichtet die Logik, das rationale Denken dann ein mehroder weniger imposantes Gebäude.
    Die Naturwissenschaft berucht auf eben solchen Annahmen und hat ein tolles Gebäude darauf errichtet.

    Wenn man aber die Annahmen untersucht kann man unter Umständen auf neuen Annahmen, Axiomen, Sichtweisen ein ganz anderes Gebäude errichten.

    Eben solche ganz andere Annahme, Sichtweise scheint mir hier vorzuliegen.
    Wenn man sich darauf einlässt könnte das zu ungeahnten neuen Durchbrüchen führen.

    Ich wünsche Dir, dass Du das erkennst.
    Freundliche Grüsse
    e.

  154. #154 MartinB
    23. November 2014

    @Eberhard
    Soweit ich sehen kann, ist diese “Theorie” Blödsinn.

    “For the practical demonstration of generation of electrical power from the medium of space, Tewari has built Space Power Generators that operate at over-unity efficiency, thereby showing that the space medium indeed is the source of generation of basic forms of energy.”
    Aber sicher – Energie aus dem Nichts kann die Theorie auch…

  155. #155 Eberhard
    23. November 2014

    Hallo Martin

    neue Erkenntniss

    Der Autor des Links
    https://www.reocities.com/CapeCanaveral/7986/&gt;
    ein amerikanischer Ingenieur
    ist nicht derselbe wie
    https://www.tewari.org/
    ein indischer Atomphysiker
    obwohl sie sich auf dieselben Erkenntnisse beziehen, soweit ich sehen kann.
    Un diese Sichtweise scheint praktische Erfolge zu zeitigen.

    Wenn Du nach kurzem Durchlesen, wie Du schriebst, nicht genau wusstest was der Autor eigentlich sagen wollte, so ist das nicht überraschend
    a.Weil es sich um ein völlig neuses Konzept handelt
    b. Uns unsere Denk-Gewohnheiten einen Streich spielen.

    In unserer Gesellschaft ist viel zu wenig über den Denkvorgang als solchen bekannt. Ich miene jetzt nicht Psychologie oder Gehirnüprozesse, sondern ganz praktisch die Art und Weise wie wir alltägliche kleine und grosse Probleme lösen.
    De Bono hat da eine wirklich bahnbrechende und weitgehd unbekannte Arbeit geleistet.

    Vielleicht war der von mir zu Deiner Bequemlichkiet geklaute Artikel nicht die richtige Auswahl aus den Artikeln die der Autor geschrieben hat.
    Es gibt noch andere, schau bei
    https://www.reocities.com/CapeCanaveral/7986/&gt;
    Es lohnt sich, wenn man die Physik auf ein neues Fundament stellen will.

    So wie ich das sehe, wurden die Artikel zuerst bei “geocities” gepostet. dann wurde diese Plattform geschlossen und ein verdienstvoller Geist übernahm wertvolle Artikel und stellte sie auf einer neuen Plattform “reocities” zur Verfügung.

    Daraus schliesse ich, dass der Autor, aus Altersgründen nicht weiter an seinen Texten interessiert ist.

  156. #156 MartinB
    23. November 2014

    @Eberhard
    “Wenn Du nach kurzem Durchlesen, wie Du schriebst, nicht genau wusstest was der Autor eigentlich sagen wollte”
    Das bezog sich auf den von dir geposteten Dialog. Ich habe aber mal einen der “Artikel” von tewari direkt angeschaut – darin werden teilchen als Wirbel in der Raumzeit angesehen und dann alle möglichen Formeln jongliert. Dumm nur, dass Elektronen sich nur mit den Mitteln der QFT sinnvol beschreiben lassen, die wird dort gar nicht erwähnt, so weit ich sehen kann.

    “Es lohnt sich, wenn man die Physik auf ein neues Fundament stellen will.”
    Nur dann, wenn das neue Fundament auch korrekt ist.

    Und nein, ich habe nicht die Zeit (oder Lust), mir diese Arbeiten ausführlich anzusehen. Warum nicht? Weil ich etwa einmal im Monat eine mail (oder einen Kommentar) von jemandem bekomme, der eine ganz neue bahnbrechende viel einfachere und revolutionäre Physik entdeckt haben will. (Dein Tewari ist kein Einzelfall, sondern einer von Vielen, wie es scheint.) Ich habe nicht die Zeit, jede dieser Theorien anzugucken und nach Fehlern zu suchen.

  157. #157 Eberhard
    23. November 2014

    @Martin

    @Eberhard
    Soweit ich sehen kann, ist diese “Theorie” Blödsinn.

    “For the practical demonstration of generation of electrical power from the medium of space, Tewari has built Space Power Generators that operate at over-unity efficiency, thereby showing that the space medium indeed is the source of generation of basic forms of energy.”

    Aber sicher – Energie aus dem Nichts kann die Theorie auch…

    Einstein:
    Neue Theorien setzen sich nicht dadurch durch, dass die Träger der bisherigen Abschauung umlernen, sondern dadurch dass sie aussterben (sinngemäss zitiert)

    Warum ist das Umlernen so schwer?
    Abgesehen von materiellen Interessen (wer riskiert gerne schon eine angesehene Stellung)
    liegt das auch an der Funktweise unseres Denkapparats.
    Bisherige Vorstellungen wirken als Filter.

    Zunächst einmal ist das “Nichts” nicht nichts, sondern etwas was mit den gegenwärtigen Messinstrumenten nicht erfassbar ist. siehe schwarze Materie.
    Wie also kann man eine Aussage über etwas machen, das man nicht erfassen kann, jedenfalls nicht mit dem gegenwärtigen Fundament der Physik.

    Und wenn dann jemand kommt mit einem möglicherweise anderem Fundament, so bringen einem die bisherigen Denkmuster dazu, sich erst garnicht damit zu beschäftigen.

    Die Herrn Physiker sollten sich alle mal mit dem Lateralen Denken befassen und die Begrenzungen ihres Weltbildes erkennen. Damit meine ich, dass JEDES Weltbild auf Annahmen beruht (Physik: “Was man nicht messen kann gibt es nicht,” neben andern) und das Kreativität darin besteht diese Annahmen bewusst zu manchen und zu verändern.

  158. #158 Eberhard
    23. November 2014

    @Martin

    danke für Deine freundliche Antwort, die ich gut verstehen kann.
    Also bleibt alles beim Alten

    Wünsche Dir noch viel Erfolg und Freude bei Deiner Arbeit.

    e.

  159. #159 MartinB
    23. November 2014

    @Eberhard
    Nicht alle wichtigen Zitate sind von Einstein.

    ” Damit meine ich, dass JEDES Weltbild auf Annahmen beruh”
    Echt jetzt? Wer hätte das gedenkt? Darüber hat sicher noch nie ein Physiker nachgedacht.

    “Und wenn dann jemand kommt mit einem möglicherweise anderem Fundament, so bringen einem die bisherigen Denkmuster dazu, sich erst garnicht damit zu beschäftigen.”
    Nein, es ist simpler. Wenn jeden Monat einer kommt, der meint, ich könnte doch mal ein paar Stunden inverstieren, um seine Lieblingstheorie zu prüfen, dann kann ich das schlicht nicht leisten.
    Deswegen haben wir so Sachen wie Fachzeitschriften etc.

  160. #160 Standre
    18. April 2015

    Was hältst du in diesem Zusammenhang von der Definition des “Da-Seins” als

    “Informationen, die sich gegenseitig in ihrer Existenz hervorrufen?”

    Das könnte auch ein Ansatz sein, das Geist-Materie-Dilemma aufzulösen.

  161. #161 MartinB
    19. April 2015

    @Standre
    Das ist mir zu unscharf und unklar (sprich: ich verstehe nicht, was da gemeitn sein soll): Informationen gib tes nicht ohne ein Medium, das die Information trägt; wie können sich Informationengegenseitig hervorrufen (in einer Rückkopplungsschleife a la Hofstadter?)?

  162. #162 Pascal S
    13. Juni 2015

    Wann genau findet eine Messung statt?

    Ich mein, wenn kein ‘Mensch’ das Messergebnis abliest, dann koennte sich doch das Messergebnis des Messgeraetes selbst in einem Überlagerungszustand befinden und erst durch die Realisierung durch Bewusstsein (siehe Von Neumann–Wigner Interpretation der QM) findet die eig. Messung statt. Dies wuerde meines Wissens nach auch das sog. Messproblem loesen.

    Koennte man eventuell in Betracht ziehen, dass Bewusstsein der notwendig subjektive Gegenspieler des doch eher objektiven Seins (nicht zwingend ausschließlich Materie) ist und das eine ohne das andere nicht existent sein kann?

    Klar gab es, basierend auf unserem Verstaendnis ueber die Geschichte des Universums, wohl Zustaende ohne bewusstes Leben. Ist aber Vergangenheit eine abstrakt gesehen hoehere Wahrheit als die Realisierung des Moments, welche wir, denke ich, jetzt gerade erfahren, in diesem Augenblick?

    Raumzeit ist ‘lediglich’ eine Ausbreitung (Dimension) und nicht das Maß aller Dinge und abstrakt betrachtet eingebettet in ‘hoeheren’ Dimensionen.

    Selbst der Gedanke eines absoluten Anfangs (Big Bang) und der eines absoluten Endes, waere ‘lediglich’ ein Gedanke unter zeitlichem Aspekt.

    free from permanence and non-existence

  163. #163 Pascal S
    13. Juni 2015

    Hier noch eine Beschreibung von Karl Jaspers zur Subjekt-Objektspaltung:

    „Allen […] Anschauungen ist eines gemeinsam: sie erfassen das Sein als etwas, das mir als Gegenstand gegenübersteht, auf das ich als auf ein mir gegenüberstehendes Objekt, es meinend, gerichtet bin. Dieses Urphänomen unseres bewußten Daseins ist uns so selbstverständlich, daß wir sein Rätsel kaum spüren, weil wir es gar nicht befragen. Das, was wir denken, von dem wir sprechen, ist stets ein anderes als wir, ist das, worauf wir, die Subjekte, als auf ein gegenüberstehendes, die Objekte, gerichtet sind. Wenn wir uns selbst zum Gegenstand unseres Denkens machen, werden wir selbst gleichsam zum anderen und sind immer zugleich als ein denkendes Ich wieder da, das dieses Denken seiner selbst vollzieht, aber doch selbst nicht angemessen als Objekt gedacht werden kann, weil es immer wieder die Voraussetzung jedes Objektgewordenseins ist. Wir nennen diesen Grundbefund unseres denkenden Daseins die Subjekt-Objekt-Spaltung. Ständig sind wir in ihr, wenn wir wachen und bewußt sind.“

  164. #164 MartinB
    13. Juni 2015

    @Pascal
    “Koennte man eventuell in Betracht ziehen”
    Ja, kann man in Betracht ziehen. Ich halte das allerdings für arg problematisch, zum einen, weil Bewusstsein nach allem, was wir wissen, ein natürlich emergentes Phänomen ist (es sei denn, man wäre Dualist, aber vom Dualismus halte ich nix), zum anderen, weil Bewusstsein nichts binäres ist. Wäre Bewusstsein für den Kollaps der WF verantwortlich, dann könnte man experimentell exakt den Moment bestimmen, in dem z.B. ein Mensch Bewusstsein erlangt, während ja alles dafür spricht, dass das ein schrittweiser Prozess ist.

    Das mit der Raumzeit ist mir zu schwammig, damit kann ich nix anfangen.

  165. […] Spiegel schickt. Diesen Aufbau habe ich schon vor langer Zeit mal benutzt, um die Seltsamkeiten der Quantenmechanik (QM) zu […]

  166. #166 Patrick
    München
    12. September 2015

    @MartinB
    Ich möchte an der Stelle, Bewusstsein wäre eine rein emergente Erscheinung etwas beifügen. Roger Penrose schließt in seinen Büchern “Computerdenken” sowie “Schatten des Geistes” genau diese Ansicht aus. Er begründet dies dadurch, das nicht-rechnerische Prozesse dem Bewusstsein zugrunde liegen müssen (also alles was mit einer Turing-Maschine nicht ausführbar ist). Da sich das Wechselspiel milliarder Neuronen bis auf einen kleinen Zufallsanteil sehr gut klassisch beschreiben lässt, können die Neuronen nicht für Bewusstsein alleine Verantwortlich sein. Er stellt eine sehr interessante Theorie auf, indem eine durch Mikrotubuli erzeugte Quantenkohärenz die Grundlage bildet. Damit würde unser derzeitiges Bild über Neuronen und Bewusstsein nicht stimmen.

  167. #167 MartinB
    13. September 2015

    @Patrick
    Ja, Penroses Bücher habe ich gelesen. Seine Behauptung (und mehr ist es nicht), das bewusstsein könne Dinge tun, die eine Turingmaschine nicht tun kann, halte ich für falsch – das was er als Argumente anführt, ist wahrlich nicht überzeugend. Die Hypothese mit den Mikrotubuli hat immerhin den Vorteil, überprüfbar zu sein – soweit ich sehe, ist sie inzwischen dadurch arg fraglich geworden, dass der von Penrose ebenfalls damit in Verbindung gebrachte Kollaps der WF durch Gravitonen so wohl nicht funktioniert, dem stehen neuere Experimente entgegen.
    Ich fand das Buch in diesen teilen extrem enttäuschend – dafür sind aber die Erklärungen zu Thermodynamik und Allg. RT im ersten Buch ganz großartig.

  168. #168 Andy
    Leipzig
    27. November 2015

    “Ein radioaktiver Atomkern hat also nicht mal eine eindeutige Identität, wenn wir ihn nicht messen – er liegt in einem Überlagerungszustand aus zwei unterschiedlichen Identitäten vor.”
    jetzt weiß ich warum das Atomzeugs so gefährlich ist. Wenn es sich ständig in einer Identitätskrise befindet, dabei auch noch schizophrene Züge annimmt sollte man besser vorsichtig sein. Bis zu diesen Zeilen für einen Laien wie mich sehr verständlich 🙂 Sehr schön. Danke dafür.

  169. #169 Helmut Wiedemann
    4. September 2016

    Dass Realität vom Beobachter unabhängig ist oder sein soll, gilt nur bei einer materialistischen Weltanschauung.
    Sorry, so allgemeingültig sind Ihre Anschauungen nicht.

  170. #170 Helmut Wiedemann
    4. September 2016

    Nachdem ich Ihren Artikel zu Ende gelesen habe, hat sich das Problem selbst gelöst. Sie haben selbst auf die Schwierigkeiten hingewiesen.
    Und zu einer eindeutigen Meinung kann ich mich nicht entscheiden.

  171. #171 Helmut Wiedemann
    4. September 2016

    Nach dem Durchlesen aller Beiträge bin ich zu der Meinung gekommen, dass viele Diskussionen sprachlicher Natur sind. Ich meine , sie beruhen auf den unterschiedlichen Auffassungen über Begriffe. Besonders Begriffen, die nicht physikalisch sind, z.B. Realität. aber es ist legitim darüber zu reden, weil das uns auf den Nägeln brennt. Was noch interessant ist, dass viele Fragen immer wieder auftauchen. Ganz neue Ideen gibt es nur selten.

  172. #172 Eugen
    7. April 2017

    “3. Wo ist das Molekül?

    Statt mit Elektronen kann man dasselbe Spiel auch mit größeren Objekten machen. Wie groß? Das ist eine zur Zeit heiß diskutierte Frage, aber zumindest mit ganzen Molekülen klappt das Experiment in ganz ähnlicher Weise, wie ich neulich erklärt habe.”

    Warum kann man das nicht weiter fortführen? Immer größere Moleküle nehmen, bis es nicht mehr klappt?

  173. #173 Eugen
    7. April 2017

    Kann ich mir das so vorstellen, dass sich z.B. ein Elektron, welches sich gerade auf den Doppelspalt zubewegt, dann gibt mir die Wellenfunktion zu jedem Zeitpunkt die Wahrscheinlichkeiten an, wo es sich gerade befinden könnte.
    Wenn ich nun aber ein größeres, zusmmengesetztes Teilchen habe, welches sich gerade auf den Doppelspalt zubewegt, hat dann jedes Teilchen, aus denen das zusammengesetzte Teilchen besteht, seine eigene Wellenfunktion? Ab irgendeiner Teilchengröße sind dann die Abweichungen der einzelnen Wellenfunktionen so groß, dass es zusammengesetzt keine Welle mehr bleiben kann und hier wird aus den Wellen ein Teilchen?

  174. #174 MartinB
    7. April 2017

    @Eugen
    Man kann das technisch nicht so einfach durchführen, weil die Experimente extrem knifflig sind und immer schwieriger werden, je größer die Moleküle sind.

    Was die zusammengesetzten Teilchen angeht, kann man sich das am einfachsten so vorstellen, dass die einzelnen Bestandteile zwar alle eine eigene Wellenfunktion haben, dass die aber (wegen der Bindung aneinander) alle im Grundzustand sind und nur eine kollektive Anregung (sprich: eine gemeinsame Bewegung) möglich ist. Die Vorstellung, dass es eine maximalgröße gibt, ober halb derer man keine Gesamt-WF bilden kann, passt in meinen Augen nicht.

  175. #175 Anonym_2018
    12. August 2018

    Halte ich die Wellenfunktion für ein reales Objekt, dann kann sich diese Wellenfunktion nichtlokal ändern …

    Könnte das folgende möglich sein?

    Die Welle, die durch die Wellenfunktion mathematisch beschrieben wird, ist ein reales Objekt, das sich durch Schwingungen der virtuellen Quantenobjekte im Vakuum ausbreitet. Durch Emmision des zugeordneten Teilchens ins Vakuum hinein entsteht diese Welle. Ohne diese Welle könnte z.B. das Doppelspalt-Experiment nicht funktionieren. Bei der Absorbtion wandelt sich die Energie dieser Welle wieder in ein Teilchen um, das gleiche Eigenschaften hat, wie das emmitierte Teilchen. Die Wahrscheinlichkeit, das Teilchen an einem bestimmten Ort aufzufangen, ist proportional zum Quadrat der Wellenamplitude. Durch die Entropieerhöhung bei der Messung kollabiert die Welle (unabhängig von der Beobachtung des Messergebnisses). Alle physikalischen Vorgänge, bei denen die Entropie nicht größer wird (also bis unmittelbar vor der Entropieerhöhung durch die Messung), unterliegen nicht der Kausalität in der SRT, d.h. eine Ursache muss nicht zeitlich vor der Wirkung liegen (Grund: der Zeitpfeil der Thermodynamik zeigt in die Zeitrichtung, in der die Entropie zunimmt.). Dadurch sollte auch ein Verschränkung möglich sein.

  176. #176 MartinB
    12. August 2018

    @Anonym_2018
    ” Bei der Absorbtion wandelt sich die Energie dieser Welle wieder in ein Teilchen um, das gleiche Eigenschaften hat, wie das emmitierte Teilchen. ”
    Das ist dann aber wieder der Kollaps – nichtlokal und instantan – und spätestens beim fall mit der Verschränkung wird es schwierig.
    Klar, wenn man bereit ist, anzunehmen, dass es bezugssysteme geben darf, in denen die Ursache nach der Wirkung liegt, kann man das tun – aber wieso da der zeitpfeil eine Rolle spielen soll (oder die Entropie), sehe ich nicht.

  177. #177 Anonym_2018
    12. August 2018

    Das ist dann aber wieder der Kollaps – nichtlokal und instantan – und spätestens beim fall mit der Verschränkung wird es schwierig.

    Könnte das neu entstandene Teilchen nicht mit dem ursprünglichen gleichartigen Teilchen über die zugeordnete Welle über zeitliche und räumliche Entfernung verschränkt sein (ähnlich wie in einem Supraleiter die Cooper-Paare über Phononen über räumliche Entfernung)?

    aber wieso da der zeitpfeil eine Rolle spielen soll (oder die Entropie), sehe ich nicht.

    Das war missvertändlich von mir formuliert. Der Zeitpfeil spielt gerade keine Rolle, solange die Entropie gleicht bleibt, daher müsste dann die Ursache zeitlich nach der Wirkung liegen können:

    Danach gehen Ursachen ihren Wirkungen stets voraus. Der kausale Zeitpfeil ist ein Postulat, das das alltägliche Erleben widerspiegelt. Es ist jedoch nicht klar, ob Kausalität zwingend ist oder erst durch Wahrnehmung generiert wird.

    Quelle:
    https://de.wikipedia.org/wiki/Zeitpfeil#Kausal

    Ich halte die Gleichungen der Relativitätstheorie und der Quantentheorie alle für richtig, weil sie experimentell bestätigt wurden. Aber ich habe den Eindruck, das ist Mathematik ohne Physik (z.B. Raumkrümmung, wo kein physisches Objekt ist, Wellengleichung ohne Welle …) und daher unvollständig. Vielleicht liegt die Lösung in den Eigenschaften des Vakuums.

  178. #178 MartinB
    12. August 2018

    @Anonym
    Das mit den neuen und alten Teilchen verstehe ich nicht.

    Wenn du akzeptierst, dass Ursachen nach ihren Wirkungen liegen können, dann kannst du natürlich den Kollaps in irgendeinem Bezugssystem festmachen. Das widerspricht dann halt nur unseren üblichen Vorstellungen von Kausalität.

    ” Aber ich habe den Eindruck, das ist Mathematik ohne Physik (z.B. Raumkrümmung, wo kein physisches Objekt ist, Wellengleichung ohne Welle …) und daher unvollständig.”
    Raumkrümmung, wo kein Objekt ist, ist nichts anderes als elektrische Felder ohne Materie – einmal gibt es Lichtwellen, einmal Gravitationswellen. Ich sehe da kein Problem. Dass die Natur für unseren verstand nicht immer leicht zu erfassen ist, heißt nicht, dass da mit der Physik was nicht stimmt…

  179. #179 Anonym_2018
    12. August 2018

    einmal gibt es Lichtwellen, einmal Gravitationswellen. Ich sehe da kein Problem

    Da sehe ich das gleiche Problem: Wellengleichung ohne Welle. Meiner Ansicht nach braucht man einen Äther (keinen klassischen, sonderen einen relativistischen und quantenmechanischen), um die Wellen zu übertragen.

    Ein emmitiertes Photon kann ja das Vakuum (wiederholt) polarisieren. Durch die erzeugten virtuellen Elektronen und Positronen müssten elektromagnetische Schwingungen entstehen, die die Welle weiterleiten (ähnlich wie vertikate Schwingungen von Wassermolekülen eine Wasserwelle auf der Oberfläche eines Sees weiterleiten).

    nichts anderes als elektrische Felder ohne Materie

    Dafür müsse das gleiche gelten, denn elektrische Felder sind Lichtwellen der Frequenz 0, d.h. virtuelle Photonen (W=h*f=0).

  180. #180 MartinB
    12. August 2018

    @Anonym
    “Meiner Ansicht nach braucht man einen Äther (keinen klassischen, sonderen einen relativistischen und quantenmechanischen), um die Wellen zu übertragen.”
    Warum? Wir wissen, dass Licht aus Photonen besteht, Materie aus Elektronen und Quarks. Beide werden durch Quantenfelder beschrieben – wenn wir akzeptieren, dass gewöhnliche materie sich im leeren Raum aufhalten kann, dann auch Licht.

    Und sorry, ich hätte elektromagnetische Felder schreiben sollen – ändert nicht viel, wie gesagt, so oder so haben wir ein Quantenfeld, egal ob das Materie oder em-Wellen oder E-Felder beschreibt.

  181. #181 Anonym_2018
    12. August 2018

    Wir wissen, dass Licht aus Photonen besteht

    Das wissen wir nur am Ort der Emmission und am Ort der Absorbtion (Messung). Da das Vakuum polarisierbar ist, könnte das Licht zwischendurch als Welle via “Äther” übertragen werden:
    Emmission (z.B: LED) -> Photon -> Welle via “Äther” -> Photon -> Absorbtion (z.B. CCD-Sensor).

    Ist das Gegenteil beweisbar?

    > Früher, als über die Struktur des Vakuum nichts bekannt war, hat man einen Äther angenommen.
    > Heute weiss man, dass das Vakuum nicht leer ist, aber man geht von Wellen ohne Äther aus.
    -> Das finde ich merkwürdig.

  182. #182 MartinB
    13. August 2018

    @Anonym
    In der Physik ist nichts beweisbar. Wir haben die Theorie der Quantenelektrodynamik,die Photonen als Anregung eines Quantenfelds beschreibt und die exzellent experimentell bestätigt ist. Wer eine alternative Theorie aufstellen will, kann das ja tun, muss aber dieselben Vorhersagen machen können.

    Physik ist die Disziplin, Theorien aufzustellen, die korrekte Vorhersagen machen, nicht die Disziplin, Theorien aufzustellen, die unserer Intuition gut passen.

    Und wie gesagt – es besteht kein wirklich fundamentaler Unterschied zwischen Licht und Materie. Wer akzeptiert, dass Materie im Vakuum sein kann, sollte das auch für Licht akzeptieren, sonst wird es ein wenig inkonsistent.

  183. #183 Anonym_2018
    2. September 2018

    Wer akzeptiert, dass Materie im Vakuum sein kann, sollte das auch für Licht akzeptieren, sonst wird es ein wenig inkonsistent.

    .. und wer das nicht für Licht akzeptiert, sollte das auch nicht für Materie akzeptieren.

    Das Doppelspaltexperiment (s. Kommentar #175) kann man nicht nur mit Lichtwellen, sondern auch mit den Materiewellen der Elektronen durchführen. Am Ort der Messung (=Entropieerhöhung) kollabiert die Materiewelle, und deren Energie wandelt sich dadurch wieder in ein (nahezu punktförmiges) Materie-Teilchen um.

  184. #184 MartinB
    2. September 2018

    @Anonym2018
    Mit Entropieerhöhng hat das erst mal gar nichts zu tun, oder allenfalls sehr wenig und sehr spekulativ…

  185. #185 Anonym_2018
    2. September 2018

    Bei der Messung eines Quantenobjekts tritt es in Wechselwirkung mit seiner Umgebung (z.B. Dopplespaltexperiment: Auftreffen einzelner Photonen auf der Fotoplatte).

    Dekohärenzeffekte ergeben sich, wenn ein bislang abgeschlossenes System mit seiner Umgebung in Wechselwirkung tritt. Dadurch werden sowohl der Zustand der Umgebung als auch der Zustand des Systems irreversibel verändert.

    Quelle:
    https://de.wikipedia.org/wiki/Dekoh%C3%A4renz

    Irreversible Veränderung = Entropieerhöhung (Zeitpfeil)

    Die Dekohärenztheorie kann jedoch keine einzelnen Messungen erklären, sondern macht nur statistische Aussagen über Ensembles aus mehreren Messvorgängen.

    Quelle:
    https://de.wikipedia.org/wiki/Dekoh%C3%A4renz#Dekoh%C3%A4renz_und_Messproblem

  186. #186 MartinB
    2. September 2018

    @Anonym_2018
    “Irreversible Veränderung = Entropieerhöhung (Zeitpfeil)”
    Nicht wirklich, oder jedenfalls nicht so einfach. Man erklärt den normalen zeitpfeil über die Erhöhung der Entropie, aber das ist reine statistische Mechanik und hat mit dem Messprozess erst mal nichts zu tun, das funktioniert in der klassischen Physik genauso.
    Zweitens ist die Veränderung der WF zwar irreversibel, aber da die Wf ja nicht beobachtbar ist, sehe ich nicht, wie das direkt was mit der Entropie zu tun hat.
    Man kann da natürlich Verbindungen herstellen, ich galube, dazu hat Penrose ein paar Ideen in “Road to Reality” (ist ne Weile her, dass ich das gelesen habe), aber so ganz offensichtlich und direkt ist die Verbindung zwischen den beiden nicht, soweit ich sehe.

    Ein paar weitere Ideen dazu gibt es auch hier, wie immer by quora mit etwas Vorsicht zu lesen:
    https://www.quora.com/Is-wave-function-collapse-or-decoherence-an-irreversible-process-If-so-is-it-the-mechanism-behind-the-increasing-entropy-in-our-universe

  187. […] zu bekommen. (Ja, das ist ziemlich seltsam. Mehr über diese Seltsamkeiten findet ihr hier und […]